You are on page 1of 142

1/1

The Official SAT Online Course


Help | Profile | My Organizer | My Bookmarks | Logout

Answers and Explanations

Back to Score Report

Test Sections

Section 1

Essay

Section 2

Online - Practice Test #4

Section 4

Section 5

Section 6

Section 7

These sample essays were originally handwritten by students but are shown typed here for ease
of reading. The essays are displayed exactly as students wrote them, without any corrections to
spelling, punctuation, or syntax. One handwritten sample essay is provided to illustrate the need
for legible and clear handwriting.

Section 8

Section 9

Section 10

Exemplars:

Essay Prompt

Think carefully about the information presented in the following excerpt and the assignment
below.

re

People's lives are the result of the choices they makeor fail to make. The path one takes in
life is not arbitrary. Choices and their consequences determine the course of every person's
life. All people, whatever their circumstances, make the choices on which their lives depend.

Are people's lives the result of the choices they make? Plan and write an essay in which you
develop your point of view on this issue. Support your position with reasoning and examples
taken from your reading, studies, experience, or observations.

te

is

eg

Copyright 2006 The College Board. All rights reserved.

Back to Score Report

Privacy Policy

Terms of Use

Contact Us

nR

file://E:\\d2.htm

2006-11-12

1/18

The Official SAT Online Course

Help | Profile | My Organizer | My Bookmarks | Logout

Answers and Explanations

Back to Score Report

Test Sections

Section 1

View Answers and Explanations

Section 2

Online - Practice Test #4

Section 4

Section 5

how much will

reduced by

Section 6

Section 7

(A)

Section 8

(B)

Section 9

If the cost per notebook is

equally priced notebooks is

The total cost of

of these notebooks cost at the new rate?

(C)

Section 10

(D)
(E)

AND EXPLANATIONS

re

Explanation for Correct Answer A :


Choice (A) is correct. Since the total cost of

equally priced notebooks was

the original cost per notebook was

originally

ANSWERS

The

so the new cost of each notebook is

cost per notebook is reduced by

te

notebooks at the new rate is

Therefore, the cost of

is

eg

Explanation for Incorrect Answer B :

Choice (B) is not correct.

is the cost of

nR

but

is the cost of

notebooks at the original price,

notebooks at the new rate.

Explanation for Incorrect Answer C :

Choice (C) is not correct.

minus

is the cost of

notebooks at the original price,

for each of the

But the reduction in price is

Thus, the total cost of

notebooks:

notebooks at the new

rate is

Explanation for Incorrect Answer D :

Choice (D) is not correct.

is the cost of

but the question asks for the cost of

notebooks at the original price,

notebooks at the new rate, which is

Explanation for Incorrect Answer E :

Choice (E) is not correct.

minus

the new rate,

is the cost of

notebooks at the original price,

however, to find the cost of

must be subtracted from the cost of

notebooks at

notebooks at the

original price:

which of the following could be a value of

If
(A)
(B)
(C)

file://E:\\d3.htm

2006-11-12

2/18

The Official SAT Online Course

(D)
(E)

AND EXPLANATIONS

ANSWERS

Explanation for Correct Answer A :


could be
Choice (A) is correct. The value of

since

Explanation for Incorrect Answer B :


cannot be
Choice (B) is not correct. The value of

since

not

Explanation for Incorrect Answer C :


cannot be
Choice (C) is not correct. The value of

since

not

Explanation for Incorrect Answer D :


cannot be
Choice (D) is not correct. The value of

ed

since

not

te
r

Explanation for Incorrect Answer E :


cannot be
Choice (E) is not correct. The value of

not

since

is

The digits of the positive three-digit integer

and

are

How many possible

nR
eg

values are there for


(A)

Three

(B)

Four

(C) Six

(D) Eight

(E)

Nine

ANSWERS

AND EXPLANATIONS

Explanation for Correct Answer C :


Choice (C) is correct. For the positive three-digit integer

there are three

Once the hundreds digit is


or
possibilities for the hundreds digit: it can be
determined, there are two possibilities for the tens digit. After the tens digit is
determined, there is only one possible value for the units digit. Therefore, there are

different possible three-digit numbers

with digits

and

An alternative solution would be to list and count all possible values of

and

Explanation for Incorrect Answer A :


Choice (A) is not correct. The possible values for
there are six possible values for

each have three digits, but

Explanation for Incorrect Answer B :


Choice (B) is not correct. There are more than four possible three-digit integers
and
They are
and
with digits

Explanation for Incorrect Answer D :

file://E:\\d3.htm

2006-11-12

3/18

The Official SAT Online Course

Choice (D) is not correct. This incorrect answer could be the result of using

to count the possible values for

instead of

Explanation for Incorrect Answer E :

Choice (E) is not correct. This incorrect answer could be the result of using

to count the possible values for

instead of

is the value of

te
re
d

(A)
(B)

(C)

(D)

(E)

is

ANSWERS

AND EXPLANATIONS

Explanation for Correct Answer E :


and
Choice (E) is correct. Since

eg

follows from the figure that

and

it follows

Since

Therefore,

nR

so

it

each have measure

which gives

that

what

If

each have measure

and

In the figure above,

Explanation for Incorrect Answer A :


Choice (A) is not correct. The measure of

measure of

is equal to

it follows from the figure that

is the

Since

Therefore,

cannot be

equal to

Explanation for Incorrect Answer B :


Choice (B) is not correct. The measure of

measure of

is equal to

it follows from the figure that

Since

Therefore,

is the

cannot be

equal to

Explanation for Incorrect Answer C :


and
Choice (C) is not correct. Since

follows from the figure that

Since

it follows that

and

each have measure

it

Thus,

not

Explanation for Incorrect Answer D :


Choice (D) is not correct. This choice,

could be the result of visually estimating

but the question does not ask for an estimate.

file://E:\\d3.htm

2006-11-12

4/18

The Official SAT Online Course

are added together, the


and
In the figure above, if the coordinates of points
result will be the coordinate of a point between which two consecutive integers?
(A)

and

(B)

and

(C)

and

(D)

and

(E)

and

ANSWERS

AND EXPLANATIONS

Explanation for Correct Answer C :


Choice (C) is correct. The coordinate of point

is approximately

and the

Therefore, if the coordinates of points


is approximately
coordinate of point
are added together, the result will be approximately
and

which is between

and

ed

Explanation for Incorrect Answer A :


Choice (A) is not correct. The coordinate of point

is greater than

Therefore, this result cannot be between

result must be greater than

er

the

and

is added to the

the result will be the coordinate of a point located

coordinate of point

is

is
t

Explanation for Incorrect Answer B :


Choice (B) is not correct. When the coordinate of point

approximately

If the

which is positive, is added to the coordinate of point

coordinate of point

units to the right of point

units to the right of

nR
eg

on the number line, because point

Therefore, this result cannot be between

Explanation for Incorrect Answer D :


Choice (D) is not correct. The coordinate of point

and

is negative with absolute value

greater than the coordinate of point

which is positive. This implies that the sum

and

must be negative and, therefore, cannot be

of the coordinates of points


and
between

Explanation for Incorrect Answer E :


is negative, and the coordinate
Choice (E) is not correct. The coordinate of point
This implies that the sum of the coordinates of points
is less than
of point
and
Therefore, this sum cannot be between
must be less than
and

and each number after the first is


In a sequence of numbers, the first number is
times the preceding number. What is the third number in the sequence?
more than
(A)

(B)

(C)

(D)

(E)

ANSWERS

AND EXPLANATIONS

Explanation for Correct Answer E :


Choice (E) is correct. The second number in the sequence is equal to

and the third number in the sequence is

file://E:\\d3.htm

2006-11-12

5/18

The Official SAT Online Course

Explanation for Incorrect Answer A :


Choice (A) is not correct. This choice,

is too small. The second and third

not

and

numbers in the sequence are

and

Explanation for Incorrect Answer B :


is the second number in the sequence.
Choice (B) is not correct. This choice,
The question asks for the third number.

Explanation for Incorrect Answer C :


Choice (C) is not correct. This choice,

is the third number in a different

times the
less than
sequence: one where each number after the first is
preceding number. The question asks for the third number where each number
times the preceding number.
more than
after the first is

Explanation for Incorrect Answer D :


Choice (D) is not correct. The third number in the sequence is

not

choice could be the result of mistakenly computing the second number as


times the first.
more than
times the first, instead of
than

This

less

re
d

te

is

eg

hours to drive from Ashton to Farley, passing through Belville, Clinton,


It took Kia
Dryden, and Edgewood on the way. The graph above shows where she was along the
hours of the trip. According to the graph, approximately how long,
route during the
in hours, did it take Kia to drive from Dryden to Edgewood?

nR

(A)

(B)

(C)

(D)
(E)

ANSWERS

AND EXPLANATIONS

Explanation for Correct Answer A :


Choice (A) is correct. According to the graph, Kia reached Dryden approximately

hours after she began her trip. She was in Edgewood approximately

hours after the beginning of her trip. Therefore, it took Kia approximately

hour to drive from Dryden to Edgewood.

Explanation for Incorrect Answer B :

file://E:\\d3.htm

2006-11-12

6/18

The Official SAT Online Course

hours to drive from Dryden

Choice (B) is not correct. It took Kia approximately

hour to drive from Dryden to Edgewood.

to Farley, but it took her approximately

Explanation for Incorrect Answer C :

hours to drive from Ashton

Choice (C) is not correct. It took Kia approximately

to Dryden. If it had taken

more hours for her to drive to Edgewood, she could

hours.

not have completed the trip in

Explanation for Incorrect Answer D :

hours to drive from Ashton

Choice (D) is not correct. It took Kia approximately

hour to drive from Dryden to Edgewood.

to Dryden, but it took her approximately

Explanation for Incorrect Answer E :

hours to drive from Ashton

Choice (E) is not correct. It took Kia approximately

hour to drive from Dryden to

to Edgewood, but it took her approximately


Edgewood.

and

If the average of

and

average of

is

and

and the average of

is

what is the

re

(A)

(B)

te

(C)

(D)
(E)

is

AND EXPLANATIONS

nR
eg

ANSWERS

Explanation for Correct Answer D :

Choice (D) is correct. If the average of

equation

then

so

satisfies the

If the average of

and

Therefore, the average of

then

is

and

and

is

is

and

Explanation for Incorrect Answer A :


Choice (A) is not correct. The average of

the average of

and

is

but the question asks for

and

is

but the question asks for

and

Explanation for Incorrect Answer B :

Choice (B) is not correct. The average of

the average of

and

Explanation for Incorrect Answer C :

Choice (C) is not correct. If the average of

but the question asks for the average of

Explanation for Incorrect Answer E :


Choice (E) is not correct. If the average of

but the question asks for the average of

file://E:\\d3.htm

and

is

then the value of

is

and

and

is

then the value of

is

and

Which of the following could be the graph in the

-plane of the function

2006-11-12

7/18

The Official SAT Online Course

(A)

(B)

(C)

re

(D)

is
te

nR
eg

(E)

ANSWERS

AND EXPLANATIONS

Explanation for Correct Answer D :

Choice (D) is correct. The graph in the

a line with a slope of

and a

-plane of the function

-intercept of

is

All five choices show the graph of

a straight line, but only the line in choice (D) has a positive slope and a positive
intercept.

Explanation for Incorrect Answer A :

Choice (A) is not correct. The graph in the

-plane of the function

is a line with a positive slope and a positive

-intercept. The line in choice (A) is

horizontal and, therefore, has a slope of

Explanation for Incorrect Answer B :

Choice (B) is not correct. The graph in the

-plane of the function

-intercept. The line in choice (B) is a


is a line with a positive slope and a positive
vertical line and, therefore, has an undefined slope and no
-intercept.

Explanation for Incorrect Answer C :

file://E:\\d3.htm

Choice (C) is not correct. The graph in the

-plane of the function

is a line with a positive slope and a positive


a negative slope.

-intercept. The line in choice (C) has

2006-11-12

8/18

The Official SAT Online Course

Explanation for Incorrect Answer E :

10

Choice (E) is not correct. The graph in the

-plane of the function

is a line with a positive slope and a positive


-intercept.
a negative

-intercept. The line in choice (E) has

could equal which of the following?

is a positive odd integer, then

If

(A)
(B)
(C)
(D)
(E)

ANSWERS

AND EXPLANATIONS

Explanation for Correct Answer C :


is a positive even
is a positive odd integer, then
Choice (C) is correct. If
Thus, the only integers between
is the next integer after
integer, and

where

that can be written as

and

Therefore, of the numbers

and

integer, are

where

is a positive odd

can be written as

given, only

is a positive odd

re

integer.

is
te

Explanation for Incorrect Answer A :


Choice (A) is not correct. The only number less than

where

Explanation for Incorrect Answer B :


Choice (B) is not correct. The only number less than

eg

where

nR

that can be written as

is a positive odd integer, is

Explanation for Incorrect Answer D :


Choice (D) is not correct. The only numbers less than

where

that can be written as

is a positive odd integer, is

is a positive odd integer, are

that can be written as

and

Explanation for Incorrect Answer E :

Choice (E) is not correct.

but this product does not satisfy all the

the value of

given conditions. In the product

would be

However, the question states that

is a positive odd integer.

11

is the point with the greatest

Point

What is the

-coordinate on the semicircle shown above.

-coordinate of point

(A)
(B)

file://E:\\d3.htm

2006-11-12

9/18

The Official SAT Online Course

(C)
(D)

(E)

ANSWERS

AND EXPLANATIONS

Explanation for Correct Answer E :


be the point at the right-hand end of the semicircle; let
Choice (E) is correct. Let

as shown below.

point

be the midpoint of

Since

is the midpoint of the diameter

-axis, that is, a vertical line segment. Thus,

must be parallel to the

and the semicircle has radius

It follows that point

re

has coordinates

is

-coordinate on the semicircle, the

is the point with the greatest

same. Since

to any point on the semicircle must be the

semicircle. Thus, the distance from

radius

it must be the center of the

on the

units to the left of

-coordinate

-axis and, therefore, has

te

is

Explanation for Incorrect Answer A :


-coordinate of
Choice (A) is not correct. The

is not found by summing the

and then taking the negative of this sum.

eg

coordinates of

Explanation for Incorrect Answer B :


Choice (B) is not correct. Since the center of the semicircle is not the origin, the
-coordinate on the
is not equal to the negative of the greatest
coordinate of
semicircle.

nR

Explanation for Incorrect Answer C :


Choice (C) is not correct. This choice may be the result of visually estimating the
answer instead of applying geometric principles.

Explanation for Incorrect Answer D :


Choice (D) is not correct. This choice may be the result of visually estimating the
answer instead of applying geometric principles.

12

bottles of shampoo of various brands, the cost and volume of each are
For
displayed in the scatterplot above, and the line of best fit for the data is shown. Of
the following, which is closest to the average (arithmetic mean) cost per ounce for
bottles?
the

file://E:\\d3.htm

2006-11-12

10/18

The Official SAT Online Course

(A)
(B)
(C)
(D)
(E)

ANSWERS

AND EXPLANATIONS

Explanation for Correct Answer C :


Choice (C) is correct. The vertical axis of the graph gives the cost of the bottle of
shampoo, and the horizontal axis gives the number of ounces of shampoo in the
bottle. Since the line of best fit for the data passes through the origin, the price of a
bottle of shampoo is about directly proportional to the number of ounces in the
bottle. The slope of the line of best fit, therefore, gives the approximate average
bottles. At the horizontal coordinate of
cost per ounce of shampoo for the

ounces, the vertical coordinate of the line of best fit is slightly more than

bottles is very slightly more than

Therefore, the average cost per ounce for the

per ounce.

ed

Explanation for Incorrect Answer A :

Choice (A) is not correct. Every bottle of shampoo shown costs more than

te
r

per ounce. Therefore, the average price per ounce for all

greater than

bottles must be

Explanation for Incorrect Answer B :

per

is

Choice (B) is not correct. Every bottle of shampoo shown costs at least

ounce, and many of the bottles cost more than this. Therefore, the average price

bottles must be greater than

per ounce for all

nR
eg

Explanation for Incorrect Answer D :


Choice (D) is not correct. Although some of the bottles of shampoo cost more than

per ounce and some cost less than this, the line of best fit has a slope that

is much closer to

per ounce than to

per ounce.

Explanation for Incorrect Answer E :


Choice (E) is not correct. Almost every bottle of shampoo shown costs less than

than

per ounce. Consequently, the line of best fit has a slope that is much less

bottles

per ounce. Therefore, the average price per ounce for all

must be less than

13

The graph of a quadratic function and the graph of a linear function in the
-plane can intersect in at most how many points?
(A)

One

(B)

Two

(C) Three

(D) Four

(E)

More than four

ANSWERS

AND EXPLANATIONS

Explanation for Correct Answer B :


Choice (B) is correct. The graph of a quadratic function and the graph of a linear
-plane can intersect in at most two points. The quadratic function
function in the

must be of the form

the linear function must be of the form

file://E:\\d3.htm

where

and

where

are constants, and

and

are

2006-11-12

11/18

The Official SAT Online Course

-coordinates of the intersection points can be found by setting the

constants. The

When

and then solving for

two functions equal,

simplified, this becomes a quadratic equation, so there can be at most two solutions
Therefore, the two graphs can intersect in at most two points.
for

Explanation for Incorrect Answer A :


Choice (A) is not correct. The graphs could have only one point in common, but

they could also have two points in common. For example,

have

and

and

two points in common,

Explanation for Incorrect Answer C :


Choice (C) is not correct. The graph of a linear function is a line, and the graph of a
quadratic function is a parabola, and a line and a parabola cannot intersect in three
points.

Explanation for Incorrect Answer D :


Choice (D) is not correct. The graph of a linear function is a line, and the graph of a
quadratic function is a parabola, and a line and a parabola cannot intersect in four
points.

ed

Explanation for Incorrect Answer E :


Choice (E) is not correct. The graph of a linear function is a line, and the graph of a
quadratic function is a parabola, and a line and a parabola can intersect in at most
two points.

is

If the length of

er

14

and the length of

which of the following could be

is
t

the length of

is

(A)

(B)

nR
eg

(C)

(D)

(E)

ANSWERS

AND EXPLANATIONS

Explanation for Correct Answer E :

could be

Choice (E) is correct. The length of

length

and

), the points

(and

Since

could form a triangle with sides of

and

Explanation for Incorrect Answer A :


Choice (A) is not correct. Since the shortest distance between two points is a

straight line, the length of

plus the distance between

and

must be less than the distance between

and

Thus, the length of

and

must be less than

cannot equal

Explanation for Incorrect Answer B :


Choice (B) is not correct. Since the shortest distance between two points is a

straight line, the length of

plus the distance between

and

must be less than the distance between

and

Thus, the length of

and

must be less than

cannot equal

Explanation for Incorrect Answer C :


Choice (C) is not correct. Since the shortest distance between two points is a

straight line, the length of

plus the distance between

and

file://E:\\d3.htm

must be less than the distance between

and

Thus, the length of

and

must be less than

cannot equal

2006-11-12

12/18

The Official SAT Online Course

Explanation for Incorrect Answer D :


Choice (D) is not correct. Since the shortest distance between two points is a

must be less than the distance between

straight line, the length of

in terms of

what is

are positive integers and

and

If

must be less than

cannot equal

and

15

Thus, the length of

and

plus the distance between

and

(A)
(B)
(C)
(D)
(E)

ANSWERS

AND EXPLANATIONS

Explanation for Correct Answer A :

are equivalent, the exponents

and

which gives

must be equal. Therefore,

re

and

are equivalent, so

and

The expressions

Since the expressions

gives

in the equation

for

Choice (A) is correct. Substituting

te

Explanation for Incorrect Answer B :

were equal to

is

Choice (B) is not correct. If

eg

imply that

would be equal to

then

would

This together with

However, this could only be true if

was equal to

and

were equal, which is not possible.

nR

Explanation for Incorrect Answer C :

Choice (C) is not correct. If

was equal to

then

were equal to

would be equal to

would imply that

This together with

and

However, this could only be true if

were

equal, which is not possible.

Explanation for Incorrect Answer D :

Choice (D) is not correct. If

were equal to

then

would be equal to

This together with

would imply that

and

was equal to

However, this could only be true if

were equal, which is not possible.

Explanation for Incorrect Answer E :

Choice (E) is not correct. If

were equal to

then

would be equal to

This together with

would imply that

and

was equal to

However, this could only be true if

were equal, which is not possible.

16

file://E:\\d3.htm

2006-11-12

13/18

The Official SAT Online Course

inches high and has a base of

The tin can in the figure above is a cylinder that is

inches, how many CANNOT fit entirely inside the can?

inches, and
(A)

One

(B)

Two

(C) Three

Five

ANSWERS

re
d

(D) Four

(E)

inches,

inches,

inches,

pencils with lengths

inches. Of

radius

AND EXPLANATIONS

Explanation for Correct Answer B :

te

of the base of the can

Choice (B) is correct. In the figure below, the diameter

and segment

form the two legs of a right

which is perpendicular to

triangle,

is

nR
eg

of this right triangle has length

By the Pythagorean Theorem, the hypotenuse,

inches, and it is the longest line segment within the can.

inches cannot fit entirely inside

Therefore, any pencil with length greater than

the can. The lengths of the pencils are

inches,

inches,

inches,

inches, so
inches. Exactly two of these pencils are longer than
inches, and
these two pencils are the only ones that cannot fit entirely inside the can.

Explanation for Incorrect Answer A :


Choice (A) is not correct. Any pencil longer than

the can. Two of the pencils are of lengths

inches cannot fit entirely inside

inches and

inches, so there is

more than one pencil that is too long to fit entirely inside the can.

Explanation for Incorrect Answer C :


Choice (C) is not correct. Three pencils ARE short enough to fit entirely within the

can. These are the pencils with lengths

the

file://E:\\d3.htm

inches,

inches, and

inches. Of

pencils, there are two that CANNOT fit entirely inside the can.

2006-11-12

14/18

The Official SAT Online Course

Explanation for Incorrect Answer D :


pencils, fewer than four are too long to fit
Choice (D) is not correct. Of the
entirely inside the can. Exactly two of the pencils are too long to fit.

Explanation for Incorrect Answer E :


pencils cannot fit entirely inside
Choice (E) is not correct. It is not true that all
the can. There are three pencils that are short enough to fit inside the can.

17

the result is the same as when

is multiplied by

When the number

is added to

What is the value of


(A)
(B)
(C)
(D)
(E)

ANSWERS

er
ed

Explanation for Correct Answer E :


Choice (E) is correct. When the number

When

AND EXPLANATIONS

is multiplied by

Subtracting

operations are the same, it follows that

sides of this equation gives

the result is

Since the results of these two

the result is

is added to

from both

is
t

Explanation for Incorrect Answer A :


Choice (A) is not correct. The statement in the question yields the equation

eg

This incorrect answer is the value of

which is

nR

value of

but the question asks for the

Explanation for Incorrect Answer B :


Choice (B) is not correct. The statement in the question yields the equation
This incorrect answer may arise from mistranslating the statement as

and then finding the value of

instead of the value of

Explanation for Incorrect Answer C :


Choice (C) is not correct. The statement in the question yields the equation

value of

It follows that the value of

is

but the question asks for the

which is

Explanation for Incorrect Answer D :


Choice (D) is not correct. The statement in the question yields the equation
This incorrect answer may arise from mistranslating the statement as

18

file://E:\\d3.htm

2006-11-12

15/18

The Official SAT Online Course

in the figure above. What could be the

A circle (not drawn) passes through point

total number of points of intersection of this circle and


I.

II.

III.

(A)

I only

(B)

II only

(C) I and II only

(D) II and III only

(E)

I, II, and III

ANSWERS

AND EXPLANATIONS

re
d

Explanation for Correct Answer E :


Choice (E) is correct. In the figure below,

te

is

the circle passes through point

the circle and

is

and the total number of points of intersection of

eg

In the figure below,

nR

the circle passes through point

the circle and

and the total number of points of intersection of

is

In the figure below,

the circle passes through point

the circle and

and the total number of points of intersection of

is

Therefore, I, II, and III each give a number that could be the total number of points
and a circle that passes through
of intersection of

file://E:\\d3.htm

2006-11-12

16/18

The Official SAT Online Course

Explanation for Incorrect Answer A :


and
Choice (A) is not correct. A circle can be drawn that passes through point
point. However, another circle can be drawn that
in only
intersects

points. Also, a third circle can

in

and intersects

passes through point

points.

in

and intersects

be drawn that passes through point

Explanation for Incorrect Answer B :


Choice (B) is not correct. A circle can be drawn that passes through point

point. Also, a third circle can be

in only

and intersects

through point

in

and intersects

drawn that passes through point

points.

Explanation for Incorrect Answer C :


Choice (C) is not correct. A circle can be drawn that passes through point

point

points. However, a third circle can be drawn

in

and intersects

points.

in

and intersects

that passes through point

Explanation for Incorrect Answer D :


Choice (D) is not correct. A circle can be drawn that passes through point

point

and

points. Another circle can be drawn that passes through

in

intersects

and

point. Another circle can be drawn that passes through

in

intersects

and

points. However, another circle can be drawn that passes

in

intersects

points. However, a third circle can be drawn

in

and intersects

point.

in

and intersects

re

that passes through point

19

is
te

eg

nR

The function

values of

graphed above is defined for

For which of the following

is

(A)

(B)
(C)
(D)

(E)

ANSWERS

AND EXPLANATIONS

Explanation for Correct Answer D :


Choice (D) is correct. A number is less than its absolute value if the number is
negative, and equal to its absolute value if the number is nonnegative. Thus,

if and only if

is negative. We can see from the graph that

which is negative. Therefore,

given for

For all of the other choices

is positive.

Explanation for Incorrect Answer A :

file://E:\\d3.htm

2006-11-12

17/18

The Official SAT Online Course

must be negative. But we

then

Choice (A) is not correct. If

which is positive. Therefore,

can see from the graph that

Explanation for Incorrect Answer B :

must be negative. But we

Choice (B) is not correct. If

then

can see from the graph that

is positive. Therefore,

Explanation for Incorrect Answer C :

must be negative. But we

then

Choice (C) is not correct. If

which is not a negative number. Therefore,

can see from the graph that

Explanation for Incorrect Answer E :

must be negative. But we

then

Choice (E) is not correct. If

which is positive. Therefore,

can see from the graph that

20

what is the smallest possible

are numbers such that

and

If

re
d

value of
(A)

(B)
(C)

is
te

(D)
(E)

AND EXPLANATIONS

eg

ANSWERS

Explanation for Correct Answer D :

The smallest possible value of

nR

then

is

The smallest possible value for this

and

then

If

the expression will have this value when

expression is

If

or

then either

Choice (D) is correct. If

In either case, the smallest

which will occur when

possible value of

is

Explanation for Incorrect Answer A :

possible; if

then either

This is not

and

then both

Choice (A) is not correct. If

or

Explanation for Incorrect Answer B :

then either

Choice (B) is not correct. If

or

or

Therefore, either

so

In either case,

cannot

be the smallest possible value of

Explanation for Incorrect Answer C :

then either

Choice (C) is not correct. If

or

or

Therefore, either

In either case,

so

cannot be the smallest possible value of

file://E:\\d3.htm

2006-11-12

18/18

The Official SAT Online Course


Explanation for Incorrect Answer E :

yet

then

and

However, if

then

and

Choice (E) is not correct. If

Therefore, the smallest possible value of

cannot be

Back to Score Report

Privacy Policy

Copyright 2006 The College Board. All rights reserved.

Terms of Use

Contact Us

te
re

is

eg

nR

file://E:\\d3.htm

2006-11-12

1/23

The Official SAT Online Course

Help | Profile | My Organizer | My Bookmarks | Logout

Answers and Explanations

Back to Score Report

Test Sections

Section 1

View Answers and Explanations

Section 2

Online - Practice Test #4

Section 4

Section 5

Section 6

Section 7

The architect advised tearing down the old structure, since he did not consider it
sufficiently ------- to ------- the heavy winds of the tropical storm the peninsula was
expecting.
(A) flimsy . . forestall

Section 8

(B)

Section 9

hardy . . forecast

(C) robust . . withstand

Section 10

(D) noteworthy . . justify

ramshackle . . repel

AND EXPLANATIONS

re

ANSWERS

(E)

Explanation for Correct Answer C :


Choice (C) is correct. Robust means sturdy or strongly constructed. To
withstand means to resist the effect of something. If one were to insert these
terms into the text, the sentence would read The architect advised tearing down
the old structure, since he did not consider it sufficiently robust to withstand the
heavy winds of the tropical storm the peninsula was expecting. It is reasonable to
suggest that the architect advised that the old structure be torn down because he
did not think it was robust, or sturdy, enough to withstand, or resist the
effect of, the heavy winds. A structure that is not strongly constructed would likely
be damaged or destroyed during a tropical storm.

is
te

eg

nR

Explanation for Incorrect Answer A :


Choice (A) is incorrect. Flimsy means lacking in physical strength or substance.
To forestall means to prevent by taking precautionary measures. If one were to
insert these terms into the text, the sentence would read The architect advised
tearing down the old structure, since he did not consider it sufficiently flimsy to
forestall the heavy winds of the tropical storm the peninsula was expecting. It is
illogical to suggest that a structure, especially a flimsy one, could forestall,
or prevent, the heavy winds of a tropical storm.

Explanation for Incorrect Answer B :


Choice (B) is incorrect. Hardy means capable of withstanding adverse
conditions. To forecast means to calculate or predict something in advance. If
one were to insert these terms into the text, the sentence would read The
architect advised tearing down the old structure, since he did not consider it
sufficiently hardy to forecast the heavy winds of the tropical storm the peninsula
was expecting. It is illogical to suggest that a structure could forecast, or
predict, the heavy winds of a tropical storm.

Explanation for Incorrect Answer D :


Choice (D) is incorrect. Noteworthy means deserving notice or attention. To
justify means to prove to be right or reasonable. If one were to insert these
terms into the text, the sentence would read The architect advised tearing down
the old structure, since he did not consider it sufficiently noteworthy to justify the
heavy winds of the tropical storm the peninsula was expecting. An old structure
might be noteworthy, or deserving of attention, but it is illogical to suggest
that a structure could justify, or prove reasonable, the heavy winds of a
tropical storm.

Explanation for Incorrect Answer E :


Choice (E) is incorrect. Ramshackle means carelessly or loosely constructed.
To repel means to drive back or to resist something. If one were to insert these
terms into the text, the sentence would read The architect advised tearing down
the old structure, since he did not consider it sufficiently ramshackle to repel the

file://E:\\d4.htm

2006-11-12

2/23

The Official SAT Online Course

heavy winds of the tropical storm the peninsula was expecting. It is illogical to
suggest that a structure could be sufficiently ramshackle to repel, or drive
back, the heavy winds of the tropical storm. A carelessly constructed structure
would be more likely to be damaged by the winds than to repel the winds.

When x-rays were discovered around the turn of the twentieth century, doctors
quickly began to ------- their newfound ability to diagnose maladies by peering
beneath the surface of the human body.
(A) bequeath
(B)

deny

(C) exploit

(D) finesse

(E)

divulge

ANSWERS

AND EXPLANATIONS

Explanation for Correct Answer C :


Choice (C) is correct. To exploit means to utilize or to make productive use of
something. If one were to insert this term into the text, the sentence would read
When x-rays were discovered around the turn of the twentieth century, doctors
quickly began to exploit their newfound ability to diagnose maladies by peering
beneath the surface of the human body. Before x-rays were discovered, doctors
did not have the ability to peer beneath the surface of the human body in order to
diagnose maladies. Once x-rays were discovered, doctors were quick to
exploit, or make productive use of, this new ability.

re

te

eg
is

Explanation for Incorrect Answer A :


Choice (A) is incorrect. To bequeath means to hand down. If one were to insert
this term into the text, the sentence would read When x-rays were discovered
around the turn of the twentieth century, doctors quickly began to bequeath their
newfound ability to diagnose maladies by peering beneath the surface of the human
body. In order for the term bequeath to make sense in this context the
sentence would have to indicate to whom the ability was bequeathed, or handed
down. Additionally, although doctors still use x-rays to diagnose maladies, it is
awkward to suggest that this ability was handed down to them.

nR

Explanation for Incorrect Answer B :


Choice (B) is incorrect. To deny means to claim to have no association with or
responsibility for something. If one were to insert this term into the text, the
sentence would read When x-rays were discovered around the turn of the
twentieth century, doctors quickly began to deny their newfound ability to diagnose
maladies by peering beneath the surface of the human body. There is no reason
to believe that doctors were quick to deny their new ability. There is nothing
inherently negative about x-rays that would cause doctors to deny, or disclaim
association with or responsibility for, the abilities that x-rays allowed.

Explanation for Incorrect Answer D :


Choice (D) is incorrect. To finesse means to handle something with skillful
maneuvering. If one were to insert this term into the text, the sentence would read
When x-rays were discovered around the turn of the twentieth century, doctors
quickly began to finesse their newfound ability to diagnose maladies by peering
beneath the surface of the human body. Although doctors might be eager to use
a newfound technology in ever more sophisticated ways, finesse contains a
suggestion of subtlety, and even of evasion, that does not really apply to a scientist
doing medical research.

Explanation for Incorrect Answer E :


Choice (E) is incorrect. To divulge means to make something known, usually
something private or secret. If one were to insert this term into the text, the
sentence would read When x-rays were discovered around the turn of the
twentieth century, doctors quickly began to divulge their newfound ability to
diagnose maladies by peering beneath the surface of the human body. Although
doctors may have quickly made it known that they could use x-rays to diagnose
maladies, there is no reason to believe that this ability was a secret or something
private. Therefore, it does not make logical sense to suggest that the doctors were
quick to divulge their newfound ability.

file://E:\\d4.htm

2006-11-12

3/23

The Official SAT Online Course

Though he was fascinated by the ------- behavior of others, Darek was, by contrast,
the model of ------- in his own comportment.
(A) hedonistic . . recklessness
(B)

unorthodox . . conformity

(C) restless . . agitation

(D) egotistical . . extremity

(E)

unwieldy . . rigidity

ANSWERS

AND EXPLANATIONS

Explanation for Correct Answer B :


Choice (B) is correct. Unorthodox means not following tradition or convention.
Conformity is a tendency to behave in ways that are socially acceptable. If one
were to insert these terms into the text, the sentence would read Though he was
fascinated by the unorthodox behavior of others, Darek was, by contrast, the model
of conformity in his own comportment. The word Though and the phrase
by contrast indicate that the missing terms will describe opposing types of
behavior. The unorthodox, or unconventional, behavior of others contrasts with
Dareks own conformity.

re

Explanation for Incorrect Answer A :


Choice (A) is incorrect. Hedonistic means pursuing pleasure and happiness.
Recklessness is irresponsibility and a lack of caution. If one were to insert
these terms into the text, the sentence would read Though he was fascinated by
the hedonistic behavior of others, Darek was, by contrast, the model of
recklessness in his own comportment. The word Though and the phrase by
contrast indicate that the missing terms will describe opposing types of behavior.
The terms hedonistic and reckless are not necessarily in opposition to each
other. Hedonism, or the pursuit of pleasure, can sometimes involve irresponsible
behavior and a lack of caution.

te

is

nR
eg

Explanation for Incorrect Answer C :


Choice (C) is incorrect. Restless means continuously moving or unsettled.
Agitation is a state of emotional disturbance. If one were to insert these terms
into the text, the sentence would read Though he was fascinated by the restless
behavior of others, Darek was, by contrast, the model of agitation in his own
comportment. The word Though and the phrase by contrast indicate
that the missing terms will describe opposing types of behavior. The terms
restless and agitation are not necessarily in opposition to each other
because both terms can refer to a state of being unsettled or disturbed.

Explanation for Incorrect Answer D :


Choice (D) is incorrect. Egotistical means having an exaggerated sense of selfimportance. Extremity is a tendency to commit drastic or desperate acts. If one
were to insert these terms into the text, the sentence would read Though he was
fascinated by the egotistical behavior of others, Darek was, by contrast, the model
of extremity in his own comportment. The word Though and the phrase by
contrast indicate that the missing terms will describe opposing types of behavior.
The terms egotistical and extremity are not necessarily in opposition to
each other. Someone could have both an exaggerated sense of self-importance and
a tendency to commit drastic acts.

Explanation for Incorrect Answer E :


Choice (E) is incorrect. Unwieldy means not easily managed or handled
because of weight or complexity. Rigidity is the quality of being unyielding and
inflexible. If one were to insert these terms into the text, the sentence would read
Though he was fascinated by the unwieldy behavior of others, Darek was, by
contrast, the model of rigidity in his own comportment. Although the second term
makes sense in this context, the first term does not. While the term unwieldy
can be used to describe something that is difficult to handle because of its physical
properties or that is difficult to manage because of its complexity, it does not make
sense to describe a persons behavior as unwieldy.

file://E:\\d4.htm

Teachers who consider cartoons and comic books harmful to students literacy skills
often use class time to ------- these media.

2006-11-12

4/23

The Official SAT Online Course

(A)

deride

(B)

rationalize

(C) vindicate

(D) foster

(E)

annotate

ANSWERS

AND EXPLANATIONS

Explanation for Correct Answer A :


Choice (A) is correct. To deride means to ridicule or treat with contemptuous
mirth. If one were to insert this term into the text, the sentence would read
Teachers who consider cartoons and comic books harmful to students literacy
skills often use class time to deride these media. Teachers who think cartoons
and comic books are harmful to their students literacy skills would probably want
to discourage their students from reading these media. Therefore, it is reasonable
to suggest that these teachers often use class time to deride, or ridicule,
cartoons and comic books.

ed

Explanation for Incorrect Answer B :


Choice (B) is incorrect. To rationalize means to cause something to seem
reasonable. If one were to insert this term into the text, the sentence would read
Teachers who consider cartoons and comic books harmful to students literacy
skills often use class time to rationalize these media. It is unlikely that teachers
would rationalize media that they consider to be harmful to their students
literacy skills.

er

Explanation for Incorrect Answer C :


Choice (C) is incorrect. To vindicate means to defend or to provide justification.
If one were to insert this term into the text, the sentence would read Teachers
who consider cartoons and comic books harmful to students literacy skills often
use class time to vindicate these media. It is unlikely that teachers would
vindicate, or defend, cartoons and comic books if they consider these media to
be harmful to their students literacy skills.

is
t

eg

Explanation for Incorrect Answer D :


Choice (D) is incorrect. To foster means to encourage or to promote the growth
of something. If one were to insert this term into the text, the sentence would read
Teachers who consider cartoons and comic books harmful to students literacy
skills often use class time to foster these media. It is unlikely that teachers would
foster, or promote the growth of, cartoons and comic books if they consider
these media to be harmful to their students literacy skills.

nR

Explanation for Incorrect Answer E :


Choice (E) is incorrect. To annotate means to provide explanatory notes for a
written work. If one were to insert this term into the text, the sentence would read
Teachers who consider cartoons and comic books harmful to students literacy
skills often use class time to annotate these media. Teachers who consider
cartoons and comic books to be harmful to students literacy skills would probably
not annotate these media. It is unlikely that these teachers would use class
time to provide explanatory notes for written works they consider harmful.

Because he had decided not to ------- himself through the sales of his new product,
the inventor anonymously donated all profits to charity.
(A)

compromise

(B)

invigorate

(C) impoverish

(D) aggrandize

(E)

debilitate

ANSWERS

AND EXPLANATIONS

Explanation for Correct Answer D :

file://E:\\d4.htm

2006-11-12

5/23

The Official SAT Online Course

Choice (D) is correct. To aggrandize means to make wealthy. If one were to


insert this term into the text, the sentence would read Because he had decided
not to aggrandize himself through the sales of his new product, the inventor
anonymously donated all profits to charity. Donating profits to charity is a logical
result of the inventors decision not to aggrandize himself, or make himself
wealthy.

Explanation for Incorrect Answer A :


Choice (A) is incorrect. To compromise means to expose oneself to suspicion. If
one were to insert this term into the text, the sentence would read Because he
had decided not to compromise himself through the sales of his new product, the
inventor anonymously donated all profits to charity. Nothing in the sentence
indicates that the inventor would be compromised, or exposed to suspicion, if he
kept the profits from his new product. There is not necessarily anything suspicious
about making a profit from the sale of a product.

Explanation for Incorrect Answer B :


Choice (B) is incorrect. To invigorate means to energize. If one were to insert
this term into the text, the sentence would read Because he had decided not to
invigorate himself through the sales of his new product, the inventor anonymously
donated all profits to charity. Although one might find it invigorating to
successfully sell a new product, it is illogical to suggest that the inventors decision
to anonymously donate his profits was a result of his decision not to invigorate,
or energize, himself.

te
re

Explanation for Incorrect Answer C :


Choice (C) is incorrect. To impoverish means to make poor. If one were to
insert this term into the text, the sentence would read Because he had decided
not to impoverish himself through the sales of his new product, the inventor
anonymously donated all profits to charity. It is illogical to suggest that the
inventor would donate his profits in order to avoid making himself poor.

Explanation for Incorrect Answer E :


Choice (E) is incorrect. To debilitate means to weaken. If one were to insert
this term into the text, the sentence would read Because he had decided not to
debilitate himself through the sales of his new product, the inventor anonymously
donated all profits to charity. There is no reason to believe that keeping profits
from the sales of the product would weaken the inventor, so it does not make sense
to say that the inventor donated the profits in order to avoid debilitating himself.

eg
is

nR

Once he had ------- sufficient ------- information, Randall felt confident in publishing
his daring article incriminating the local politician.

(A)

written . . substantial

(B)

believed . . sensational

(C) obtained . . corroborating

(D) reported . . hackneyed

(E)

discovered . . contradicting

ANSWERS

AND EXPLANATIONS

Explanation for Correct Answer C :


Choice (C) is correct. To obtain means to gather or to gain. Corroborating
means supporting or confirming with evidence. If one were to insert these terms
into the text, the sentence would read Once he had obtained sufficient
corroborating information, Randall felt confident in publishing his daring article
incriminating the local politician. If Randall wrote an article that would
incriminate, or expose the crimes or fault of, the local politician, he would need
evidence to support his claims. Obtaining, or gathering, corroborating
information would most likely make Randall feel confident in publishing his article.

Explanation for Incorrect Answer A :


Choice (A) is incorrect. To write means to compose a written work.
Substantial means important and essential. If one were to insert these terms
into the text, the sentence would read Once he had written sufficient substantial

file://E:\\d4.htm

2006-11-12

6/23

The Official SAT Online Course

information, Randall felt confident in publishing his daring article incriminating the
local politician. The first term makes sense in this context, but the second term
does not. Although Randall would most likely include important information in his
article, information that is substantial would not necessarily incriminate, or
expose the crimes or fault of, the local politician.

Explanation for Incorrect Answer B :


Choice (B) is incorrect. To believe means to accept as true or real.
Sensational means arousing strong interest or curiosity. If one were to insert
these terms into the text, the sentence would read Once he had believed
sufficient sensational information, Randall felt confident in publishing his daring
article incriminating the local politician. While it is likely that Randall would
believe that the information in his article was true, it does not make sense to say
that Randall would have to believe a sufficient amount of sensational
information before he could feel confident about publishing his article.

Explanation for Incorrect Answer D :


Choice (D) is incorrect. To report means to give an account. Hackneyed
means overly familiar and not original. If one were to insert these terms into the
text, the sentence would read Once he had reported sufficient hackneyed
information, Randall felt confident in publishing his daring article incriminating the
local politician. While the first term makes sense in this context, the second does
not. If Randall reported hackneyed, or overly familiar, information, it is unlikely
that his article would be daring.

re

Explanation for Incorrect Answer E :


Choice (E) is incorrect. To discover means to find or obtain knowledge of
something. Contradicting means opposing or implying the opposite of
something. If one were to insert these terms into the text, the sentence would read
Once he had discovered sufficient contradicting information, Randall felt confident
in publishing his daring article incriminating the local politician. It is not clear
what the information Randall had discovered was contradicting. If he had
found pieces of information that contradict each other, it is unlikely that Randall
would feel confident in publishing his article.

te

is

Since her personal pleas had failed to make her noisy neighbors change their ways,
the homeowner felt that her only ------- was to notify the police.

eg

(A)

backlash

(B)

recourse

nR

(C) bromide

(D) reckoning

(E)

forbearance

ANSWERS

AND EXPLANATIONS

Explanation for Correct Answer B :


Choice (B) is correct. Recourse is a source of help or strength. If one were to
insert this term into the text, the sentence would read Since her personal pleas
had failed to make her noisy neighbors change their ways, the homeowner felt that
her only recourse was to notify the police. Because the homeowners personal
pleas did not quiet her noisy neighbors, she felt that the police were her only
recourse, or source of help.

Explanation for Incorrect Answer A :


Choice (A) is incorrect. Backlash is a strong adverse reaction. If one were to
insert this term into the text, the sentence would read Since her personal pleas
had failed to make her noisy neighbors change their ways, the homeowner felt that
her only backlash was to notify the police. While the noisy neighbors might see
notifying the police as a backlash, or an adverse reaction to their behavior, the
homeowner would almost certainly consider it the normal course of action for
someone whose personal pleas had failed to work.

Explanation for Incorrect Answer C :


Choice (C) is incorrect. Bromide is a commonplace or tiresome statement or
idea. If one were to insert this term into the text, the sentence would read Since
her personal pleas had failed to make her noisy neighbors change their ways, the
homeowner felt that her only bromide was to notify the police. It does not make

file://E:\\d4.htm

2006-11-12

7/23

The Official SAT Online Course

sense to say that notifying the police was the homeowners only bromide.
Requesting the help of the police is not a tiresome or hackneyed statement or idea.

Explanation for Incorrect Answer D :


Choice (D) is incorrect. Reckoning is a settling of accounts. If one were to
insert this term into the text, the sentence would read Since her personal pleas
had failed to make her noisy neighbors change their ways, the homeowner felt that
her only reckoning was to notify the police. While the homeowner might see
notifying the police as a settling of the problem with her neighbors, it does not
really make sense in this context to refer to the problem as an account that could
be settled.

Explanation for Incorrect Answer E :


Choice (E) is incorrect. Forbearance is an act of patience. If one were to insert
this term into the text, the sentence would read Since her personal pleas had
failed to make her noisy neighbors change their ways, the homeowner felt that her
only forbearance was to notify the police. It is somewhat illogical to say that
notifying the police was an act of patience. On the contrary, it is most likely that the
homeowner decided to notify the police because she had lost her patience with her
noisy neighbors.

Despite pressure from reporters to discuss the scandal in which Senator Scottsdale
was currently ------- , the press secretary would not ------- the details of the
senators upcoming public address.
(A)

imbued . . rescind

(B)

connected . . consort

ed

(D) embroiled . . divulge

compliant . . quash

AND EXPLANATIONS

eg
i

ANSWERS

st

(E)

er

(C) entangled . . repeal

Explanation for Correct Answer D :


Choice (D) is correct. Embroiled means involved in a difficulty or conflict. To
divulge means to reveal or make public. If one were to insert these terms into
the text, the sentence would read Despite pressure from reporters to discuss the
scandal in which Senator Scottsdale was currently embroiled, the press secretary
would not divulge the details of the senators upcoming public address. The use
of despite implies that the reporters were applying pressure in order to get the
press secretary to give them details about the senators upcoming speech; in
spite of this pressure, however, the press secretary refuses to make public the
contents of this speech so that the reporters, interested in news about the scandal,
do not know what to expect.

nR

Explanation for Incorrect Answer A :


Choice (A) is incorrect. Imbued means inspired, influenced thoroughly, or
pervaded. To rescind means to remove or take back. If one were to insert
these terms into the text, the sentence would read Despite pressure from
reporters to discuss the scandal in which Senator Scottsdale was currently imbued,
the press secretary would not rescind the details of the senators upcoming public
address. Neither term makes sense in this context. Even if the senator had been
influenced by the scandal or it had pervaded his life, it would not make sense to say
he was imbued in it. It is also illogical to say that the press secretary refused
to take back information that has not yet been given out.

Explanation for Incorrect Answer B :


Choice (B) is incorrect. Connected means linked. To consort means to
unite or associate. If one were to insert these terms into the text, the sentence
would read Despite pressure from reporters to discuss the scandal in which
Senator Scottsdale was currently connected, the press secretary would not consort
the details of the senators upcoming public address. While the senator
certainly could be linked to a scandal, it does not make sense to say that the
secretary would not "consort," or associate with, the details.

Explanation for Incorrect Answer C :


Choice (C) is incorrect. Entangled means involved, usually in a perplexing or
troublesome situation. To repeal means to revoke or rescind by an

file://E:\\d4.htm

2006-11-12

8/23

The Official SAT Online Course

authoritative act. If one were to insert these terms into the text, the sentence would
read Despite pressure from reporters to discuss the scandal in which Senator
Scottsdale was currently entangled, the press secretary would not repeal the details
of the senators upcoming public address. The first term makes sense here: the
senator could certain be involved in a troublesome way in a scandal. But it makes
little sense to say that the secretary refused to repeal, or revoke, information
that has not yet been given out.

Explanation for Incorrect Answer E :


Choice (E) is incorrect. Compliant means submissive or willing to comply. To
quash means to suppress or extinguish. If one were to insert these terms into the
text, the sentence would read Despite pressure from reporters to discuss the
scandal in which Senator Scottsdale was currently compliant, the press secretary
would not quash the details of the senators upcoming public address. It is
unlikely that a senator would be submissive to or willing to comply with a scandal.
Additionally, it makes little sense to say that the secretary refused to quash, or
suppress, information that has not yet been given out.

Modern historians use the terms feudalism and manorialism to refer to the
ways that medieval European societies were organized.

Passage 1

ed

Feudalism is one of those words that


have taken on so many extended and
figurative meanings that the original
Line meaning has been obscured. Today any
5
oppressive government, greedy
landholder, or brutal exploiter of labor is
called feudalalways with disapproval.
This is unfair to feudalism. The word is
also often confused with the manorial
10 system, which tied peasants to the land
they worked. Feudalism is a total
organization of society. It is a scheme of
political organization, based in law and
overlapping with social and economic
15 organization.

is
te
r

eg

nR

Passage 2

Older historians used the term feudalism


for the whole medieval social order, which
was a peasant society dominated by a
military, land-owning aristocracy. Modern
20 usage generally restricts the word to the
network of relations between tenants and
lords within the aristocracy. The system
governing the peasants relation to the
lord, which was the economic foundation
25 of medieval society, is usually designated
the manorial system. The relationships
embodied in the feudal and manorial
systems were simple enough in theory:
In the manorial system, a peasant labored
30 for a lord in return for land of his own; in
the feudal system, a lord held lands from
the king or the overlord in return for

file://E:\\d4.htm

2006-11-12

9/23

The Official SAT Online Course

supplying soldiers on demand.


9

Which of the following is closest to what the author of Passage 1 means by the phrase
This is unfair to feudalism (line 8)?
(A)

Feudalism should not be used figuratively.

(B)

Feudalism should not be used to refer to a system of government.

(C) Those who lived in feudal societies would disapprove of the way the term
is currently used.

(D) Feudalism should not necessarily have only negative connotations.

(E)

Feudalism has been misused to the extent that the term no longer has
a meaning.

ANSWERS

AND EXPLANATIONS

Explanation for Correct Answer D :


Choice (D) is correct. In this section of the passage, this refers back to the
previous sentence: Today any oppressive government, greedy landholder, or
brutal exploiter of labor is called feudalalways with disapproval. The author of
Passage 1 has also just reminded us that the original meaning [of feudalism] has
been obscured through overuse and misuse. It can be inferred, then, that the
unfairness the author sees is a result of this oversimplification and that the term, in
the authors view, should not have only negative connotations, or should not
be associated only with greed, oppression, and brutality.

re

is
te

Explanation for Incorrect Answer A :


Choice (A) is incorrect. The author of Passage 1 does not comment on whether or
not the term feudalism should be used figuratively, but, rather, is concerned
that the term has unfortunately taken on nothing but negative connotations. This
author is not stating or suggesting that feudalism should be used only in the
literal sense.

eg

Explanation for Incorrect Answer B :


Choice (B) is incorrect. The author of Passage 1 does not say that feudalism
should not be used to refer to a system of government; rather, the author says that
the term should not be associated only with oppressive forms of government.

nR

Explanation for Incorrect Answer C :


Choice (C) is incorrect. The author of Passage 1 does not comment on how those
who lived under feudalism would have felt about the term itself, nor that they even
used the term.

Explanation for Incorrect Answer E :


Choice (E) is incorrect. The author of Passage 1 does think that the term
feudalism has been misused, but does not say that the term has become
meaningless; on the contrary, the author says that the original meaning of the
word has become obscured, and that in current usage, the term has only
negative connotations.

10

The word extended in line 2 most nearly means


(A)

prolonged

(B)

expanded

(C) removed

(D) allocated

(E)

intensive

ANSWERS

AND EXPLANATIONS

Explanation for Correct Answer B :


Choice (B) is correct. In this sentence, the author is saying that the term
feudalism has acquired so many extended and figurative meanings that its
original meaning has been mostly forgotten. Before finally defining feudalism at the
end of Passage 1, the author goes through various negative meanings that the

file://E:\\d4.htm

2006-11-12

10/23

The Official SAT Online Course

word has taken on to its detriment. From its start as a total organization of
society, feudalism has, in the modern mind, come to symbolize oppression,
servitude, and other such woes; these are part of its extended, or expanded,
meaningsin the authors view, erroneous ones.

Explanation for Incorrect Answer A :


Choice (A) is incorrect. In this sentence, the author is decrying the extended and
figurative meanings that the term feudalism has acquired. While the new
definitions may indeed be prolonged in the sense of being long-lasting, it is the
number of new connotations that the author is concentrating on herethe terms
expansion.

Explanation for Incorrect Answer C :


Choice (C) is incorrect. In this sentence, the author is decrying the extended and
figurative meanings that the term feudalism has acquired. While
removed can mean distanced, and while the author can be said to be
speaking of definitions that are distanced from the original, this would be a
cumbersome and awkward way of getting at the intended meaningthat the term
has expanded in definition.

Explanation for Incorrect Answer D :


Choice (D) is incorrect. In this sentence, the author is decrying the extended and
figurative meanings that the term feudalism has acquired. In this context, it
would make little sense to say that the term has taken on allocated, or
designated, new meanings.

ed

Explanation for Incorrect Answer E :


Choice (E) is incorrect. In this sentence, the author is decrying the extended and
figurative meanings that the term feudalism has acquired. In this context, it
would make little sense to say that the term has taken on intensive, or
concentrated, new meanings.

According to the author of Passage 2, the term manorial refers to

eg
i

11

st
er

(A)

the whole medieval social order

(B)

the relationships among the members of the medieval aristocracy

(C) the economic relationship between medieval peasants and lords

nR

(D) the exchange of military protection for land ownership

(E)

the system of laws governing overlords

ANSWERS

AND EXPLANATIONS

Explanation for Correct Answer C :


Choice (C) is correct. In the third sentence of Passage 2while trying to distinguish
between manorialism and feudalismthe author says that the manorial system is
usually defined as the system governing the peasants relation to the lord,
which was the economic foundation of medieval society.

Explanation for Incorrect Answer A :


Choice (A) is incorrect. The author of Passage 2 does mention the whole medieval
social order, but connects it with the term feudalism rather than with the
term manorialism: Older historians used the term feudalism for the
whole medieval social order.

Explanation for Incorrect Answer B :


Choice (B) is incorrect. The author of Passage 2 does mention relationships
between tenants and lords within the aristocracy, but connects these
relationships with the term feudalism rather than with the term
manorialism: Modern usage generally restricts the word to the network of
relations between tenants and lords within the aristocracy.

Explanation for Incorrect Answer D :


Choice (D) is incorrect. The author of Passage 2 does mention the exchange of
military protection for land ownership, but connects it with the term feudalism

file://E:\\d4.htm

2006-11-12

11/23

The Official SAT Online Course

rather than with the term manorialism: in the feudal system, a lord held
lands from the king or the overlord in return for supplying soldiers on demand.

Explanation for Incorrect Answer E :


Choice (E) is incorrect. The only mention in Passage 2 of overlords has to do with
military protection in feudalism, not manorialism: in the feudal system, a lord
held lands from the king or the overlord in return for supplying soldiers on
demand.

12

Compared with the tone of Passage 1, the tone of Passage 2 is more


(A)

objective

(B)

disdainful

(C) lively

(D) unsympathetic

(E)

argumentative

ANSWERS

AND EXPLANATIONS

ed

Explanation for Correct Answer A :


Choice (A) is correct. Whereas the tone of Passage 1 is opinionated, even
argumentativeToday any oppressive government, greedy landholder, or brutal
exploiter of labor is called feudalalways with disapproval. This is unfair to
feudalismthe tone of Passage 2 is, by comparison, unemotional, or
objective. Since Passage 2 is largely concerned with definitions, this objectivity
is appropriate: Older historians used, Modern usage generally restricts,
and other such phrases inject a decidedly unemotional, rational tone.

er

st

eg
i

Explanation for Incorrect Answer B :


Choice (B) is incorrect. While it might be valid to call Passage 1 disdainful
that is, disdainful of those who have overused and misused the term
feudalismthe tone of Passage 2 is objective, dispassionate, and explanatory.

Explanation for Incorrect Answer C :


Choice (C) is incorrect. It would be hard to find anything particularly lively or
colorful in Passage 2, either in comparison with Passage 1 or not. The tone of
Passage 2 is objective, dispassionate, and explanatory.

nR

Explanation for Incorrect Answer D :


Choice (D) is incorrect. While it might be valid to call Passage 1
unsympatheticthat is, unsympathetic with those who have overused and
misused the term feudalismthe tone of Passage 2 is objective, dispassionate,
and explanatory.

Explanation for Incorrect Answer E :


Choice (E) is incorrect. While Passage 1 could accurately be called
argumentativethat is, it argues that the term feudalism has unfairly
been misused and overusedthe tone of Passage 2 is, by comparison, objective,
dispassionate, and explanatory.

In the following excerpt from a novel, Samuel Tyne, a Canadian of Ghanaian descent,
returns to work at the Canadian Ministry of Economics after attending his uncles
funeral.

His overt melancholy aggravated


his boss, for it made Samuel hard to
approach. Just a glance into Samuels
Line cubicle gave his co-workers much to gloat
5
about. It seemed a wonder he was such
an exacting employee, with the swift but
pitiful stride that brought him,
file://E:\\d4.htm

2006-11-12

12/23

The Official SAT Online Course

25

30

35

er
ed

20

is
t

15

eg

10

disillusioned, to the threshold of every


meeting. Yet he was so indispensable in
that ministry that his co-workers regretted
every slur they flung at him, lest the
slights drive him to suicide. For not only
would the department collapse without his
doting, steady logic to balance it, but it
seemed at times that the entire Canadian
economy depended on the reluctant, softwristed scribbling he did in his green
ledger.
There Samuel sat each day,
painfully tallying his data, his pencil
poised like a scalpel in his hand, frowning
at the gruesome but inevitable task ahead
of him. Dwarfed by a monstrous blue
suit, Samuel would finger the mournful
pre-war bowler that never left his head.
And it was such an earnest sight, such an
intimate window into a man whose nature
seemed to be all windowspeople
wondered if he actually had a public self
that he might have been the only man in
the world to claim vulnerability as his
greatest asset.
The day after the funeral, Samuel
returned to work to find a note from his
bosses on his desk: Come See Us.
What could they possibly
reprimand him for? He was a fast and
diligent worker, with enough gumption to
use a little imaginative reasoning when
some economic nuisance called for it. He
was punctual and tidy, not overly familiar
with his co-workers; quite simply, the best
employee they had. Rather than
indignation, though, Samuel only felt fear.
To buy himself time, he crumpled a few
clean papers from his ledger, and walked
the narrow aisles between cubicles to
throw them in the hallway garbage bin.
He returned to find both bosses,

40

nR

45

50 Dombey and Son *, as hed nicknamed

them, at his desk. Dombeys German


sense of humor failed to translate, at least
to Samuel, who always overdid his laugh
to mask confusion. Son, whose current
55 prestige was pure nepotism, looked at
Samuel with the coldness that cloaked all
of his dealings, as if he knew he was inept
and needed to compensate.

file://E:\\d4.htm

2006-11-12

13/23

The Official SAT Online Course

Tyne, Dombey said, we need to


60 talk about the Olds account.
Samuel pinched the brim of his hat
with his thumbs. Ah, yes. Sorry, yes. I
think, sir, I handed that in before I took
day leave for my uncles funeral.
65
It contains a dreadful error, said
Son, blinking violently behind his glasses.
He jerked the report at Samuel.
There it was, plain as day, on page
six. A miscalculation Samuel must have
70 made while thinking about Jacobs death
and the house. He stood there, hat in
hand, aghast.
We realize, continued Son, that
the job sometimes gets stressful. That,
75 per se, there are times when one cannot
always be as on-the-ball as is required.
But this defies all. Not only is it not up to
standard, its downright misleading.
That was the way Son spoke, as
80 though he hadnt mastered the
bureaucratic language, wielding phrases
such as per se and not up to standard
like the residue of some management
handbook. Even Dombey seemed
85 perplexed by this at times.
The muscle in Samuels cheek
trembled. He nodded.
We understand youve just
suffered a big loss, Samuel, said
90 Dombey, but as you know this is a
federal workplace. What would happen,
say, if you made this kind of error daily?
Now, were certainly not saying that you
do. But what would happen? Ill tell you
95 what would happen. Youd have ladies
collapsing in ten-hour lines just to get a
loaf of bread to feed their families. Youd
have children skipping school because
there arent enough clothes to go around.
100 Babies dying without milk. Old folks
crumbling in their rockers. Itd be
pandemonium with a capital P
depression. We are the economy. We
answer to the prime minister. There is no
105 room for error here. Dombey scratched
his head and looked wistful. Oh, dont
look so glum.
Again, Samuel nodded.
Son, fearing his role in the

re
d

te

is

eg

nR

file://E:\\d4.htm

2006-11-12

14/23

The Official SAT Online Course

110 reprimand unnecessary, added, We are,

of course, deeply sorry for your loss, but


you must remember our country is in your
hands.
Dombey frowned at Son, and the
115 two men walked off. When they left,
Samuel heard through the divider the
rude laughter of Sally Mather. His face
burning, he sat at his desk, and picking up
his green ledger, tried to make up for the
120 ten minutes lost time.
* Dombey and Son is a novel by the English writer
Charles Dickens.
In line 20, the word painfully is closest in meaning to
(A) laboriously
(B)

critically

ed

13

(C) sensitively

(D) harmfully

ANSWERS

er

acutely

AND EXPLANATIONS

st

(E)

eg
i

Explanation for Correct Answer A :


Choice (A) is correct. In this context, painfully most closely means
laboriously. Tynes job is tallying data and is a gruesome but
inevitable task, descriptions that imply something monotonous and never-ending
but something he has to do. Even so, Tyne performs the task with concentrated
attention because he is an exacting employee and a fast and diligent
worker.

nR

Explanation for Incorrect Answer B :


Choice (B) is incorrect. Although Tynes work might be critical (it seemed at
times that the entire Canadian economy depended on Tyne), Tyne does not
perform his job critically, or in a critical manner. Instead, painfully most
closely means laboriously. Tynes job is tallying data and is a
gruesome but inevitable task, descriptions that imply something monotonous
and never-ending but something he has to do.

Explanation for Incorrect Answer C :


Choice (C) is incorrect. Tynes job does not require Tyne to perform it with
sensitivity. Instead, painfully most closely means laboriously. Tynes job
is tallying data and is a gruesome but inevitable task, descriptions that
imply something monotonous and never-ending but something he has to do.

Explanation for Incorrect Answer D :


Choice (D) is incorrect. Tyne does not perform his job in a harmful way, and
performing the job does not harm Tyne. Instead, painfully most closely means
laboriously. Tynes job is tallying data and is a gruesome but
inevitable task, descriptions that imply something monotonous and never-ending
but something he has to do.

Explanation for Incorrect Answer E :


Choice (E) is incorrect. It would make no sense to say that Tyne performs his job
acutely, or severely. Instead, painfully most closely means
laboriously. Tynes job is tallying data and is a gruesome but
inevitable task, descriptions that imply something monotonous and never-ending
but something he has to do.

file://E:\\d4.htm

2006-11-12

15/23

The Official SAT Online Course

14

In line 22, the word gruesome is closest in meaning to


(A) shocking
(B)

repugnant

(C) frightening

(D) crude

(E)

sensational

ANSWERS

AND EXPLANATIONS

Explanation for Correct Answer B :


Choice (B) is correct. As used in line 22, gruesome most closely means
repugnant. Although Tyne is an exacting employee and a fast and
diligent worker, he dislikes the work. Each day Tyne sits at his desk, painfully
tallying his data and frowning at the work in front of him.

Explanation for Incorrect Answer A :


Choice (A) is incorrect. Tynes work is not shocking or surprising to him. He
performs the work each day. Instead, the work is repugnant. Although Tyne is
an exacting employee and a fast and diligent worker, he dislikes the work.
Each day Tyne sits at his desk, painfully tallying his data and frowning at
the work in front of him.

ed

eg
is
te
r

Explanation for Incorrect Answer C :


Choice (C) is incorrect. Although gruesome can mean frightening, that is
not the meaning here. Tyne is not frightened by his work. He is bored and
disgusted with it.

Explanation for Incorrect Answer D :


Choice (D) is incorrect. Tynes work is not crude. It is boring and
repugnant paperwork.

Explanation for Incorrect Answer E :


Choice (E) is incorrect. Tynes work is not sensational or exciting. On the
contrary, it is boring and repugnant paperwork.

nR

15

The second paragraph implies that Tynes clothes make him appear which of the
following?

(A)

Casual

(B)

Pathetic

(C) Stylish

(D) Proud

(E)

Inappropriate

ANSWERS

AND EXPLANATIONS

Explanation for Correct Answer B :


Choice (B) is correct. Tynes clothes make him appear pathetic. The suit Tyne
wears every day is much too large for him (dwarfed by a monstrous blue suit),
and his hat, which never left his head, is very much out of style (mournful
pre-war bowler). These descriptions imply that the clothes are hand-me-downs or
perhaps purchased from a used-clothing store and certainly that they were not
Tynes originally. The description is meant to suggest that he looks silly.

Explanation for Incorrect Answer A :


Choice (A) is incorrect. Tynes clothes are not casual. Tyne wears a suit and
hat to work.

file://E:\\d4.htm

2006-11-12

16/23

The Official SAT Online Course

Explanation for Incorrect Answer C :


Choice (C) is incorrect. Tynes clothes are certainly not stylish. Instead, they
are unstylish and make Tyne appear pathetic. The suit Tyne wears every day is
much too large for him (dwarfed by a monstrous blue suit), and his hat, which
never left his head, is very much out of style (mournful pre-war bowler).
These descriptions imply that the clothes are hand-me-downs or perhaps purchased
from a used-clothing store and certainly that they were not Tynes originally.

Explanation for Incorrect Answer D :


Choice (D) is incorrect. Instead of proud, Tynes clothes make him appear
pathetic. The suit Tyne wears every day is much too large for him (dwarfed
by a monstrous blue suit), and his hat, which never left his head, is very
much out of style (mournful pre-war bowler). These descriptions imply that the
clothes are hand-me-downs or perhaps purchased from a used-clothing store and
certainly that they were not Tynes originally.

Explanation for Incorrect Answer E :


Choice (E) is incorrect. Tynes clothes are not inappropriate. A suit and hat
are appropriate attire in the workplace. Instead, Tynes clothes make him appear
pathetic. The suit Tyne wears every day is much too large for him (dwarfed
by a monstrous blue suit), and his hat, which never left his head, is very
much out of style (mournful pre-war bowler). These descriptions imply that the
clothes are hand-me-downs or perhaps purchased from a used-clothing store and
certainly that they were not Tynes originally.

(A)

sinister and calculating

(B)

happy and ambitious

Tynes attitude at work could best be described as

(D) cheerful and obedient

professional and morose

eg
is

(E)

te

(C) insubordinate and stubborn

re

16

ANSWERS

AND EXPLANATIONS

Explanation for Correct Answer E :


Choice (E) is correct. Tynes attitude at work is professional but morose. His
professionalism can be inferred not only from the descriptions of Tyne as an
exacting, punctual, and diligent employee but also from his reaction
to the reprimand. When his bosses reprimand Tyne for his error, he neither talks
back nor makes excuses for himself. Instead, he stood there, hat in hand,
aghast and nodded. He then returns to his desk and starts working immediately
to make up for the ten minutes lost time. Although he is professional, Tyne is
morose. In fact, his overt melancholy aggravated his boss, for it made Samuel
hard to approach. This melancholy is not simply a temporary reaction to the
death of his uncle. He walks with a pitiful stride and frowns while he works.
More importantly, his co-workers think he could be driven to commit suicide by
the slurs they flung at him.

nR

Explanation for Incorrect Answer A :


Choice (A) is incorrect. Tyne is neither sinister nor calculating. In fact, he is a
man whose nature seemed to be all windows.

Explanation for Incorrect Answer B :


Choice (B) is incorrect. Tyne is not happy, but morose: His overt melancholy
aggravated his boss, for it made Samuel hard to approach. This melancholy is not
simply a temporary reaction to the death of his uncle. He walks with a pitiful
stride and frowns while he works. Also, Tyne is professional but not ambitious. He
arrived at meetings disillusioned. Nowhere in the passage does it suggest that
Tyne is interested in advancing himself in the company.

Explanation for Incorrect Answer C :


Choice (C) is incorrect. Tyne might be a little stubborn (he puts off the meeting
with his bosses by throwing clean paper in the trash), but he is not insubordinate.
Instead, he is professional about his work. His professionalism can be inferred not
only from the descriptions of Tyne as an exacting, punctual, and
diligent employee but also from his reaction to the reprimand. When his bosses
reprimand Tyne for his error, he neither talks back nor makes excuses for himself.

file://E:\\d4.htm

2006-11-12

17/23

The Official SAT Online Course

Instead, he stood there, hat in hand, aghast and nodded. He then returns to
his desk and starts working immediately to make up for the ten minutes lost
time.

Explanation for Incorrect Answer D :


Choice (D) is incorrect. Tyne is obedient in that he is very professional about his
work, but he is morose, not happy. In fact, his overt melancholy aggravated his
boss, for it made Samuel hard to approach. This melancholy is not simply a
temporary reaction to the death of his uncle. He walks with a pitiful stride and
frowns while he works. More importantly, his co-workers think he could be driven to
commit suicide by the slurs they flung at him.

17

The phrase whose nature seemed to be all windows in lines 27-28 suggests that
(A) Tynes colleagues all had different opinions of him
(B)

Tyne behaved in many inconsistent ways

(C) Tynes true feelings were easy to observe

(D) Tyne did not appear to be a solid person

(E)

Tyne always talked about himself to his colleagues

AND EXPLANATIONS

ed

ANSWERS

Explanation for Correct Answer C :


Choice (C) is correct. The description whose nature seemed to be all windows
means that Tynes feelings were easy to observe. He shows his melancholy so
openly (overt melancholy) that it aggravates his boss, and his co-workers need
only a glance to have much to gloat about. Tyne lacks a public self or,
in other words, he made no attempts to present himself differently from how he
felt. This makes him vulnerable to the observation of his colleagues.

te
r

is

nR
eg

Explanation for Incorrect Answer A :


Choice (A) is incorrect. The description whose nature seemed to be all windows
does not mean that Tynes colleagues all have different opinions of Tyne. In fact,
they all apparently have the same opinion of their colleague: Yet he was so
indispensable in that ministry that his co-workers regretted every slur they flung at
him. The description of Tynes nature means that Tynes feelings were easy to
observe. He shows his melancholy so openly (overt melancholy) that it
aggravates his boss, and his co-workers need only a glance to have much to
gloat about.

Explanation for Incorrect Answer B :


Choice (B) is incorrect. The description whose nature seemed to be all windows
does not mean that Tyne behaved inconsistently. On the contrary, he is apparently
very predictable, behaving the same way every day: There Samuel sat each day
The description instead means that Tynes feelings were easy to observe. He
shows his melancholy so openly (overt melancholy) that it aggravates his boss,
and his co-workers need only a glance to have much to gloat about.

Explanation for Incorrect Answer D :


Choice (D) is incorrect. Tyne, in fact, is a very solid person, at least at work: He is
an exacting employee whose doting, steady logic kept the ministry in
balance. The description whose nature seemed to be all windows means that
Tynes feelings were easy to observe. He shows his melancholy so openly (overt
melancholy) that it aggravates his boss, and his co-workers need only a
glance to have much to gloat about.

Explanation for Incorrect Answer E :


Choice (E) is incorrect. Tyne is a very melancholy man who is not overly familiar
with co-workers, so he must not have talked much about himself. Instead, the
description whose nature seemed to be all windows means that Tynes
feelings were easy to observe. He shows his melancholy so openly (overt
melancholy) that it aggravates his boss, and his co-workers need only a
glance to have much to gloat about.

18

file://E:\\d4.htm

In line 41, the word familiar is closest in meaning to

2006-11-12

18/23

The Official SAT Online Course

(A)

common

(B)

expected

(C) forward

(D) natural

(E)

recognizable

ANSWERS

AND EXPLANATIONS

Explanation for Correct Answer C :


Choice (C) is correct. From the context and other descriptions of Tyne, the word
familiar most closely means forward. Therefore, the statement means that
Tyne does not share his private life with his co-workers and does not inquire into
theirs. The adverbs not overly imply that Tyne is probably polite but certainly
not chummy. Descriptions that show Tynes professionalism (exacting,
fast and diligent) also support this idea: Tyne is at work to work, not to pass
the time in conversations with his colleagues.

ed

Explanation for Incorrect Answer A :


Choice (A) is incorrect. The word familiar can mean common, but that is
not the meaning here. Instead, from the context and other descriptions of Tyne, the
word familiar most closely means forward. Therefore, the statement
means that Tyne does not share his private life with his co-workers and does not
inquire into theirs. The adverbs not overly imply that Tyne is probably polite
but certainly not chummy.

st
er

Explanation for Incorrect Answer B :


Choice (B) is incorrect. Although familiar can mean expected (as in The
old house was a familiar sight), that is not the meaning in this sentence. Instead,
the word familiar most closely means forward. Therefore, the statement
means that Tyne does not share his private life with his co-workers and does not
inquire into theirs. The adverbs not overly imply that Tyne is probably polite
but certainly not chummy.

eg
i

Explanation for Incorrect Answer D :


Choice (D) is incorrect. The word familiar does not mean natural here.
Instead, the word familiar most closely means forward. Therefore, the
statement means that Tyne does not share his private life with his co-workers and
does not inquire into theirs. The adverbs not overly imply that Tyne is probably
polite but certainly not chummy.

nR

Explanation for Incorrect Answer E :


Choice (E) is incorrect. The word familiar can mean recognizable, but that
is not the meaning here. Instead, the word familiar most closely means
forward. Therefore, the statement means that Tyne does not share his private
life with his co-workers and does not inquire into theirs. The adverbs not overly
imply that Tyne is probably polite but certainly not chummy.

19

The description of Tynes job performance in lines 37-43 primarily serves to


(A)

illustrate that Tyne enjoyed doing his job

(B)

imply that Tyne deserves a promotion

(C) suggest that Tynes work habits are generally beyond reproach

(D) indicate that Tynes bosses will probably not discipline him

(E)

demonstrate that Tynes bosses have always liked him

ANSWERS

AND EXPLANATIONS

Explanation for Correct Answer C :


Choice (C) is correct. The description of Tynes work performance (fast and
diligent, punctual and tidy, the best employee) suggests that Tynes
work is generally beyond reproach. The author uses this description of Tynes
usual good behavior at work to suggest that Tyne is not usually summoned by his
bosses to be reprimanded.

file://E:\\d4.htm

2006-11-12

19/23

The Official SAT Online Course

Explanation for Incorrect Answer A :


Choice (A) is incorrect. Nothing in the description of Tynes work habits indicates
that that Tyne enjoyed his job. In fact, other statements suggest that he does not
like his work (Samuel sat each day frowning at the gruesome but inevitable
task ahead). Instead, the description in lines 3743 (fast and diligent,
punctual and tidy, the best employee) suggests that Tynes work is
generally beyond reproach.

Explanation for Incorrect Answer B :


Choice (B) is incorrect. For the most part, this description does not suggest that
Tyne does anything beyond his job; occasionally he uses a little imaginative
reasoning to solve a problem. Instead of suggesting that Tyne deserves a
promotion, the description (fast and diligent, punctual and tidy, the
best employee) suggests that Tynes work is generally beyond reproach.

Explanation for Incorrect Answer D :


Choice (D) is incorrect. The description in lines 3743 certainly indicates that Tyne
does not deserve a reprimand, but the description does not indicate whether or not
he will actually receive one. Instead, the description of Tynes work habits (fast
and diligent, punctual and tidy) indicates only that his performance is
generally beyond reproach.

ed

Explanation for Incorrect Answer E :


Choice (E) is incorrect. Nothing in this description suggests that Tynes bosses
liked him. In fact, Dombey finds Tyne hard to approach, and Son appears to take
himself and his status too seriously to like anyone. Instead, the description in lines
3743 (fast and diligent, punctual and tidy, the best employee)
suggests that Tynes work is generally beyond reproach.

The gesture in lines 45-48 (To buybin) is meant primarily to indicate Tynes
(A)

desire to look busy

(B)

unwillingness to meet with his bosses

eg
i

20

st
er

(C) avoidance of work

(D) attempt to escape punishment

dissatisfaction with his surroundings

nR

(E)

ANSWERS

AND EXPLANATIONS

Explanation for Correct Answer B :


Choice (B) is correct. Tyne crumples clean paper and walks to the hallway trash bin
to dispose of it to buy himself time because he is unwilling to meet with his
bosses. He simply does not like to deal with his bosses, whom he has nicknamed
Dombey and Son (a derogatory nickname), partly because he does not
understand Dombeys German sense of humor, and Son treats the workers
with a coldness that cloaked all his dealings. Going to the trash bin to throw
out the paper is a way of delaying the unwanted confrontation with his bosses.

Explanation for Incorrect Answer A :


Choice (A) is incorrect. Tyne crumples clean paper and walks to the hallway trash
bin to dispose of it to buy himself time, not to look busy. Tyne apparently
always has enough work to keep him busy, so he need not try to look busy.

Explanation for Incorrect Answer C :


Choice (C) is incorrect. Tyne crumples clean paper and walks to the hallway trash
bin to dispose of it to buy himself time, not to avoid work. Descriptions of
Tynes work habits (for example, fast and diligent) and his trying to make
up for the ten minutes lost time after the reprimand indicate that Tyne works
hard.

Explanation for Incorrect Answer D :


Choice (D) is incorrect. Tyne crumples clean paper and walks to the hallway trash
bin to dispose of it to buy himself time. He is not trying to avoid punishment
because he has no reason to think that he will be reprimanded. After all, he was the

file://E:\\d4.htm

2006-11-12

20/23

The Official SAT Online Course

best employee they had. Furthermore, he does eventually go meet with his
bosses. If he wanted to avoid punishment, he would have probably waited until they
came to him again.

Explanation for Incorrect Answer E :


Choice (E) is incorrect. Tyne is not dissatisfied with his surroundings; in fact, he is
probably not very aware of his surroundings because he works so diligently.
Although Tyne is not overly thrilled with his work, nothing in the passage reveals his
opinion about his work environment.

21

The narrator most likely refers to Tynes second boss only as Son throughout
the passage in order to
(A)

imply that his authority is undeserved

(B)

indicate his official title at work

(C) reinforce his position of power

(D) suggest a friendship between him and Tyne

(E)

portray him as an approachable manager

ANSWERS

AND EXPLANATIONS

re

Explanation for Correct Answer A :


Choice (A) is correct. Calling Tynes second boss Son, rather than by a title or
even a name, implies that his position is undeserved. Indeed, the author states
earlier that Sons current prestige was pure nepotism, meaning that Son was
given his position as a favor to a relative, not as a reward for hard work or
knowledge or experience. The consistent reference to the second boss as Son
makes him nameless, a non-person.

te

eg
is

Explanation for Incorrect Answer B :


Choice (B) is incorrect. Son is not the official title of Tynes second boss. It is,
in fact, derogatory and implies that the mans position is undeserved. Indeed, the
author states earlier that Sons current prestige was pure nepotism, meaning
that Son was given his position as a favor to a relative, not as a reward for hard
work or knowledge or experience. The consistent reference to the second boss as
Son makes him nameless, a non-person.

nR

Explanation for Incorrect Answer C :


Choice (C) is incorrect. Calling Tynes second boss Son implies that his
position is undeserved; it does not reinforce his position of power. Indeed, the
author states earlier that Sons current prestige was pure nepotism, meaning
that Son was given his position as a favor to a relative, not as a reward for hard
work or knowledge or experience. The consistent reference to the second boss as
Son makes him nameless, a non-person.

Explanation for Incorrect Answer D :


Choice (D) is incorrect. There is no friendship between Tyne and his second boss. In
fact, Son cloaks all of his dealings with coldness, not with friendliness. Instead,
calling Tynes second boss Son, rather than by a title or even a name, implies
that his position is undeserved.

Explanation for Incorrect Answer E :


Choice (E) is incorrect. Tynes second boss is certainly not approachable. In fact,
Son cloaks all his dealings with coldness. As used here, Son is not a kind form
of address; instead, it is derogatory and implies that the mans position is
undeserved.

22

The narrator suggests that Sons comment to Tyne in lines 110-113 demonstrates
(A)

genuine concern for Tynes loss

(B)

a misunderstanding of Tynes situation

(C) a disagreement with Dombey about the error

(D) a desire to be seen as important

file://E:\\d4.htm

2006-11-12

21/23

The Official SAT Online Course

(E)

an effort to help Tyne improve his performance

ANSWERS

AND EXPLANATIONS

Explanation for Correct Answer D :


Choice (D) is correct. As the narrator states, Son fears that his part in the
reprimand has been unnecessary (fearing his role unnecessary) and adds
the comments in lines 110113 to make himself feel important. This idea is
supported by an earlier description of Son: He treats others coldly as if he knew
he was inept and needed to compensate. Sons comments show this coldness
because, in effect, he is saying that Tynes job is more important than anything in
Tynes personal life, even his uncles death.

Explanation for Incorrect Answer A :


Choice (A) is incorrect. Sons comments do not show a genuine concern for Tyne.
The comments actually show Sons coldness because in effect, he is saying that
Tynes job is more important than anything in Tynes personal life, even his
uncles death. Instead, Son fears that his part in the reprimand has been
unnecessary (fearing his role unnecessary) and adds these comments to
make himself feel important.

re

Explanation for Incorrect Answer B :


Choice (B) is incorrect. Sons comments do not show an understanding of Tynes
loss or of Tynes job. The comments actually show Sons coldness because in
effect, he is saying that Tynes job is more important than anything in Tynes
personal life, even his uncles death. Instead, Son fears that his part in the
reprimand has been unnecessary (fearing his role unnecessary) and adds
these comments to make himself feel important.

is
te

Explanation for Incorrect Answer C :


Choice (C) is incorrect. Son does not disagree with Dombey. His comments actually
follow the same line as Dombeys speechexaggerating the significance of their
work and of Tynes job. Instead, Son fears that his part in the reprimand has
been unnecessary (fearing his role unnecessary) and adds the comments in
lines 110113 to make himself feel important.

eg

Explanation for Incorrect Answer E :


Choice (E) is incorrect. Earlier descriptions of Son indicate that he would not be
inclined to help the employees in any way. He treats others coldly as if he knew
he was inept and needed to compensate. Sons comments show this coldness
because, in effect, he is saying that Tynes job is more important than anything in
Tynes personal life, even his uncles death. Instead, Son fears that his part in
the reprimand has been unnecessary (fearing his role unnecessary) and
adds the comments in lines 110113 to make himself feel important.

nR

U
23

Tynes response to his bosses reprimand could best be characterized as


(A)

defensive

(B)

passionate

(C) rude

(D) submissive

(E)

deceitful

ANSWERS

AND EXPLANATIONS

Explanation for Correct Answer D :


Choice (D) is correct. Tynes response to his bosses reprimand is submissive.
When Tyne is shown his error, he does not make excuses for himself or say
anything. He simply stands there hat in hand, aghast. During the reprimand,
Tyne simply nods, again silent. After the reprimand, Tyne does not show anger or
defensiveness (for example, by talking back, muttering to himself, pounding his
desk, or complaining to his co-workers). Instead, Tyne silently returns to his desk,
his face burning, and tries to make up for the ten minutes lost time.

file://E:\\d4.htm

2006-11-12

22/23

The Official SAT Online Course

Explanation for Incorrect Answer A :


Choice (A) is incorrect. Tynes response to his bosses reprimand is submissive,
not defensive. When Tyne is shown his error, he does not make excuses for himself
or say anything. He simply stands there hat in hand, aghast. During the
reprimand, Tyne simply nods, again silent. After the reprimand, Tyne does not
show anger or defensiveness (for example, by talking back, muttering to himself,
pounding his desk, or complaining to his co-workers). Instead, Tyne silently returns
to his desk, his face burning, and tries to make up for the ten minutes lost
time.

Explanation for Incorrect Answer B :


Choice (B) is incorrect. Tynes response is not passionate. In fact, the only signs
of emotions are Tynes trembling cheek and his burning face, signs of fear and
embarrassment, not passion. Instead, Tyne is submissive. When Tyne is shown his
error, he does not make excuses for himself or say anything. He simply stands
there hat in hand, aghast. During the reprimand, Tyne simply nods, again
silent. After the reprimand, Tyne does not show anger or defensiveness (for
example, by talking back, muttering to himself, pounding his desk, or complaining
to his co-workers). Instead, Tyne silently returns to his desk, his face burning,
and tries to make up for the ten minutes lost time.

Explanation for Incorrect Answer C :


Choice (C) is incorrect. Tyne is definitely not rude. His response is submissive.
When Tyne is shown his error, he does not make excuses for himself or say
anything. He simply stands there hat in hand, aghast. During the reprimand,
Tyne simply nods, again silent. After the reprimand, Tyne does not show anger or
defensiveness (for example, by talking back, muttering to himself, pounding his
desk, or complaining to his co-workers). Instead, Tyne silently returns to his desk,
his face burning, and tries to make up for the ten minutes lost time.

eg
is
te
re

Explanation for Incorrect Answer E :


Choice (E) is incorrect. Tyne does not respond to his bosses reprimand
deceitfully, by denying that he made the error or by casting blame on someone
else. Instead, Tyne is submissive. When Tyne is shown his error, he does not make
excuses for himself or say anything. He simply stands there hat in hand,
aghast. During the reprimand, Tyne simply nods, again silent. After the
reprimand, Tyne does not show anger or defensiveness (for example, by talking
back, muttering to himself, pounding his desk, or complaining to his co-workers).
Instead, Tyne silently returns to his desk, his face burning, and tries to make
up for the ten minutes lost time.

nR

24

The author mentions the rude laughter of Sally Mather in line 117 primarily in
order to

(A)

indicate that most of Tynes colleagues dislike him

(B)

imply that Sally Mather has reported Tynes error to his bosses

(C) underscore the humiliation Tyne is experiencing

(D) show that Tynes bosses wanted his colleagues to know about his error

(E)

suggest that Tynes bosses are making fun of him

ANSWERS

AND EXPLANATIONS

Explanation for Correct Answer C :


Choice (C) is correct. The rude laughter of Sally Mather underscores the
humiliation Tyne feels. Immediately following this statement, Tyne is described:
His face burning, he sat at his desk. He neither speaks nor looks with anger or
indignation at the two bosses as they leave, and he does not respond to Sallys
rude laughter. His face is burning, not from anger or indignation, but from deep
humiliation. Not only did he receive a humiliating reprimand from his bosses, but
now he has to endure the fact that his colleague overheard the scolding and is
laughing at him.

Explanation for Incorrect Answer A :


Choice (A) is incorrect. Although Tynes colleagues may dislike him (Just a
glance into Samuels cubicle gave his co-workers much to gloat about), only
one, Sally Mather, has laughed at his reprimand. None of Tynes other colleagues
are mentioned.

file://E:\\d4.htm

2006-11-12

23/23

The Official SAT Online Course


Explanation for Incorrect Answer B :
Choice (B) is incorrect. Sally Mathers rude laughter does not imply that she
reported Tynes error. Tynes co-workers may dislike him, but they consider him
indispensable in that ministry and regretted every slur lest the slights
drive him to suicide. Sally Mathers rude laughter underscores the humiliation
Tyne feels.

Explanation for Incorrect Answer D :


Choice (D) is incorrect. Nothing in the passage indicates that the bosses intend
Tynes colleagues to hear his reprimand. In fact, Tyne had been asked to go the
bosses office (the note on his desk read, Come See Us).

Explanation for Incorrect Answer E :


Choice (E) is incorrect. Nothing in this passage implies that the bosses are making
fun of Tyne. Both bosses take themselves, their jobs, and the reprimand very
seriously. They did not joke during the reprimand, but rather spoke with elevated
self-importance and seriousness.

Privacy Policy

Terms of Use

Contact Us

er
e

Copyright 2006 The College Board. All rights reserved.

Back to Score Report

st

eg
i

nR

file://E:\\d4.htm

2006-11-12

1/23

The Official SAT Online Course

Help | Profile | My Organizer | My Bookmarks | Logout

Answers and Explanations

Back to Score Report

Test Sections

Section 1

View Answers and Explanations

Section 2

Online - Practice Test #4

Section 4

Section 5

Unlike with many animals, humans do not swim by instinct.


(A)

with many animals

Section 7

(B)

what many animals do

Section 8

(C) many animals

Section 6

Section 9

(D) many animals who do it

Section 10

(E)

many animals do

AND EXPLANATIONS

re
d

ANSWERS

Explanation for Correct Answer C :


Choice (C) is correct. It avoids the error of the original by removing the
unnecessary pronoun with so that the sentence logically compares two nouns,
animals and humans.

te

Explanation for Incorrect Answer A :


Choice (A) contains an unnecessary preposition that creates a comparison between
unparallel elements. The preposition with creates a comparison between a
prepositional phrase (with many animals) and a noun (humans), instead
of between a noun (many animals) and another noun (humans).

eg

is

Explanation for Incorrect Answer B :


Choice (B) creates an illogical comparison. The clause what many animals do is
compared to a noun humans. It does not make sense to compare humans to
swimming by instinct (what many animals do).

nR

Explanation for Incorrect Answer D :


Choice (D) contains an improper pronoun and wordiness. The pronoun who
should be used to refer to humans, not to animals or things. Also, the clause
many animals who do it can be replaced with the simpler phrase many
animals.

Explanation for Incorrect Answer E :


Choice (E) creates an illogical comparison. The clause many animals do is
compared to the noun humans. It does not make sense to compare humans to
swimming by instinct (many animals do).

First run in 1867 and still taking place every summer, the Belmont Stakes, a horse
race for thoroughbred three-year-olds, was one of the oldest races in the United
States.
(A)

was

(B)

is

(C) were

(D) are

(E)

has been

ANSWERS

file://E:\\d5.htm

AND EXPLANATIONS

2006-11-12

2/23

The Official SAT Online Course

Explanation for Correct Answer B :


Choice (B) is correct. It avoids the verb-tense error of the original by replacing the
past-tense was with the present-tense is.

Explanation for Incorrect Answer A :


Choice (A) involves an error in verb tense. The present tense should be used to
express an action that occurs regularly. The verb of the main clause (the Belmont
Stakes was one ) should be in the present tense (is) because the race
regularly takes place every summer.

Explanation for Incorrect Answer C :


Choice (C) creates errors in subjectverb agreement and verb tense. The subject
of the main clause, Belmont Stakes, is singular (it is a horse race) and so
requires a singular verb, not the plural verb were. Also the verb of the main
clause should be in the present tense (is) because the race still takes place
every summer.

Explanation for Incorrect Answer D :


Choice (D) creates an error in subjectverb agreement. The subject of the main
clause, Belmont Stakes, is singular (it is a horse race) and so requires the
singular verb is, not the plural verb are.

re

Explanation for Incorrect Answer E :


Choice (E) creates an error in verb tense. The verb of the main clause should be in
the present tense (is), not the present perfect (has been), because the
present tense is used to express an action that occurs regularly, and the race
regularly takes place every summer.

st
e

Folklore scholars think of fables probably originating among the Semitic peoples of
the Middle East, moving first to India and then west to Greece.
(A)

of fables probably originating

(B)

of fables that probably originated

eg
i

(C) that the fables origins were probably

(D) that the origin of fables probably was

that fables probably originated

nR

(E)

ANSWERS

AND EXPLANATIONS

Explanation for Correct Answer E :


Choice (E) is correct. It avoids the errors of the original by providing a noun clause
(that fables probably originated) that serves as the direct object of the verb
think. In the context of the sentence, it does not make sense to say that
scholars think of fables. In addition, the participial phrase that begins with
moving illogically modifies scholars, making the sentence suggest that
scholars are moving first to India and then west to Greece.

Explanation for Incorrect Answer A :


Choice (A) involves an error in modification and faulty logic. In the context of the
sentence, it does not make sense to say that scholars think of fables. In
addition, the participial phrase that begins with moving illogically modifies
scholars, making the sentence suggest that scholars are moving first to
India and then west to Greece.

Explanation for Incorrect Answer B :


Choice (B) involves an error in modification and faulty logic. In the context of the
sentence, it does not make sense to say that scholars think of fables. In
addition, the participial phrase that begins with moving illogically modifies
scholars, making the sentence suggest that scholars are moving first to
India and then west to Greece.

Explanation for Incorrect Answer C :


Choice (C) involves awkward phrasing and an error in modification that produces
faulty logic. The awkward clause that the fables origins probably were can be

file://E:\\d5.htm

2006-11-12

3/23

The Official SAT Online Course

reduced to the simpler clause that fables probably originated. In addition, the
participial phrase that begins with moving illogically modifies the fables
origins, making the sentence suggest that origins are moving first to India
and then west to Greece.

Explanation for Incorrect Answer D :


Choice (D) involves awkward phrasing and an error in modification that produces
faulty logic. The awkward clause that the origin of fables probably was can be
reduced to the simpler clause that fables probably originated. In addition, the
participial phrase that begins with moving illogically modifies the origin,
making the sentence suggest that the origin is moving first to India and then
west to Greece.

Althea Gibson, the first African American tennis player that they recognized as a
world champion, began playing amateur tennis in the 1940's.
(A)

that they recognized as

(B)

that was recognized to be

(C) recognized because she was

(D) to be recognized as

recognizing her as

ANSWERS

AND EXPLANATIONS

ed

(E)

Explanation for Correct Answer D :


Choice (D) is correct. It avoids the pronoun error of the original by replacing the
clause that they recognized as with the infinitive phrase to be recognized
as.

is
te
r

eg

Explanation for Incorrect Answer A :


Choice (A) involves awkward phrasing and an error in pronoun use. The awkward
phrase that they is unnecessary and should be changed to to be. Also, the
plural pronoun they has no antecedent to which it can logically refer.

nR

Explanation for Incorrect Answer B :


Choice (B) involves wordiness and an improper idiom. The clause that was
recognized to be can be replaced with the simpler phrase to be recognized
as. The phrase that was is awkward and unnecessary. In addition, the
phrase to be is not idiomatic after recognized.

Explanation for Incorrect Answer C :


Choice (C) involves faulty logic. It is not logical to say that Althea Gibson was the
first African American tennis player because she was a world champion.

Explanation for Incorrect Answer E :


Choice (E) involves an incorrect verb form that results in faulty logic. The participial
phrase recognizing her as illogically suggests that Althea Gibson recognized
herself as a world champion.

Societies acting through their governments make the rules to state which acts are
illegal, but although war is the most violent of human activities, it has not been
declared illegal by any of the worlds governments or their agencies.
(A)

to state

(B)

stating

(C) when they state

(D) that are stating

(E)

where they state

ANSWERS

file://E:\\d5.htm

AND EXPLANATIONS

2006-11-12

4/23

The Official SAT Online Course

Explanation for Correct Answer B :


Choice (B) is correct. It avoids the error of the original by using an idiomatic verb
form, the present participle stating, to modify the noun rules.

Explanation for Incorrect Answer A :


Choice (A) involves an error in verb form. The participle stating, instead of the
infinitive to state, is the idiomatic verb form to modify the noun rules.

Explanation for Incorrect Answer C :


Choice (C) creates faulty logic and involves an unclear pronoun reference. The
clause when they state modifies the verb make, but it is not clear what
kinds of rules are being discussed. Also, the plural pronoun they has no clear
antecedent; it could refer to societies, governments, or rules.

Explanation for Incorrect Answer D :


Choice (D) creates an unnecessary shift in verb tense. The progressive tense of
are stating is inconsistent with the simple present tense of the main verb
make.

Explanation for Incorrect Answer E :


Choice (E) involves an unclear pronoun reference and improper phrasing. The plural
pronoun they has no clear antecedent; it could refer to societies,
governments, or rules. Also, where should be used to refer to a noun
that is a place, but in this case it modifies a noun that is a thing.

ed

The 2003 Nobel Peace Prize was awarded to Shirin Ebadi, an Iranian lawyer, writer,
and teacher, she gained prominence as an advocate for democracy and human rights.

te
r

(A)

teacher, she gained

(B)

teacher, she had gained

(C) teacher, gaining

eg
is

(D) teacher who gained

(E)

teacher having gained

AND EXPLANATIONS

nR

ANSWERS

Explanation for Correct Answer D :


Choice (D) is correct. It avoids the comma-splice error of the original by turning the
independent clause she rights into a subordinate clause introduced by the
relative pronoun who. The subordinate clause serves as an adjective modifying
the proper noun Shirin Ebadi.

Explanation for Incorrect Answer A :


Choice (A) creates a comma splice. Two independent clauses are connected only by
a comma instead of by a semicolon or a coordinating conjunction and a comma.

Explanation for Incorrect Answer B :


Choice (B) creates a comma splice. Two independent clauses are connected only by
a comma instead of by a semicolon or a coordinating conjunction and a comma.

Explanation for Incorrect Answer C :


Choice (C) involves an unclear modifier that results in faulty logic. Changing the
second independent clause into a participial phrase eliminates the comma-splice
error of the original, but the participial phrase gaining prominence creates
faulty logic. The phrase illogically modifies the subject of the sentence, The 2003
Nobel Peace Prize. It does not make sense to say that the prize gained
prominence.

Explanation for Incorrect Answer E :


Choice (E) involves an ambiguous modifier and awkward phrasing. Changing the
second independent clause into a participial phrase eliminates the comma-splice
error of the original, but the participial phrase having gained prominence
possibly creates faulty logic. It is not clear whether the phrase modifies the subject
of the sentence, The 2003 Nobel Peace Prize or Ebadi. Furthermore, the

file://E:\\d5.htm

2006-11-12

5/23

The Official SAT Online Course

awkward having gained should be changed to who gained so that the


sentence clearly indicates that it was Ebadi who gained prominence

Because its early history is not fully known, origami, the art of folding objects out of
paper without cutting, pasting, or decorating, seems to have developed from the
older art of folding cloth.
(A)

Because

(B)

In that

(C) Since

(D) Although

(E)

As

ANSWERS

AND EXPLANATIONS

Explanation for Correct Answer D :


Choice (D) is correct. It avoids the faulty logic of the original by introducing the
subordinate clause with although, which correctly indicates a contrast between
the idea stated in the subordinate clause (Although known) and the idea
stated in the main clause (origami cloth).

te
re

Explanation for Incorrect Answer A :


Choice (A) involves an imprecise word that creates faulty logic. The subordinating
conjunction because implies a reason. It is not logical to say that origami
developed because its history is not fully known.

Explanation for Incorrect Answer B :


Choice (B) involves an imprecise phrase that creates faulty logic. The phrase In
that is used as a subordinating conjunction and means because. It is not
logical to say that origami developed in that, or because, its history is not
fully known.

eg
is

Explanation for Incorrect Answer C :


Choice (C) involves an imprecise word that creates faulty logic. The subordinating
conjunction since can imply a reason (like because). In the context of the
sentence, it is not logical to say that origami developed since, or because, its
history is not fully known.

nR

Explanation for Incorrect Answer E :


Choice (E) involves an imprecise word that creates faulty logic. The subordinating
conjunction as implies a reason (like because). It is not logical to say that
origami developed because its history is not fully known.

One of the most popular singers of his time, more than twenty languages were
mastered by Paul Robeson, allowing him to perform classical repertory, spirituals, and
folk songs from around the world.
(A)

more than twenty languages were mastered by Paul Robeson, allowing


him to perform

(B)

Paul Robesons mastery of more than twenty languages allowed him to


perform

(C) mastering more than twenty languages allowed Paul Robeson to perform

(D) his mastery of more than twenty languages allowed Paul Robeson to be
performing

(E)

Paul Robeson mastered more than twenty languages, allowing him to


perform

ANSWERS

AND EXPLANATIONS

Explanation for Correct Answer E :


Choice (E) is correct. It is the only option that presents a noun (Paul Robeson)
that can logically be modified by the sentences opening phrase, One of the
most popular singers of his time.

file://E:\\d5.htm

2006-11-12

6/23

The Official SAT Online Course

Explanation for Incorrect Answer A :


Choice (A) involves an error in modification that results in faulty logic. It does not
make logical sense to say that more than twenty languages were One of the
most popular singers of his time.

Explanation for Incorrect Answer B :


Choice (B) involves an error in modification that results in faulty logic. It does not
make logical sense to say that Paul Robesons mastery was One of the
most popular singers of his time.

Explanation for Incorrect Answer C :


Choice (C) involves an error in modification that results in faulty logic. It does not
make logical sense to say that the gerund mastering was One of the most
popular singers of his time.

Explanation for Incorrect Answer D :


Choice (D) involves an error in modification that results in faulty logic. It does not
make logical sense to say that his mastery was One of the most popular
singers of his time.

(A)

having been

(B)

that he was

Babe Ruth is regarded by many having been the greatest baseball player in history,
and he remains arguably the most celebrated figure in North American sports.

re

(C) for being

as

AND EXPLANATIONS

nR
eg

ANSWERS

is

(E)

te

(D) to be

Explanation for Correct Answer E :


Choice (E) is correct. It avoids the improper phrasing of the original by replacing
the verb phrase having been with the preposition as.

Explanation for Incorrect Answer A :


Choice (A) involves improper phrasing and inconsistent verb tense. In the context
of this sentence, the verb regarded must be followed by the preposition as
( regarded by many as the greatest ). In addition, the present progressive
tense of the verb phrase having been is inconsistent with the simple present
tense of the verbs is regarded and remains.

Explanation for Incorrect Answer B :


Choice (B) involves improper phrasing. In the context of this sentence, the verb
regarded must be followed by the preposition as ( regarded by many
as the greatest ). Also, the noun clause that he was is used as a direct
object, even though a direct object cannot be used with a passive verb (is
regarded).

Explanation for Incorrect Answer C :


Choice (C) involves improper phrasing. In the context of this sentence, the verb
regarded must be followed by the preposition as ( regarded by many
as the greatest ), not by the preposition for.

Explanation for Incorrect Answer D :


Choice (D) involves improper phrasing. In the context of this sentence, the verb
regarded must be followed by the preposition as ( regarded by many
as the greatest ), not by an infinitive (to be).

10

file://E:\\d5.htm

Digital technology, as every marketer knows, is synonymous to speed, precision, and


the future.

2006-11-12

7/23

The Official SAT Online Course

(A)

to

(B)

of

(C) with

(D) for

(E)

through

ANSWERS

AND EXPLANATIONS

Explanation for Correct Answer C :


Choice (C) is correct. It avoids the word-choice error of the original by replacing the
preposition to with the preposition with, which is the idiomatic preposition
to follow synonymous.

Explanation for Incorrect Answer A :


Choice (A) involves a word-choice error. The idiomatic preposition to follow
synonymous is with, not to.

Explanation for Incorrect Answer B :


Choice (B) involves a word-choice error. The idiomatic preposition to follow
synonymous is with, not of.

er
e

Explanation for Incorrect Answer D :


Choice (D) involves a word-choice error. The idiomatic preposition to follow
synonymous is with, not for.

Explanation for Incorrect Answer E :


Choice (E) involves a word-choice error. The idiomatic preposition to follow
synonymous is with, not through.

st

eg
i

11

After carefully studying both of the articles, Dr. Rodriguez and Nurse Alba found that
the only difference between them were their titles.

them were their titles

nR

(A)

(B)

them were the titles

(C) the articles were the titles

(D) the articles was that of the titles

(E)

the articles was their titles

ANSWERS

AND EXPLANATIONS

Explanation for Correct Answer E :


Rationale (justify the key): Choice (E) is correct. It avoids the use of an unclear
pronoun and an error in subjectverb agreement. First, the plural pronoun
them in the subordinate clause could refer to the articles or to Dr.
Rodriguez and Nurse Alba. Replacing them with the articles makes clear
which two things were different. Second, the subject of the subordinate clause
(that titles) is the singular noun difference, which requires a singular
verb, was.

Explanation for Incorrect Answer A :


Choice (A) contains an unclear pronoun reference and an error in subjectverb
agreement. The plural pronoun them in the subordinate clause (that
titles) could refer either to the articles or to Dr. Rodriguez and Nurse
Alba. In addition, the subject of the subordinate clause (that titles) is the
singular noun difference, which requires a singular verb, not the plural verb
were.

Explanation for Incorrect Answer B :


Choice (B) involves an unclear pronoun reference and an error in subjectverb

file://E:\\d5.htm

2006-11-12

8/23

The Official SAT Online Course

agreement. The plural pronoun them in the subordinate clause (that


titles) could refer either to the articles or to Dr. Rodriguez and Nurse
Alba. Replacing the second plural pronoun their with the does not solve
this problem. In addition, the subject of the subordinate clause (that titles)
is the singular noun difference, which requires a singular verb, not the plural
verb were.

Explanation for Incorrect Answer C :


Choice (C) creates an error in subjectverb agreement. The subject of the
subordinate clause (that titles) is the singular noun difference, which
requires a singular verb, not the plural verb were.

Explanation for Incorrect Answer D :


Choice (D) involves wordiness. Replacing the original phrase them were their
titles corrects the unclear pronoun reference and the subjectverb disagreement
of the original. However, the phrase the articles was that of their titles is
unidiomatic in this context. The pronoun that refers logically to the
difference, but it is not idiomatic to say that the difference between the articles
was that [the difference] of the titles. The preposition in would be needed
instead.

At

the meeting of the planning board, the councilwoman

was active

seeking a long-term solution to the citys parking problem.

No error

eg
is
te
r

she

assured
her constituents that

ed

12

ANSWERS

AND EXPLANATIONS

Corrected Sentence: At the meeting of the planning board, the councilwoman


assured her constituents that she was actively seeking a long-term solution to the
citys parking problem.

Explanation for Correct Answer D :


The error in this sentence occurs at (D), where an adjective (active) is
incorrectly used to modify a verb (was seeking). An adverb (actively) is
needed instead.

nR

Explanation for Incorrect Answer A :


There is no error at (A). The preposition At properly introduces the
prepositional phrase (At board) that modifies the verb assured.

Explanation for Incorrect Answer B :


There is no error at (B). The past tense of the verb assured appropriately
indicates an action that was completed in the past.

Explanation for Incorrect Answer C :


There is no error at (C). The singular helping verb was agrees with the singular
subject of the dependent clause, she, and the past progressive tense (was
seeking) appropriately expresses an action that was continuing at the time of the
meeting.

Explanation for Incorrect Answer E : There is an error in the sentence.

13

Used in sculpture, carving is the process of reducing substances such as stone, wood,

or ivory to a desired shape by cutting or to chip away unnecessary parts. No error

file://E:\\d5.htm

2006-11-12

9/23

The Official SAT Online Course

ANSWERS

AND EXPLANATIONS

Corrected Sentence: Used in sculpture, carving is the process of reducing


substances such as stone, wood, or ivory to a desired shape by cutting or chipping
away unnecessary parts.

Explanation for Correct Answer D :


The error in this sentence occurs at (D), where there is a lack of parallelism. The
compound objects of the preposition by should be grammatically parallel. The
infinitive to chip is not parallel to the gerund cutting. The gerund
chipping is needed instead.

Explanation for Incorrect Answer A :


There is no error at (A). The singular verb is agrees with its singular subject
carving, and the noun phrase the process appropriately serves as a
subject complement that renames, or describes, the subject of the sentence
(carving).

Explanation for Incorrect Answer B :


There is no error at (B). The preposition to is an idiomatic preposition to follow
reducing, and it properly introduces the prepositional phrase (to shape)
that modifies the verb form reducing.

re

Explanation for Incorrect Answer C :


There is no error at (C). The adjective desired properly modifies the noun
shape, and the noun shape correctly serves as the object of the
preposition to.

eg
is
te

Explanation for Incorrect Answer E : There is an error in the sentence.

14

New York City is an important center of American Buddhism, in part becauseits

residents included immigrants from most of the countries that have strong Buddhist

nR

traditions. No error

ANSWERS

AND EXPLANATIONS

Corrected Sentence: New York City is an important center of American Buddhism,


in part because its residents include immigrants from most of the countries that
have strong Buddhist traditions.

Explanation for Correct Answer C :


The error in this sentence occurs at (C), where there is an improper verb tense. The
tense used in the dependent clause (in part becausetraditions) should be
consistent with the tense used in the main clause (New York CityBuddhism),
but it is not. To fix this problem, the present-tense verb include should be
used.

Explanation for Incorrect Answer A :


There is no error at (A). The article an and the adjective important
properly modify the noun center.

Explanation for Incorrect Answer B :


There is no error at (B). The subordinating conjunction because properly
introduces the clause that modifies the verb of the main clause, is.

Explanation for Incorrect Answer D :

file://E:\\d5.htm

2006-11-12

10/23

The Official SAT Online Course

There is no error at (D). The preposition from is an idiomatic preposition to


follow immigrants, and it properly introduces the prepositional phrase (from
most) that modifies the noun immigrants.

Explanation for Incorrect Answer E : There is an error in the sentence.

15

In addition to being

who wrote

musicologist

music.

a talented pianist and composer, Bla Bartk was a

respected

several books on Hungarian, Slovakian, and Romanian folk

No error

ANSWERS

AND EXPLANATIONS

Corrected Sentence:

re

Explanation for Correct Answer E : There is no error in this sentence.

te

Explanation for Incorrect Answer A :


There is no error at (A). The phrase In addition to is an idiomatic multi-word
preposition and properly introduces the prepositional phrase (In composer)
that modifies the proper noun Bla Bartk.

Explanation for Incorrect Answer B :


aThere is no error at (B). The gerund being appropriately serves as the object
of the multi-word preposition In addition to.

eg

is

Explanation for Incorrect Answer C :


There is no error at (C). The participle respected appropriately serves as an
adjective that modifies the noun musicologist.

nR

Explanation for Incorrect Answer D :


There is no error at (D). The pronoun who properly refers to a person, Bla
Bartk, and the past tense of the verb wrote correctly shows an action
completed in the past.

U
16

Whereas the caterpillars of most butterflies are harmless, moth caterpillars caused an

enormous amount of damage to plants, forest and shade trees, clothing, and

household goods. No error

ANSWERS

AND EXPLANATIONS

Corrected Sentence: Whereas the caterpillars of most butterflies are harmless,


moth caterpillars can cause an enormous amount of damage to plants, forest and
shade trees, clothing, and household goods.

Explanation for Correct Answer B :


The error in this sentence occurs at (B), where there is an improper verb tense.
Statements of general truth, such as the statement about moth caterpillars, require
the present tense (can cause), not the past tense (caused).

file://E:\\d5.htm

2006-11-12

11/23

The Official SAT Online Course

Explanation for Incorrect Answer A :


There is no error at (A). The preposition of properly introduces the
prepositional phrase (of most butterflies) that modifies the noun
caterpillars, and the adjective most properly modifies the noun
butterflies.

Explanation for Incorrect Answer C :


There is no error at (C). The noun damage correctly serves as the object of the
preposition of. The preposition to is an idiomatic preposition to follow
damage, and it properly introduces the prepositional phrase (to goods)
that modifies the noun damage.

Explanation for Incorrect Answer D :


There is no error at (D). The coordinating conjunction and is correctly used to
introduce the last object of the preposition to in the series of four prepositional
objects (plants, trees, clothing, and goods).

Explanation for Incorrect Answer E : There is an error in the sentence.

employingtheir sense of smell to


Homing pigeons can navigate over long distances,

ed

17

ascertain their initial location and using the positionof the Sun determiningthe

er

direction in which they must fly. No error

st

AND EXPLANATIONS

eg
i

ANSWERS

Corrected Sentence: Homing pigeons can navigate over long distances,


employing their sense of smell to ascertain their initial location and using the
position of the Sun to determine the direction in which they must fly.

nR

Explanation for Correct Answer D :


The error in this sentence occurs at (D), where a participle (determining) is
incorrectly used. An infinitive (to determine) is the correct verb form to modify
the participle using.

Explanation for Incorrect Answer A :


There is no error at (A). The verb navigate can be plural or singular and agrees
with the plural subject pigeons. The preposition over is an idiomatic
preposition to follow navigate, and it properly introduces the prepositional
phrase that modifies the verb navigate.

Explanation for Incorrect Answer B :


There is no error at (B). The participle employing properly introduces the
phrase that modifies the noun pigeons.

Explanation for Incorrect Answer C :


There is no error at (C). The noun phrase the position properly serves as the
object of the verb form using.

Explanation for Incorrect Answer E : There is an error in the sentence.

18

Salt is valued not only because of its properties as a condiment and preservative,

file://E:\\d5.htm

2006-11-12

12/23

The Official SAT Online Course

the health of
they are
but also
humans and animals.
essential to
because

ANSWERS

No error

AND EXPLANATIONS

Corrected Sentence: Salt is valued not only because of its properties as a


condiment and preservative, but also because it is essential to the health of
humans and animals.

Explanation for Correct Answer C :


The error in this sentence occurs at (C), where an improper pronoun is used. The
plural pronoun they is used incorrectly to refer to the singular subject salt.
The singular pronoun it should be used instead. In addition, the plural verb
are should be changed to the singular is.

Explanation for Incorrect Answer A :


There is no error at (A). The singular possessive pronoun its correctly refers to
the singular noun salt, and the noun properties correctly serves as the
object of the two-word preposition because of.

er
e

Explanation for Incorrect Answer B :


There is no error at (B). The conjunction but also is the second part of the
correlative conjunction pair not only but also, and it properly introduces the
subordinate clause that modifies the verb is valued.

Explanation for Incorrect Answer D :


There is no error at (D). The noun phrase the health correctly serves as the
object of the preposition to, and the preposition of properly introduces the
prepositional phrase (of animals) that modifies the noun health.

is
t

eg

Explanation for Incorrect Answer E : There is an error in the sentence.

19

nR

After two terms in the Texas State Senate, Barbara Jordan electedto the United

States House of Representatives, where she served from 1973 to 1979. No error

ANSWERS

AND EXPLANATIONS

Corrected Sentence: After two terms in the Texas State Senate, Barbara Jordan
was elected to the United States House of Representatives, where she served from
1973 to 1979.

Explanation for Correct Answer B :


The error in this sentence occurs at (B), where a sentence fragment is created. A
main verb (was elected) is needed to indicate that Barbara Jordan, the subject
of the sentence, is performing the action described in the sentence (After two
terms in the Texas State Senate, Barbara Jordan was elected)

Explanation for Incorrect Answer A :


There is no error at (A). The preposition After properly introduces the
prepositional phrase (After Senate) that modifies the verb elected.

Explanation for Incorrect Answer C :


There is no error at (C). The subordinating conjunction where properly
introduces the clause (where 1979) that modifies the proper noun House
of Representatives.

file://E:\\d5.htm

2006-11-12

13/23

The Official SAT Online Course

Explanation for Incorrect Answer D :


There is no error at (D). The past-tense verb served is used correctly to show
an action that was completed in the past.

Explanation for Incorrect Answer E : There is an error in the sentence.

20

A sweetener, normally

as

are used
either
in almost all bread
sugar or syrup,

an aid to yeast growth.

ANSWERS

for taste
or

No error

AND EXPLANATIONS

Corrected Sentence: A sweetener, normally either sugar or syrup, is used in


almost all bread for taste or as an aid to yeast growth.

Explanation for Correct Answer B :


The error in this sentence occurs at (B), where the plural verb are used does
not agree with the singular subject sweetener. The singular verb is used is
needed instead.

ed

er

Explanation for Incorrect Answer A :


There is no error at (A). The conjunction either properly functions as the first
part of the paired correlative conjunctions either or (either sugar or
syrup).

is
t

eg

Explanation for Incorrect Answer C :


There is no error at (C). The preposition for properly introduces the
prepositional phrase (for taste) that modifies the verb used.

Explanation for Incorrect Answer D :


There is no error at (D). The preposition as is an idiomatic preposition to follow
used, and it properly introduces the prepositional phrase (as growth)
that modifies the verb used.

nR

Explanation for Incorrect Answer E : There is an error in the sentence.

U
21

Willie Dixon's upbeat blues compositions helped usher in the Chicago blues sound

during the 1950s and have become standard numbers for the many young rock

groups trying to achieve popularity during the 1960s. No error

ANSWERS

AND EXPLANATIONS

Corrected Sentence: Willie Dixons upbeat blues compositions helped usher in


the Chicago blues sound during the 1950s and became standard numbers for the
many young rock groups trying to achieve popularity during the 1960s.

Explanation for Correct Answer B :


The error in this sentence occurs at (B), where there is an improper verb tense. The
verb helped and the phrases during the 1950s and during the 1960
s clearly establish the past tense, but the verb have become is in the present
perfect tense. The present perfect tense indicates an action that is still occurring,
but clearly the action of the sentence was completed in the 1960s.

file://E:\\d5.htm

2006-11-12

14/23

The Official SAT Online Course

Explanation for Incorrect Answer A :


There is no error at (A). The adjectives upbeat and blues appropriately
modify the noun compositions.

Explanation for Incorrect Answer C :


There is no error at (C). The participle trying properly modifies the noun
groups, and the word to is part of the infinitive to achieve, which is
the idiomatic verb form to follow a participle (trying to achieve).

Explanation for Incorrect Answer D :


There is no error at (D). The preposition during properly introduces the
prepositional phrase (during the 1960s) that modifies the participle
trying.

Explanation for Incorrect Answer E : There is an error in the sentence.

22

When

the Spanish conquistadors reached Peru in 1532,

it extended
along the Pacific coast of South America from

te
re
d

empire of the Incas,

No error
inland across
the Andes.

modern Ecuador to central Chile and

AND EXPLANATIONS

is

ANSWERS

they encountered
the vast

nR
eg

Corrected Sentence: When the Spanish conquistadors reached Peru in 1532, they
encountered the vast empire of the Incas, which extended along the Pacific coast of
South America from modern Ecuador to central Chile and inland across the Andes.

Explanation for Correct Answer C :


The error in this sentence occurs at (C), where an independent clause (it
extendedAndes) is connected to the preceding independent clause (they
Incas) by only a comma. A subordinating conjunction (which extended) is
needed along with the comma.

Explanation for Incorrect Answer A :


There is no error at (A). The subordinating conjunction When properly
introduces the clause (When 1532) that modifies the verb encountered.

Explanation for Incorrect Answer B :


There is no error at (B). The plural pronoun they correctly refers to the plural
noun conquistadors and properly serves as the subject of the independent
clause. Furthermore, the past-tense verb encountered appropriately shows an
action completed in the past.

Explanation for Incorrect Answer D :


There is no error at (D). The adverb inland properly modifies the verb
extended, and the preposition across properly introduces the prepositional
phrase (across Andes) that modifies the adverb inland.

Explanation for Incorrect Answer E : There is an error in the sentence.

23

Unlike her best friend Margie, making the varsity soccer team as a freshman, Jill

file://E:\\d5.htm

2006-11-12

15/23

The Official SAT Online Course

No error
did not
make the team until her junior year.

ANSWERS

AND EXPLANATIONS

Corrected Sentence: Unlike her best friend Margie, who made the varsity soccer
team as a freshman, Jill did not make the team until her junior year.

Explanation for Correct Answer B :


The error in this sentence occurs at (B), where the improper use of a gerund
(making) results in a sentence fragment. There is no main verb to indicate that
Margie is performing the action described in the first part of the sentence. The
pronoun who, combined with the past-tense verb made, is needed
(UnlikeMargie, who made).

Explanation for Incorrect Answer A :


There is no error at (A). The singular pronoun her correctly refers to the
singular proper noun Jill.

re

Explanation for Incorrect Answer C :


There is no error at (C). The preposition as properly introduces the
prepositional phrase (as a freshman) that modifies the gerund making.
(Please see the explanation of the correct answer to see why making is not
used properly in this sentence.)

te

Explanation for Incorrect Answer D :


There is no error at (D). The past-tense helping verb did appropriately
expresses an action that was completed in the past, and the adverb not is
properly placed before the main verb make.

is

24

nR
eg

Explanation for Incorrect Answer E : There is an error in the sentence.

Although the precise date and place of the origin of baseball are hotly debated, it is

beyond dispute that the New York City boroughs of Manhattan and Brooklyn playan

important role in its early development. No error

ANSWERS

AND EXPLANATIONS

Corrected Sentence: Although the precise date and place of the origin of baseball
are hotly debated, it is beyond dispute that the New York City boroughs of
Manhattan and Brooklyn played an important role in its early development.

Explanation for Correct Answer C :


The error in this sentence occurs at (C), where the present tense of a verb
(play) is incorrectly used to express an action that was completed in the past.
The past-tense verb played is needed instead.

Explanation for Incorrect Answer A :


There is no error at (A). The adverb hotly appropriately modifies the verb
debated, and the past participle (debated) appropriately completes the
passive verb phrase are debated.

Explanation for Incorrect Answer B :


There is no error at (B). The prepositional phrase beyond dispute is idiomatic

file://E:\\d5.htm

2006-11-12

16/23

The Official SAT Online Course

and appropriately serves as an adjective complement describing the subject it.

Explanation for Incorrect Answer D :


There is no error at (D). The singular possessive pronoun its properly refers to
the singular noun baseball.

Explanation for Incorrect Answer E : There is an error in the sentence.

25

By 2003,

more than

684,000 students in the United States had enrolled in charter

schools, publicly funded schools

unencumbered by

ANSWERS

that pledged
better academic results and were

many of the regulations

No error
governing
ordinary public schools.

AND EXPLANATIONS

Corrected Sentence:

re

Explanation for Correct Answer E : There is no error in this sentence.

te

Explanation for Incorrect Answer A :


There is no error at (A). The comparative phrase more than is idiomatic and
properly introduces the complete subject of the sentence, more States.

is

eg

Explanation for Incorrect Answer B :


There is no error at (B). The relative pronoun that properly refers to the noun
schools and introduces the subordinate clause that modifies schools. The
past tense of the verb pledged is consistent with the past tense of the other
verb in the subordinate clause (were).

nR

Explanation for Incorrect Answer C :


There is no error at (C). The adjective unencumbered functions properly as a
subject complement after the linking verb were, and the preposition by
properly introduces the prepositional phrase that modifies the adjective
unencumbered.

Explanation for Incorrect Answer D :


There is no error at (D). The participle governing properly modifies the noun
schools.

26

The common cold, like chickenpox, measles, and many other viral diseases,

can be spread both before and after their symptoms emerge. No error

ANSWERS

AND EXPLANATIONS

Corrected Sentence: The common cold, like chickenpox, measles, and many other
viral diseases, can be spread both before and after its symptoms emerge.

Explanation for Correct Answer D :


The error in this sentence occurs at (D), where the plural possessive pronoun
their does not agree with the singular noun, cold, to which it refers. The
singular possessive pronoun its is needed instead.

file://E:\\d5.htm

2006-11-12

17/23

The Official SAT Online Course

Explanation for Incorrect Answer A :


There is no error at (A). The preposition like properly introduces the
prepositional phrase (like diseases) that modifies the noun cold.

Explanation for Incorrect Answer B :


There is no error at (B). The adjectives many and other properly modify
the noun diseases and combine to form an idiomatic phrase.

Explanation for Incorrect Answer C :


There is no error at (C). The verb phrase can be spread, which can be singular
or plural, agrees with the singular subject, cold. The passive voice of the verb
is appropriately formed with the past participle of a main verb (spread) and a
form of be.

Explanation for Incorrect Answer E : There is an error in the sentence.

Although

he had never played organized sports, whenever Justin,

uncommonly tall,

autograph.

who was

ask him for


attends
an
a basketball game, fans would

ed

27

No error

AND EXPLANATIONS

is
t

ANSWERS

er

eg

Corrected Sentence: Although he had never played organized sports, whenever


Justin, who was uncommonly tall, attended a basketball game, fans would ask him
for an autograph.

Explanation for Correct Answer C :


The error in this sentence occurs at (C), where there is an improper verb form.
Because the action of the dependent clause (whenever game) occurred
before the action of the main clause (fans autograph), the past-tense verb
attended should be used.

nR

Explanation for Incorrect Answer A :


There is no error at (A). The subordinating conjunction Although properly
introduces the dependent clause (Although sports) that modifies the verb of
the main clause (would ask).

Explanation for Incorrect Answer B :


There is no error at (B). The relative pronoun who properly refers to a person
(Justin), and the singular verb was agrees with the singular subject of the
relative clause, who.

Explanation for Incorrect Answer D :


There is no error at (D). The verb form ask is the appropriate form to combine
with the helping verb would. The singular pronoun him agrees with the
singular noun (Justin) to which it refers and is the correct case to serve as the
indirect object of the verb would ask. The preposition for is the idiomatic
preposition to follow ask, and it appropriately introduces the prepositional
phrase (for an autograph) that modifies the verb would ask.

Explanation for Incorrect Answer E : There is an error in the sentence.

28

From its modest beginnings as a series of brief vignettes and its establishment as the

file://E:\\d5.htm

2006-11-12

18/23

The Official SAT Online Course

longest-running prime-time comedy series on television, The Simpsons has

transformed

the way that


both audiences and television programmers

No error

animated sitcom.

ANSWERS

view
the

AND EXPLANATIONS

Corrected Sentence: From its modest beginnings as a series of brief vignettes to


its establishment as the longest-running prime-time comedy series on television,
The Simpsons has transformed the way that both audiences and television
programmers view the animated sitcom.

Explanation for Correct Answer B :


The error in this sentence occurs at (B), where a coordinating conjunction
(and) is improperly used. The preposition to is the idiomatic preposition to
follow From and to introduce the prepositional phrase (to television)
that modifies the proper noun The Simpsons.

ed

Explanation for Incorrect Answer A :


There is no error at (A). The preposition From appropriately introduces the first
prepositional phrase that modifies the proper noun The Simpsons.

er

st

Explanation for Incorrect Answer C :


There is no error at (C). The noun phrase the way properly serves as the direct
object of the verb has transformed, and the relative pronoun that
appropriately introduces the clause that modifies the noun way.

eg
i

Explanation for Incorrect Answer D :


There is no error at (D). The plural verb view agrees with the plural compound
subjects audiences and programmers, and its present tense appropriately
expresses an action that occurs after the action of the present perfect main verb
has transformed.

nR

Explanation for Incorrect Answer E : There is an error in the sentence.

U
29

Venezuela devotes a higher percentage of its budget to education than do other large

Latin American countries such as Mexico and Brazil. No error

ANSWERS

AND EXPLANATIONS

Corrected Sentence:

Explanation for Correct Answer E : There is no error in this sentence.

Explanation for Incorrect Answer A :


There is no error at (A). The singular verb devotes agrees with the singular
subject Venezuela.

Explanation for Incorrect Answer B :


There is no error at (B). The preposition of properly introduces the
prepositional phrase that modifies the noun percentage, and the singular

file://E:\\d5.htm

2006-11-12

19/23

The Official SAT Online Course

pronoun its correctly refers to the singular noun Venezuela.

Explanation for Incorrect Answer C :


There is no error at (C). The word than appropriately completes the
comparative phrase more . . . than, and the plural verb do agrees in
number with the plural noun countries.

Explanation for Incorrect Answer D :


There is no error at (D). The phrase such as is used idiomatically to introduce
the other countries that the sentence talks about.

(1) Aristotle was a great philosopher and scientist. (2) Aristotle lived in Greece
over 2300 years ago. (3) Aristotle was extraordinarily curious about the world
around him. (4) He was also a master at figuring out how things
worked. (5) Aristotle passed it on to his pupil Theophrastus.

(6) Theophrastus was famous among his contemporaries as the co-founder of


the Lyceum, a school in Greece, he is best known today as "the father of
botany." (7) Botany is the branch of science dealing with plants. (8) Two
famous books he wrote were Natural History of Plants and Reasons for
Vegetable Growth. (9) His books were translated from Greek into Latin in
14831800 years after he wrote themthey influenced thousands of readers.

ed

(10) Theophrastus made accurate observations about all aspects of plant life,
including plant structure, plant diseases, seed use, and medicinal
properties. (11) He even described the complex process of plant reproduction
correctly, hundreds of years before it was formally proven. (12) In 1694
Rudolph Jakob Camerarius used experiments to show how plants
reproduced. (13) According to some accounts, Theophrastus did his research in
a garden he maintained at his school which was called the Lyceum. (14) But
Theophrastus also wrote about plants that grew only in other countries, which
he heard about from returning soldiers. (15) By comparing these plants to
plants he grew in his garden, Theophrastus established principles that are still
true today.

er

Which of the following is the best version of the underlined portion of sentence 1 and
sentence 2 (reproduced below)?

eg

30

is
t

Aristotle was a great philosopher and scientist. Aristotle lived in Greece over 2300
years ago.

nR

philosopher and a scientist, living

(B)

philosopher and scientist who lived

(A)

(C) philosopher, and, as a scientist, lived

(D) philosopher and scientist; Aristotle lived

(E)

philosopher, scientist, and lived

ANSWERS

AND EXPLANATIONS

Explanation for Correct Answer B :


Choice (B) is correct. It properly uses a relative clause (who lived) to
connect the information about Aristotle in sentence 2 to the information in sentence
1.

Explanation for Incorrect Answer A :


Choice (A) is unsatisfactory because it results in an awkward sentence.

Explanation for Incorrect Answer C :


Choice (C) is unsatisfactory because it unnecessarily separates the idea of Aristotle
as a philosopher from the idea of Aristotle as a scientist. According to the passage,
he practiced both philosophy and science in Greece 2300 years ago.

Explanation for Incorrect Answer D :


Choice (D) is unsatisfactory because the resulting sentence repeats "Aristotle"
unnecessarily.

file://E:\\d5.htm

2006-11-12

20/23

The Official SAT Online Course

Explanation for Incorrect Answer E :


Choice (E) is unsatisfactory because it creates a non-parallel series. If the sentence
were "Aristotle was a great philosopher, scientist, and mathematician," such a
formulation would be correct, but that is not the case here.

31

What would best replace "it" in sentence 5?


(A)

that

(B)

them

(C) these traits

(D) the world

(E)

his things

ANSWERS

AND EXPLANATIONS

Explanation for Correct Answer C :


Choice (C) is correct. Sentence 3 and sentence 4 refer to qualities, or traits, that
Aristotle possessed. Sentence 5 indicates that Aristotle passed something on to his
pupil Theophrastus, and we know from context that Theophrastus possessed both
of these traits, so it makes sense to say that Aristotle passed the traits on to
Theophrastus.

ed

te
r

Explanation for Incorrect Answer A :


Choice (A) is unsatisfactory because the paragraph specifies two traits that Aristotle
possessed. It is illogical to conclude that he only passed one of them to
Theophrastus.

eg
is

Explanation for Incorrect Answer B :


Choice (B) is unsatisfactory because, while Aristotle passed two traits on to
Theophrastus, the traits need to be identified as such; "them" is ambiguous in
context.

nR

Explanation for Incorrect Answer D :


Choice (D) is unsatisfactory because it makes no sense to say that Aristotle passed
on "the world" to his student.

Explanation for Incorrect Answer E :


Choice (E) is unsatisfactory because, while Aristotle may have passed "his
things" (physical possessions) on to Theophrastus, the passage does not indicate
this. On the other hand, the passage makes it clear that Theophrastus possessed
both curiosity and ingenuity.

32

What word should be inserted between "Greece," and "he" in sentence 6 (reproduced
below)?

Theophrastus was famous among his contemporaries as the co-founder of the


Lyceum, a school in Greece, he is best known today as "the father of botany."

(A)

and

(B)

but

(C) for

(D) thus

(E)

moreover

ANSWERS

AND EXPLANATIONS

Explanation for Correct Answer B :


Choice (B) is correct. The conjunction "but" properly indicates the relationship

file://E:\\d5.htm

2006-11-12

21/23

The Official SAT Online Course

between the two contrasting statements about Theophrastus's fame.

Explanation for Incorrect Answer A :


Choice (A) is unsatisfactory because, while "and" creates a correct sentence, it is
not as effective in context as "but." The word "and" suggests that the two
statements about Theophrastus's fame are complementary facts with no contrast.

Explanation for Incorrect Answer C :


Choice (C) is unsatisfactory because the word "for" makes no sense in context.

Explanation for Incorrect Answer D :


Choice (D) is unsatisfactory because the word "thus" implies a cause-effect
relationship that does not exist.

Explanation for Incorrect Answer E :


Choice (E) is unsatisfactory because the word "moreover" signifies that the second
statement is "in addition" to the first. "Moreover" does not show the necessary
contrast.

Which sentence should be inserted between sentence 8 and sentence 9?

Theophrastus's ideas had a lasting impact.

(B)

Theophrastus's books were instantly successful.

(A)

re

33

(C) The first book is still studied today in botany classes.

(D) They challenged the conclusions of Aristotle.

te

Theophrastus also taught botany to hundreds of students.

ANSWERS

is

(E)

AND EXPLANATIONS

eg

Explanation for Correct Answer A :


Choice (A) is correct. The inserted sentence connects logically to both sentence 8
("ideas" clearly refers to the content of the books) and sentence 9 (the "lasting
impact" is shown by the success of the 1483 translation) and supports the claim
made in the first sentence of the paragraph.

nR

Explanation for Incorrect Answer B :


Choice (B) is unsatisfactory because, while Theophrastus's books may have been
instantly successful (although the fact that he was famous among his
contemporaries for other things suggests otherwise), this success is not mentioned
in the passage, and is not relevant in context.

Explanation for Incorrect Answer C :


Choice (C) is unsatisfactory because the inserted sentence is out of order
chronologically. It would make more sense to discuss Theophrastus's relevance
today after discussing his relevance in the 1400s.

Explanation for Incorrect Answer D :


Choice (D) is unsatisfactory because there is no support in the passage for the
claim that Theophrastus's books challenged the conclusions of Aristotle.

Explanation for Incorrect Answer E :


Choice (E) is unsatisfactory because any sentence inserted between sentence 8 and
sentence 9 should discuss Theophrastus's books (or the ideas in these books), not
his teaching.

34

Which revision appropriately shortens sentence 13 (reproduced below)?

According to some accounts, Theophrastus did his research in a garden he maintained


at his school which was called the Lyceum.

file://E:\\d5.htm

2006-11-12

22/23

The Official SAT Online Course

(A)

Delete "his school which was called".

(B)

Delete "According to some accounts,".

(C) Delete "in a garden he maintained".

(D) Replace "According to some accounts" with "Therefore".

(E)

Replace "Theophrastus" with "he".

ANSWERS

AND EXPLANATIONS

Explanation for Correct Answer A :


Choice (A) is correct. The resulting sentence ends "did his research in a garden
he maintained at the Lyceum." As the reader already knows that the Lyceum is the
name of Theophrastus's school (sentence 6), this revision is appropriate and
necessary.

Explanation for Incorrect Answer B :


Choice (B) is unsatisfactory because "According to some accounts" suggests that
there is some doubt about the information presented. It would not be appropriate
to delete this important qualifier.

ed

Explanation for Incorrect Answer C :


Choice (C) is unsatisfactory because it removes the important fact about the garden
and maintains the redundancy of the original.

er

Explanation for Incorrect Answer D :


Choice (D) is unsatisfactory because "Therefore" makes no sense in context.

is
t

Explanation for Incorrect Answer E :


Choice (E) is unsatisfactory because sentence 12 refers to the botanist Camerarius.
Sentence 13 must then identify Theophrastus by name, not by "he."

eg

35

nR

The third paragraph would be improved by the deletion of which sentence?

(A)

Sentence 10

(B)

Sentence 11

(C) Sentence 12

(D) Sentence 14

(E)

Sentence 15

ANSWERS

AND EXPLANATIONS

Explanation for Correct Answer C :


Choice (C) is correct. Sentence 12 discusses the achievements of Camerarius,
which are only loosely related to those of Theophrastus.

Explanation for Incorrect Answer A :


Choice (A) is unsatisfactory because sentence 10 provides important introductory
information about the details of Theophrastus's achievements.

Explanation for Incorrect Answer B :


Choice (B) is unsatisfactory because sentence 11, which is directly linked to
sentence 10 through the word "even," highlights a key achievement of
Theophrastus.

Explanation for Incorrect Answer D :


Choice (D) is unsatisfactory because if sentence 14 is deleted, the phrase "these
plants" in sentence 15 will make no sense.

file://E:\\d5.htm

2006-11-12

23/23

The Official SAT Online Course


Explanation for Incorrect Answer E :
Choice (E) is unsatisfactory because sentence 15 connects Theophrastus's research
methods to his achievements, thus bringing together the two ideas of the
paragraph.

Back to Score Report

Privacy Policy

Copyright 2006 The College Board. All rights reserved.

Terms of Use

Contact Us

ed

er

is
t

eg

nR

file://E:\\d5.htm

2006-11-12

1/11

The Official SAT Online Course

Help | Profile | My Organizer | My Bookmarks | Logout

Answers and Explanations

Back to Score Report

Test Sections

Section 1

View Answers and Explanations

Section 2

Online - Practice Test #4

Section 4

Section 5

Section 7

(A)

Section 8

(B)

Section 9

pictures

pictures are taken, there are

and

in terms of

left. What is

Section 6

pictures. After

A new roll of film has

(C)

Section 10

(D)
(E)

AND EXPLANATIONS

re

ANSWERS

Explanation for Correct Answer B :


Choice (B) is correct. The new roll of film has

pictures; thus, after

pictures left. It follows that

taken, there are

pictures are

which implies

is
te

Explanation for Incorrect Answer A :


Choice (A) is not correct. The number of pictures on the new roll,

nR
eg

sum of the number of pictures taken,

Solving for

is,

gives

That

not

Explanation for Incorrect Answer C :


Choice (C) is not correct. The new roll of film has

are taken, there are

is equal to the

and the number of pictures left,

pictures; thus, after

pictures left. It follows that

pictures

which implies

not

Explanation for Incorrect Answer D :


the number of pictures taken, will not be
Choice (D) is not correct. In general,

For example, if

equal to

are taken, then

pictures are on the roll and

pictures

pictures will be left. However,

Explanation for Incorrect Answer E :


the number of pictures taken, will not be
Choice (E) is not correct. In general,

equal to

For example, if

are taken, then

The prime number

pictures are on the roll and

pictures

pictures will be left. However,

is a factor of

and is also a factor of

How many possible

values are there for


(A)

One

(B)

Two

Three

file://E:\\d6.htm

2006-11-12

2/11

The Official SAT Online Course

(C)
(D) Four

(E)

Five

ANSWERS

AND EXPLANATIONS

Explanation for Correct Answer B :


Choice (B) is correct. The prime factors of

has two possible values:

Therefore,

and

are

factors of

Explanation for Incorrect Answer A :


has two possible values:
Choice (A) is not correct.

or

Explanation for Incorrect Answer C :


has two possible values:
Choice (C) is not correct.

or

and

factor of both

but

or

The number

is a

is not a prime number.

Explanation for Incorrect Answer D :


has two possible values:
Choice (D) is not correct.

The numbers

or

and

but neither is a prime number.

and

are factors of both

and the prime

and

are

There are other

but they are not prime numbers.

and

numbers that are factors of both

or

re

Explanation for Incorrect Answer E :


has two possible values:
Choice (E) is not correct.

te

is

eg

nR

According to the graph above, for which month was sales minus cost greatest?

(A)

June

(B)

July

(C) August

(D) September

(E)

October

ANSWERS

AND EXPLANATIONS

Explanation for Correct Answer D :


Choice (D) is correct. According to the graph, in September, sales minus cost was

approximately

minus

or

This was greater than sales

minus cost for any of the other four months shown.

Explanation for Incorrect Answer A :

Choice (A) is not correct. In June, sales minus cost was approximately

file://E:\\d6.htm

2006-11-12

3/11

The Official SAT Online Course

Sales minus cost was greater for September.

or

minus

Explanation for Incorrect Answer B :

Choice (B) is not correct. In July, sales minus cost was approximately

Sales minus cost was greater for September.

or

minus

Explanation for Incorrect Answer C :

Choice (C) is not correct. In August, sales minus cost was approximately

Sales minus cost was greater for September.

or

minus

Explanation for Incorrect Answer E :

Choice (E) is not correct. In October, sales minus cost was approximately

Sales minus cost was greater for September.

or

minus

re

te

If the measure of

circle at point

is the center of the circle and segment

In the figure above,

is

is

is tangent to the

how many possible values are

there for

eg

(A)

One

(B)

Two

nR

(C) Three

(D) Four

(E)

More than four

ANSWERS

AND EXPLANATIONS

Explanation for Correct Answer A :

is a radius of the circle, and segment

Choice (A) is correct. Segment

tangent to the circle at point

is

then

and

It follows that

must be

This means that the measure of

must be equal to

is

are perpendicular.

Thus, if the measure of

Therefore, there is only one possible

value for

Explanation for Incorrect Answer B :


The
Choice (B) is not correct. It is not true that there are two possible values for
is the only possible
Therefore,
must be
which is
measure of

value for

Explanation for Incorrect Answer C :


Choice (C) is not correct. It is not true that there are three possible values for

The measure of

which is

must be

Therefore,

is the only

possible value for

Explanation for Incorrect Answer D :


Choice (D) is not correct. It is not true that there are four possible values for

The measure of

file://E:\\d6.htm

which is

must be

Therefore,

is the only

2006-11-12

4/11

The Official SAT Online Course

possible value for

Explanation for Incorrect Answer E :


Choice (E) is not correct. It is not true that there are more than four possible values

to city

The bus fare from city

more for adults than for children. If a group

is

to city

to travel by bus from city

children pay a total of

adults and

of

is the

Therefore,

must be

which is

The measure of
for
only possible value for

what is the cost of the ticket for one adult?


(A)
(B)
(C)
(D)
(E)

ANSWERS

AND EXPLANATIONS

travel from city

adults and

then

to city

to

children pay a total of

re

dollars. If

childs ticket is

Explanation for Correct Answer C :


be the cost, in dollars, of one adult bus ticket from city
Choice (C) is correct. Let
more for adults than for children, so the cost of each
The fare is
to city

This is equivalent to

gives

so the cost of a ticket for one

te

Solving for

adult is

is

Explanation for Incorrect Answer A :

eg

Choice (A) is not correct. If the cost of a ticket for one adult were

then the

A group of

adults and

cost of a ticket for one child would be

to travel from city

nR

children would then pay

to city

However, this cost is

Therefore, the cost of one

not

adult ticket cannot be

Explanation for Incorrect Answer B :

Choice (B) is not correct. If the cost of a ticket for one adult were

A group of

cost of a ticket for one child would be

to travel from

children would then pay

city

to city

then the

adults and

However, this cost is

Therefore, the cost of

not

one adult ticket cannot be

Explanation for Incorrect Answer D :

Choice (D) is not correct. If the cost of a ticket for one adult were

A group of

cost of a ticket for one child would be

to city

However, this cost is

adults and

to travel

children would then pay

from city

then the

not

Therefore, the cost

of one adult ticket cannot be

Explanation for Incorrect Answer E :

Choice (E) is not correct. If the cost of a ticket for one adult were

A group of

cost of a ticket for one child would be

to city

However, this cost is

adults and

to travel

children would then pay

from city

then the

not

Therefore, the cost

of one adult ticket cannot be

file://E:\\d6.htm

2006-11-12

5/11

The Official SAT Online Course

are all equal in size. If the area of


and Figure
The smallest squares in Figure
square centimeters, what is the area, in square centimeters, of
is
Figure

Figure
(A)
(B)

(C)

(D)

ANSWERS

er
e

(E)

AND EXPLANATIONS

is
t

Explanation for Correct Answer C :


of the smallest squares. Since the
consists of
Choice (C) is correct. Figure
square centimeters, the area of each of the smallest
is
area of Figure

squares, in square centimeters, is

Figure

of the smallest

consists of

nR
eg

squares. Therefore, the area, in square centimeters, of Figure

is

Explanation for Incorrect Answer A :


square centimeters,
is
Choice (A) is not correct. Since the area of Figure
of the smallest squares, the area of each of the
and this figure consists of

smallest squares, in square centimeters, is

smallest squares, not

Figure

consists of

of the

Therefore, the area, in square centimeters, of Figure

is

not

Explanation for Incorrect Answer B :


square centimeters,
is
Choice (B) is not correct. Since the area of Figure
of the smallest squares, the area of each of the
and this figure consists of

smallest squares, in square centimeters, is

smallest squares, not

Figure

consists of

of the

Therefore, the area, in square centimeters, of Figure

is

not

Explanation for Incorrect Answer D :


square centimeters,
is
Choice (D) is not correct. Since the area of Figure
of the smallest squares, the area of each of the
and this figure consists of

smallest squares, in square centimeters, is

smallest squares, not

Figure

consists of

of the

Therefore, the area, in square centimeters, of Figure

is

not

Explanation for Incorrect Answer E :


Choice (E) is not correct. This choice could be the result of mistakenly using an area
square centimeters for each of the smallest squares. The smallest squares
of
square centimeters.
each have an area of

file://E:\\d6.htm

2006-11-12

6/11

The Official SAT Online Course

to

1.

Add

2.

Multiply the sum by

3.

Subtract

from the product.

If the steps above are followed in order, which of the following is a simplified
expression for the result?
(A)
(B)
(C)
(D)
(E)

ANSWERS

AND EXPLANATIONS

Explanation for Correct Answer E :

Choice (E) is correct. After step

After step

the result is

the result is

ed

and after step

the result is

te
r

Explanation for Incorrect Answer A :

the result is

Choice (A) is not correct. After step

After step

the result

This incorrect answer could arise from taking the result

of step

is

is

However, step

and subtracting

calls for subtracting

from the result of step

not

eg

Explanation for Incorrect Answer B :

Choice (B) is not correct. After step

nR

is

of step

and adding

the result is

After step

the result

This incorrect answer could arise from taking the result

However, step

calls for subtracting

from

the result of step

Explanation for Incorrect Answer C :


Choice (C) is not correct. This incorrect answer could arise from an algebraic error.

After step

the result is

After step

and after step

the result is

the result is

Explanation for Incorrect Answer D :


Choice (D) is not correct. This incorrect answer could arise from an algebraic error.

After step

the result is

After step

and after step

the result is

the result is

file://E:\\d6.htm

2006-11-12

7/11

The Official SAT Online Course

What is the area of triangle

and

above,

In triangle

(A)

(B)
(C)
(D)
(E)

ANSWERS

AND EXPLANATIONS

Explanation for Correct Answer D :

is the length of altitude

Choice (D) is correct. The height of triangle

is

Therefore, the area of triangle

ed

Explanation for Incorrect Answer A :


Choice (A) is not correct. The area of triangle

is

st
er

for the area of triangle

Explanation for Incorrect Answer B :


Choice (B) is not correct. The product of half of

and

is

is

but

which is

The base of

Therefore, the area of triangle

not

is

not

nR

is

eg
i

Explanation for Incorrect Answer C :


Choice (C) is not correct. The height of triangle

triangle

but the question asks

and half of

is half the product of

the area of triangle

which is

is the length of

The base of triangle

which is

Explanation for Incorrect Answer E :


Choice (E) is not correct. The area of triangle

is

but the question asks

for the area of triangle

U
9

At a science conference, the length of the question period is directly proportional to


-minute question period in a
the length of the session. If there is a
-minute session, how many minutes long is the question period
-minute session at this conference?
in an

Your Response:

Correct Response(s): 20

Explanation:

. Since the length of the question period is directly


The correct answer is
proportional to the length of the session, the relationship between them can be

written as

where

is the length, in minutes, of the question period,

the length, in minutes, of the session, and

question period of

so that

question period is

file://E:\\d6.htm

minutes in a

and

is

is a constant. Since there is a

-minute session, it follows that

Therefore, for an

-minute session, the

minutes long.

2006-11-12

8/11

The Official SAT Online Course

10

feet?

What is the volume, in cubic feet, of a cube with edges of length

Your Response:

Correct Response(s): 1000

Explanation:

cubic feet.

volume of the cube is

11

what is the value of

If

feet, then the

. If a cube has edges of length

The correct answer is

when

Your Response:

Correct Response(s): 15/7, 2.14

ed

Explanation:

. Since

or

The correct answer is

eg
is
te
r

gives

This simplifies to

gives

The answer can be gridded as

equivalent,

substituting

Solving for

or as its rounded decimal

12

nR

To hold a birthday party for


is given by the function

to hold a birthday party for


answer.)

people, the amount a party planner charges, in dollars,


above. How much, in dollars, does the planner charge

people? (Disregard the

sign when gridding your

Your Response:

Correct Response(s): 179

Explanation:

. Since the planner is holding a birthday party for


The correct answer is
Therefore, the planners charge for the party, in dollars, is
people,

13

For all

the function

is defined by

and the function

is defined by

What is the value of

Your Response:

Correct Response(s): 152

Explanation:

file://E:\\d6.htm

2006-11-12

9/11

The Official SAT Online Course

it follows that

and

. Since

The correct answer is

Therefore,

14

Table above shows the number of juniors and seniors taking Physics and Statistics
above gives the enrollment
at Midland High School. The partially completed Table
which are the only periods Physics and Statistics
for these classes in periods and
are taught. If only juniors and seniors take these classes, what is the total number of
students who take Physics or Statistics in period

Your Response:

Correct Response(s): 39

Explanation:

ed

seniors take
juniors and
shows that
. Table
The correct answer is
Physics. Only juniors and seniors take Physics, so the total enrollment for Physics is
are the only periods in which Physics is taught.
and
Periods

the total enrollment in Physics is

Thus, from Table

Therefore, from Table

te
r

and so

is

students who take Physics or Statistics in period

eg
is

15

term of a sequence is given by the formula

The

term of the sequence than the

It follows that

the total number of

How much larger is the

term?

nR

Your Response:

Correct Response(s): 21

Explanation:

The correct answer is

formula

term of the sequence is given by the

. Since the

it follows that the

term of the sequence is

and the

Therefore, the

term is

term is larger than the

term by

16

In the figure above,

is similar to

What is the length of side

Your Response:

Correct Response(s): 10.5, 21/2

file://E:\\d6.htm

2006-11-12

10/11

The Official SAT Online Course

Explanation:

is similar to triangle

. Triangle

or

The correct answer is

so

and

From the figure,

so

This answer can also be

Therefore,

gridded as the equivalent improper fraction

17

What is the

the remainder is

is divided by

When the positive integer

is divided by

remainder when

Your Response:

Correct Response(s): 12

Explanation:

The correct answer is

Thus,

remainder is

. When the positive integer

is divided by

where

can be written as

nonnegative integer. Multiplying both sides of the equation

by

Now rewrite the right-hand side of this equation so that

gives

times some nonnegative integer plus a remainder between

re

it is

the

is some

and

te

is

eg

Therefore, the remainder when

is

nR

18

is divided by

In the

and

-coordinate plane, the distance between points

is

What is one possible value of

Your Response:

Correct Response(s): 3 or 25

Explanation:

The possible correct answers are

. If the distance between points

and

then by the distance formula,

is

and

This equation simplifies to

Squaring both sides now yields

This equation is equivalent to

Therefore,

must equal

then

If

or

Either

or

then

If

may be gridded as the answer.

Back to Score Report

file://E:\\d6.htm

2006-11-12

11/11

The Official SAT Online Course


Privacy Policy

Copyright 2006 The College Board. All rights reserved.

Terms of Use

Contact Us

ed

er

is
t

eg

nR

file://E:\\d6.htm

2006-11-12

1/23

The Official SAT Online Course

Help | Profile | My Organizer | My Bookmarks | Logout

Answers and Explanations

Back to Score Report

Test Sections

Section 1

View Answers and Explanations

Section 2

Online - Practice Test #4

Section 4

Section 5

The critics found the play -------, in that its social message was unfortunately lost in
the awkward twists and turns of the plot.
(A) convoluted

Section 6

Section 7

Section 8

(B)

Section 9

susceptible

(C) suspect

Section 10

(D) condemnatory

preachy

AND EXPLANATIONS

re

ANSWERS

(E)

Explanation for Correct Answer A :


Choice (A) is correct. Convoluted means complicated or having many twists
and turns. If one were to insert this term into the text, the sentence would read
The critics found the play convoluted, in that its social message was
unfortunately lost in the awkward twists and turns of the plot. The sentence
structure indicates that what follows the comma will explain the idea that precedes
it. The missing term must describe a play with twists and turns. Convoluted
precisely describes a play with twists and turns of plot.

is
te

eg

Explanation for Incorrect Answer B :


Choice (B) is incorrect. Susceptible means impressionable or open to influence.
If one were to insert this term into the text, the sentence would read The critics
found the play susceptible, in that its social message was unfortunately lost in the
awkward twists and turns of the plot. Although a play with awkward twists and
and turns could be described as susceptible to misunderstanding, it is illogical to
suggest that a play could be merely susceptible, or impressionable.

nR

Explanation for Incorrect Answer C :


Choice (C) is incorrect. Suspect means regarded with doubt or suspicion. If one
were to insert this term into the text, the sentence would read The critics found
the play suspect, in that its social message was unfortunately lost in the awkward
twists and turns of the plot. The sentence structure indicates that what follows
the comma will explain the idea that precedes it. The missing term must describe a
play with twists and turns. A plot with twists and turns would not necessarily cause
a play to be suspect, or regarded with doubt.

Explanation for Incorrect Answer D :


Choice (D) is incorrect. Condemnatory means declaring something to be wrong
or unfit. If one were to insert this term into the text, the sentence would read
The critics found the play condemnatory, in that its social message was
unfortunately lost in the awkward twists and turns of the plot. The sentence
structure indicates that what follows the comma will explain the idea that precedes
it. The missing term must describe a play with twists and turns. A play that is
condemnatory, or declares something to be wrong, does not necessarily have a
plot with twists and turns.

Explanation for Incorrect Answer E :


Choice (E) is incorrect. Preachy means self-righteous or smug. If one were to
insert this term into the text, the sentence would read The critics found the play
preachy, in that its social message was unfortunately lost in the awkward twists
and turns of the plot. The sentence structure indicates that what follows the
comma will explain the idea that precedes it. The missing term must describe a play
with twists and turns. A plot with twists would not necessarily cause a play to be
preachy, or self-righteous.

file://E:\\d7.htm

2006-11-12

2/23

The Official SAT Online Course

Muriel was so fond of her dog that their brief separation left her not just saddened,
but in a state of -------.
(A) vagary
(B)

abhorrence

(C) bereavement

(D) degeneration

(E)

elation

ANSWERS

AND EXPLANATIONS

Explanation for Correct Answer C :


Choice (C) is correct. Bereavement is sadness caused by a meaningful loss. If
one were to insert this term into the text, the sentence would read Muriel was so
fond of her dog that their brief separation left her not just saddened, but in a state
of bereavement. The structure of the sentence indicates that the missing term
will have a meaning similar to saddened, or a state of sadness.
Bereavement is sadness caused by a lossin this case, Muriels loss or
brief separation from her dog.

re

Explanation for Incorrect Answer A :


Choice (A) is incorrect. Vagary is erratic or unpredictable behavior. If one were
to insert this term into the text, the sentence would read Muriel was so fond of
her dog that their brief separation left her not just saddened, but in a state of
vagary. The structure of the sentence indicates that the missing term will have a
meaning similar to saddened, or a state of sadness. Although someone who is
sad might behave unpredictably, vagary does not refer specifically to sadness
caused by a loss.

te

is

eg

Explanation for Incorrect Answer B :


Choice (B) is incorrect. Abhorrence is disgust for something or someone. If one
were to insert this term into the text, the sentence would read Muriel was so fond
of her dog that their brief separation left her not just saddened, but in a state of
abhorrence. The structure of the sentence indicates that the missing term will
have a meaning similar to saddened, or a state of sadness. The term
abhorrence does not refer specifically to sadness caused by a loss. Additionally,
Muriel is fond of her dog, so it is unlikely that a brief separation would cause her to
feel disgust for her dog.

nR

Explanation for Incorrect Answer D :


Choice (D) is incorrect. Degeneration is a weakness or decline of condition. If
one were to insert this term into the text, the sentence would read Muriel was so
fond of her dog that their brief separation left her not just saddened, but in a state
of degeneration. The structure of the sentence indicates that the missing term
will have a meaning similar to saddened, or a state of sadness. Although
someone who is sad might feel weak, degeneration does not refer specifically
to sadness caused by a loss.

Explanation for Incorrect Answer E :


Choice (E) is incorrect. Elation is a state of extreme happiness. If one were to
insert this term into the text, the sentence would read Muriel was so fond of her
dog that their brief separation left her not just saddened, but in a state of elation.
The structure of the sentence indicates that the missing term will have a meaning
similar to saddened, or a state of sadness. Elation, or extreme happiness,
is the opposite of sadness.

Deliberately designed to be devoid of elaborate carving or other -------, Biedermeier


furniture was known for its -------.
(A)

customization. . uniqueness

(B)

spareness. . starkness

(C) embellishment. . garnishes

(D) ornamentation. . simplicity

file://E:\\d7.htm

2006-11-12

3/23

The Official SAT Online Course

(E)

flamboyance. . flourishes

ANSWERS

AND EXPLANATIONS

Explanation for Correct Answer D :


Choice (D) is correct. Ornamentation is something that adorns or embellishes.
Simplicity is a quality of being simple and unadorned. If one were to insert
these terms into the text, the sentence would read Deliberately designed to be
devoid of elaborate carving or other ornamentation, Biedermeier furniture was
known for its simplicity. The phrase or other indicates that the first missing
term will describe a type of decorative detail that is similar to elaborate
carving. Ornamentation, or adornments and embellishments, are such
decorative details. If Biedermeier furniture was devoid of carving and adornments,
it makes sense to say that it was known for its simplicity, or simple and
unadorned quality.

Explanation for Incorrect Answer A :


Choice (A) is incorrect. Customization is an aspect built according to individual
specifications. Uniqueness is a quality of being the only one. If one were to
insert these terms into the text, the sentence would read Deliberately designed
to be devoid of elaborate carving or other customization, Biedermeier furniture was
known for its uniqueness. Customization, or an aspect built to individual
specifications, usually contributes to an objects uniqueness. It is somewhat
illogical to suggest that a lack of customized details would cause Biedermeier
furniture to be described as unique or one of a kind.

is
te
re
d

Explanation for Incorrect Answer B :


Choice (B) is incorrect. Spareness is a lack of excess or extravagance.
Starkness is a quality of being bare and lacking ornaments. If one were to
insert these terms into the text, the sentence would read Deliberately designed
to be devoid of elaborate carving or other spareness, Biedermeier furniture was
known for its starkness. Although the term starkness makes sense in this
context, the term spareness does not. The phrase or other indicates that
the first missing term will describe a type of decorative detail that is similar to
elaborate carving. The term spareness, or a lack of extravagance, does
not describe a type of decorative detail. Additionally, it does not make sense to say
that something is devoid of spareness.

eg

Explanation for Incorrect Answer C :


Choice (C) is incorrect. An embellishment is an ornamental detail intended to
make something more attractive. Garnishes are ornaments or embellishments.
If one were to insert these terms into the text, the sentence would read
Deliberately designed to be devoid of elaborate carving or other embellishment,
Biedermeier furniture was known for its garnishes. If Biedermeier furniture was
designed to be devoid of elaborate carving or other embellishment, it does not
make sense to say that it was known for its garnishes. Furniture that does not
have elaborate carving or ornamental details does not have garnishes, or
ornaments and embellishments.

nR

Explanation for Incorrect Answer E :


Choice (E) is incorrect. Flamboyance is a strikingly elaborate or colorful style.
Flourishes are decorative details. If one were to insert these terms into the
text, the sentence would read Deliberately designed to be devoid of elaborate
carving or other flamboyance, Biedermeier furniture was known for its flourishes.
If Biedermeier furniture was designed to be devoid of elaborate carving or other
flamboyance, or elements of a strikingly elaborate style, it is unlikely that it
was known for its flourishes, or decorative details. Furniture that does not have
elaborate carving or a strikingly elaborate style would most likely not have
flourishes.

Because she had mistakenly assumed that the disputes between the parties could be
successfully -------, the attorney had not prepared herself for the ------- of a long,
drawn-out public trial.
(A)

mediated . . eventuality

(B)

eased . . probability

(C) exacerbated . . contingency

manipulated . . particularity

file://E:\\d7.htm

2006-11-12

4/23

The Official SAT Online Course

(D)
(E)

foreseen . . inevitability

ANSWERS

AND EXPLANATIONS

Explanation for Correct Answer A :


Choice (A) is correct. To mediate means to bring about a settlement or
agreement in an argument. Eventuality means a possible event or outcome. If
one were to insert these terms into the text, the sentence would read Because
she had mistakenly assumed that the disputes between the parties could be
successfully mediated, the attorney had not prepared herself for the eventuality of
a long, drawn-out public trial. The attorney thought that she could bring about a
settlement in the disputes between the parties, and therefore did not prepare for
the possible outcome of a long trial. It makes sense to suggest that the attorney
did not expect a drawn-out trial if she assumed that the disputes could be
mediated.

Explanation for Incorrect Answer B :


Choice (B) is incorrect. To ease means to make less difficult or painful.
Probability is the chance that a certain event will happen. If one were to insert
these terms into the text, the sentence would read Because she had mistakenly
assumed that the disputes between the parties could be successfully eased, the
attorney had not prepared herself for the probability of a long, drawn-out public
trial. While it might be reasonable to suggest that the handling of a dispute could
be made less difficult, it is illogical to suggest that a dispute itself could be
eased, or made less difficult. Even if a dispute could be eased, its easing
would not necessarily make the occurrence of a trial less probable.

te
re

Explanation for Incorrect Answer C :


Choice (C) is incorrect. To exacerbate means to make something more bitter or
severe. Contingency is something that might happen as a result of something
else. If one were to insert these terms into the text, the sentence would read
Because she had mistakenly assumed that the disputes between the parties could
be successfully exacerbated, the attorney had not prepared herself for the
contingency of a long, drawn-out public trial. Although a long trial could have
been contingent on, or a result of, the disputes between the parties, it is unlikely
that the attorney would have wanted to exacerbate these disputes. On the
contrary, attorneys usually try to settle disputes.

nR
eg

is

Explanation for Incorrect Answer D :


Choice (D) is incorrect. To manipulate means to manage or control something.
Particularity is a detail or individual characteristic. If one were to insert these
terms into the text, the sentence would read Because she had mistakenly
assumed that the disputes between the parties could be successfully manipulated,
the attorney had not prepared herself for the particularity of a long, drawn-out
public trial. A trial might have particularities, or individual characteristics, but
there is no logical connection between the attorneys assumption that she could
manage the disputes and her lack of preparation for the particularity of a long
trial.

Explanation for Incorrect Answer E :


Choice (E) is incorrect. To foresee means to know that something will happen.
Inevitability is something that cannot be avoided. If one were to insert these
terms into the text, the sentence would read Because she had mistakenly
assumed that the disputes between the parties could be successfully foreseen, the
attorney had not prepared herself for the inevitability of a long, drawn-out public
trial. The attorneys ability to foresee disputes would not necessarily affect the
length of the trial, especially if a long trial were an inevitability, or something
that could not be avoided. Furthermore, it is awkward to suggest that disputes
could have been successfully foreseen.

Anthropology was much more than ------- for the novelist Zora Neale Hurston: she
studied at Barnard College with Franz Boas, who is often called the Father of
American Anthropology.
(A)

an obsession

(B)

a career

an avocation

file://E:\\d7.htm

2006-11-12

5/23

The Official SAT Online Course

(C)
(D) an encumbrance

(E)

a commitment

ANSWERS

AND EXPLANATIONS

Explanation for Correct Answer C :


Choice (C) is correct. An avocation is a hobby or something one does for
enjoyment. If one were to insert this term into the text, the sentence would read
Anthropology was much more than an avocation for the novelist Zora Neale
Hurston: she studied at Barnard College with Franz Boas, who is often called the
Father of American Anthropology. The colon indicates that the second part of
the sentence will provide an example to support the idea expressed in the first part
of the sentence. Studying anthropology with someone important in the field
demonstrates a level of interest that goes beyond that of a hobby. Zora Neale
Hurstons choice to study the subject at Barnard College supports the idea that
anthropology was more than an avocation for Hurston.

Explanation for Incorrect Answer A :


Choice (A) is incorrect. An obsession is something one is interested in to an
extreme or even unhealthy extent. If one were to insert this term into the text, the
sentence would read Anthropology was much more than an obsession for the
novelist Zora Neale Hurston: she studied at Barnard College with Franz Boas, who
is often called the Father of American Anthropology. The colon indicates that
the second part of the sentence will provide an example to support the idea
expressed in the first part of the sentence. Someone with an extreme interest in
anthropology would probably choose to study the subject with someone important
in the field, but Zora Neale Hurstons studies at Barnard College do not clearly
demonstrate that anthropology was more than an obsession for her.

ed

er

st

Explanation for Incorrect Answer B :


Choice (B) is incorrect. A career is a type of job one takes on permanently or
for an extended period of time. If one were to insert this term into the text, the
sentence would read Anthropology was much more than a career for the novelist
Zora Neale Hurston: she studied at Barnard College with Franz Boas, who is often
called the Father of American Anthropology. The colon indicates that the
second part of the sentence will provide an example to support the idea expressed
in the first part of the sentence. Someone with a career in anthropology would
most likely have studied the subject in college, but Zora Neale Hurstons studies
at Barnard College do not clearly demonstrate that anthropology was more than a
career to her.

eg
i

nR

Explanation for Incorrect Answer D :


Choice (D) is incorrect. An encumbrance is a burden or hindrance. If one were
to insert this term into the text, the sentence would read Anthropology was much
more than an encumbrance for the novelist Zora Neale Hurston: she studied at
Barnard College with Franz Boas, who is often called the Father of American
Anthropology. The colon indicates that the second part of the sentence will
provide an example to support the idea expressed in the first part of the sentence.
Zora Neale Hurstons choice to study anthropology in college does not support the
idea that she viewed the subject as an encumbrance. It is unlikely that Hurston
would have pursued something she considered to be a hindrance.

Explanation for Incorrect Answer E :


Choice (E) is incorrect. A commitment is a strong dedication to something. If
one were to insert this term into the text, the sentence would read Anthropology
was much more than a commitment for the novelist Zora Neale Hurston: she
studied at Barnard College with Franz Boas, who is often called the Father of
American Anthropology. The colon indicates that the second part of the
sentence will provide an example to support the idea expressed in the first part of
the sentence. Someone with a strong dedication to anthropology would be likely to
have studied the subject in college, but Zora Neale Hurstons studies at Barnard
College do not clearly demonstrate that anthropology was more than a
commitment.

By the standards of the day,


feminism was very powerful in New York
file://E:\\d7.htm

2006-11-12

6/23

The Official SAT Online Course

City at the end of the Second World War


Line in 1945the phrase male chauvinist had
5
already been used there. New York
women were famously independent, and
famously smart. Taverns still
sometimes declined to serve them, it was
true, but even that resilient prejudice had
10 been weakened by the pressures of
*

Prohibition , and it was many long years


since the actress Lily Langtry, refused ale
and a mutton chop at Keens chophouse,
had taken the management to court, and
15 won. Innumerable womens organizations
sustained this liberty, bolstered the sense
of feminine power, provided cultural uplift,
offered professional support, or fulfilled
charitable leanings.

ed

* The period from 1920 to 1933 during which alcoholic


beverages were prohibited by law in the United States.

er

eg
is
t

The author of the passage implies which of the following about the term male
chauvinist?

(A)

That it was first used in New York

(B)

That it was first used in 1945

(C) That its use was common only among college-educated women

(D) That its use was associated with the Second World War

That its use became common outside New York after 1945

nR

(E)

ANSWERS

AND EXPLANATIONS

Explanation for Correct Answer E :


Choice (E) is correct. The author uses two adverbs in the statement about the term
male chauvinist. The adverb already means prior to a specified time, and
the adverb there refers to New York City. Therefore, the term male
chauvinist had been used in New York City before 1945. Furthermore, together
the two adverbs imply the term had not been commonly used anywhere else.
Together these two ideas imply that the term later achieved a wider usage.

Explanation for Incorrect Answer A :


Choice (A) is incorrect. It cannot be stated absolutely that the term male
chauvinist had never been used outside New York City before 1945. The reader
can infer only that its use outside New York City was uncommon.

Explanation for Incorrect Answer B :


Choice (B) is incorrect. In the statement about the term male chauvinist, the
author uses the adverb already, which means prior to or before a specified
time. The specified time is 1945. Therefore, the term had been used before 1945,
not for the first time in 1945.

Explanation for Incorrect Answer C :


Choice (C) is incorrect. The author describes the women in New York City as
famously independent and famously smart, and describes the
activities or effects of womens organizations. The author, however, neither
mentions the educational levels of women nor connects that idea to the term
male chauvinist.

file://E:\\d7.htm

2006-11-12

7/23

The Official SAT Online Course

Explanation for Incorrect Answer D :


Choice (D) is incorrect. The author uses the Second World War simply to set the
time period when feminism was powerful in New York City. However, the author
does not explain how the Second World War might have affected women and
feminism or might have given rise to the term male chauvinist.

It can be inferred from the passage that Prohibition had the effect of
(A)

curbing rights previously afforded to women

(B)

creating new businesses popular among women

(C) making tavern service to women more common

(D) limiting the food offerings in taverns

(E)

inspiring laws that prohibited discrimination

ANSWERS

AND EXPLANATIONS

Explanation for Correct Answer C :


Choice (C) is correct. The refusal of tavern service to women was becoming less,
not more, common. First, the author uses the adverbial phrase still sometimes
to modify declined, meaning that this practice did not occur all the time and
that it was actually becoming unusual. The author further states that the
pressures of Prohibition were one reason that this practice, or resilient
prejudice, had decreased, or weakened.

re
d

is
te

Explanation for Incorrect Answer A :


Choice (A) is incorrect. In fact, the passage says that Prohibition strengthened at
least one right for womenthat of being served in taverns. The refusal of tavern
service to women was becoming less, not more, common.

Explanation for Incorrect Answer B :


Choice (B) is incorrect. The author specifically discusses only one business
tavernsand only in terms of tavern service to women, not ownership of or
investment in taverns by women. The author does not state that Prohibition caused
women to go into the tavern business but does indicate that Prohibition had
decreased, or weakened, the practice by taverns of refusing service to women.

eg

nR

Explanation for Incorrect Answer D :


Choice (D) is incorrect. The only tavern food mentioned in the passage is a mutton
chop, which Keens chophouse had refused to serve to Lily Langtry. This episode
obviously took place before Prohibition.

Explanation for Incorrect Answer E :


Choice (E) is incorrect. The only type of discrimination mentioned in the passage is
the refusal of tavern service to women. Although this practice had decreased, or
weakened, because of Prohibition, it was the pressures of Prohibition, not
a specific law, that had extended the right of tavern service to women. Also, refusal
of tavern service was still practiced after Prohibition (Taverns still sometimes
declined to serve them), and the author does not state that such discrimination
was illegal.

It is commonly claimed that Einstein


"overthrew" Newtonian physics, but this
claim is misleading. Newton's law still
Line describes motions in the Solar System
5
with good precision and is adequate for
programming the trajectories of space
probes to the Moon and planets.
Einstein's theory, however, copes (unlike
Newton's) with objects whose speeds are
10 close to that of light, with the ultra-strong
file://E:\\d7.htm

2006-11-12

8/23

The Official SAT Online Course

gravity that could induce such enormous


speeds, and with the effect of gravity on
light itself. More important, Einstein
deepened our understanding of gravity.
15 To Newton, it was a mystery why all
particles fell at the same rate and followed
identical orbits, but Einstein showed that
these phenomena were a natural
consequence of all bodies taking the same
20 "straightest" path in a space-time curved
by mass and energy.
8

The author places the word "straightest" (line 20) in quotation marks most probably
in order to
(A) make the concept easier for readers to remember
(B)

indicate that the word is not being used literally

(C) stress the differences between Newton's and Einstein's ideas

(D) suggest that the word's meaning may change at some point in the future

show that Einstein is being quoted

ANSWERS

ed

(E)

AND EXPLANATIONS

is
te
r

Explanation for Correct Answer B :


Choice (B) is correct. The quotation marks around straightest indicate that the
word is not meant literally. In fact, the author explains that the straightest
path taken by an object in the spacetime dimension is curved by mass and
energy.

eg

Explanation for Incorrect Answer A :


Choice (A) is incorrect. The use of quotation marks around straightest might
draw the readers attention to the word and therefore make it easier to
remember, but that is not the authors primary purpose here. The quotation
marks indicate that the word is not meant literally. In fact, the author explains that
the straightest path taken by an object in the spacetime dimension is
curved by mass and energy.

nR

Explanation for Incorrect Answer C :


Choice (C) is incorrect. In this sentence, the author is explaining how Einsteins
theory expanded on, not differed from, Newtons law, so the purpose of the
quotation marks around straightest is not to stress differences. The quotation
marks around straightest indicate that the word is not meant literally. In fact,
the author explains that the straightest path taken by an object in the space
time dimension is curved by mass and energy.

Explanation for Incorrect Answer D :


Choice (D) is incorrect. There is nothing in this passage to suggest that the
meaning of the word straightest will change. The quotation marks around
straightest indicate that the word is not meant literally. In fact, the author
explains that the straightest path taken by an object in the spacetime
dimension is curved by mass and energy.

Explanation for Incorrect Answer E :


Choice (E) is incorrect. Quotation marks are indeed often used to show that
someone elses words are being quoted. However, the author does not indicate
(by using a phrase such as Einstein called or In Einsteins words )
that Einstein is being quoted. Instead, the quotation marks around straightest
are used to indicate that the word is not meant literally. In fact, the author explains
that the straightest path taken by an object in the spacetime dimension is
curved by mass and energy.

file://E:\\d7.htm

In the authors view, an advantage of Einsteins theory over Newtons law

2006-11-12

9/23

The Official SAT Online Course

is that it deals with


(A) the Moon
(B)

particles

(C) rapidly-moving objects

(D) planets

(E)

human-engineered spacecraft

ANSWERS

AND EXPLANATIONS

Explanation for Correct Answer C :


Choice (C) is correct. According to the author, Einsteins theory has an advantage
over Newtons law because Einsteins theory deals with rapidly moving objects.
The author explains that Newtons law is goodas far as it goesbut that
Einsteins theory goes beyond Newtons law (unlike Newtons) in dealing
with objects whose speeds are close to that of light.

Explanation for Incorrect Answer A :


Choice (A) is incorrect. According to the author, Einsteins theory did not replace
Newtons law with regard to the Moon. In fact, Newtonian physics are still used to
describe motions in the Solar System and to program the trajectories of
space probes to the Moon and planets.

re

Explanation for Incorrect Answer B :


Choice (B) is incorrect. Both Newtons law and Einsteins theory deal with
particles. In fact, with his theory, Einstein was able to explain what had been a
mystery to Newton: why all particles fell at the same rate and followed identical
orbits.

te

Explanation for Incorrect Answer D :


Choice (D) is incorrect. According to the author, Einsteins theory did not replace
Newtons law with regard to the planets. In fact, Newtonian physics are still used
to describe motions in the Solar System and to program the trajectories of
space probes to the Moon and planets.

is

eg

Explanation for Incorrect Answer E :


Choice (E) is incorrect. Although Einsteins theory is applied to human-engineered
spacecraft, so is Newtons law. In fact, Newtonian physics are still used to
describe motions in the Solar System and to program the trajectories of
space probes to the Moon and planets.

nR

The following passage is excerpted from a biography of the American writer J.D.
Salinger, author of the popular novel The Catcher in the Rye.

In the careers of most modern and


contemporary writers, a pattern of activity
emerges. After writers establish
Line themselves, they produce their work, and
5
periodically, about every three of four
years, release that work by way of a
publisher to the public. There are
exceptions, since publishing-industry
norms may or may not serve idiosyncratic
10 writers. The author may be less prolific,
as in the case of F. Scott Fitzgerald,
because he or she struggles with a piece
of writing for years before being able to
let it go. Or they may write only one
15 book, which ends up being a masterpiece,

file://E:\\d7.htm

2006-11-12

10/23

The Official SAT Online Course

30

35

25

er
e

20

as Harper Lee did with To Kill a


Mockingbird. Or the author may die
before the public comes to appreciate the
full genius of his or her work, as was the
case with Sylvia Plath. However, most
authors, even those inspired by true
genius, write and publish on a regular
basis, primarily because they want to
communicate with an audience. In all
likelihood, the same impulse forces writers
to make themselves available to their
readers in the various ways writers have
access toby giving readings, for
example, or answering fan mail. After all,
should authors be successful, it is the
readers, the people who buy the books,
who allow authors to enjoy the success
they have achieved.
Almost all writers play by the rules
of the game that have evolved in the
publishing-industry establishmentthey
do so, of course, because they want to
stay in the good graces of the publishers,
the people who make the rulesbut, in a
career that has spanned over half a
century, J.D. Salinger has refused to
comply with even the most basic of the
rules. Only onceteaching a class at
Sarah Lawrencehas he appeared before
an audience at all. He has made phone
calls to journalists and has had chance
encounters with some, he has sat for a
deposition or two, but he has never done
a traditional interview. After the initial
printings of his first book, he soon refused
to allow his publisher to use a photograph
of him on the dust jacket of any of his
books. He has never communicated with
his readers; over the years he has even
gone so far as to instruct his agents to
throw away his fan mail without even
bothering to show it to him.
But theres more. At one point in
his career, he decided he didnt want his
stories reproduced in anthologies; then he
demanded that the four books he did
publish between 1951 and 1963 could
remain in print in paperback only if each
edition featured the text between two
plain covers and nothing elseno
advertising copy on the front cover, no

is
t

nR

45

eg

40

U
50

55

60

65

file://E:\\d7.htm

2006-11-12

11/23

The Official SAT Online Course

70

75

80

re

85

glowing blurbs on the back cover, no


biographical information about the author
anywhere, nothing resembling the
trappings a publisher uses to sell and
promote an author and his work. Finally
after 1965, even though he has often
gone out of his way to let the public know
he was continuing to write, he stopped
publishing his work in either magazines or
book form. By doing this, Salinger has
achieved a kind of perverse celebrity: He
has become a famous writer who writes
but doesnt publish.
Consequently, Salingers
reputation, at least in the latter part of his
life, is based not on the books he has
written but on the books he has allowed to
be published. Of course, The Catcher in
the Rye is his major work. Salinger is a
writer of great charm and purposefully
limited scope and a perfection within that
narrow compass, says Harold Bloom.
The Catcher in the Rye struck a nerve for
one generation but it seems to appeal to
sensitive young people who are going to
develop a consciousness and a distrust of
the adult world. Probably it will survive.
Tom Wolfe agrees: The Catcher in the
Rye captures the mood of the adolescent
who wants desperately to fit in but doesnt
want to seem as if he does, who wants to
act flippantly but who, underneath that
flippancy, has a great sorrow.

te

90

is

eg

nR

95

U
10

The pattern of activity mentioned in line 2 refers to


(A)

the path most modern writers take to achieve initial literary success

(B)

the creative process writers use to express their ideas through writing

(C) the way most successful writers continue to supply their readers with new
material

(D) the strategies writers use to convince publishers to distribute their work

(E)

the various ways writers seek to change publishing-industry norms

ANSWERS

AND EXPLANATIONS

Explanation for Correct Answer C :


Choice (C) is correct. The passage contrasts the way most successful writers
interact with their audience with the unusual and deliberate way J.D. Salinger shuns
the public. The first paragraph describes the pattern of activity or the normal
process that successful writers use to produce and to publish their work. According
to the passage, this pattern is the way writers produce their work, and
periodically, about every three or four years, release that work by way of a
publisher to the public.

file://E:\\d7.htm

2006-11-12

12/23

The Official SAT Online Course

Explanation for Incorrect Answer A :


Choice (A) is incorrect. The first paragraph describes the pattern of activity or
the normal process that successful writers use to produce and to publish their work.
This pattern of activity refers to how writers continue to publish works after
becoming successful, not what they did initially to become successful. The passage
explicitly focuses on the pattern that occurs after writers establish themselves.

Explanation for Incorrect Answer B :


Choice (B) is incorrect. The paragraph discusses the way successful writers interact
with the public through publishing their work. The pattern of activity describes
the timing and methods writers use to publish work they have written, not the
process they used to actually create their work. The focus is on the public release,
not the creation, of the work.

Explanation for Incorrect Answer D :


Choice (D) is incorrect. The pattern of activity describes the timing and
methods writers use to publish work they have written. This pattern includes the
release of work by way of a publisher to the public. That publishers want to
publish the work of these successful and established writers is assumed in the
pattern. The paragraph does not include the ways writers might convince publishers
to publish their work in the pattern of activity.

Explanation for Incorrect Answer E :


Choice (E) is incorrect. The pattern of activity refers to the usual way
successful writers periodically write and then release their work through publishers.
This pattern coincides with the publishing-industry norms. The paragraph does
mention that not all writers conform to these norms, but these deviations are
exceptions to the normal pattern of activity.

ed

The phrase struggles with in line 12 is closest in meaning to


(A)

resists

(B)

battles

st
er

11

(E)

eg
i

(D) toils over

(C) strains against

strives toward

AND EXPLANATIONS

nR

ANSWERS

Explanation for Correct Answer D :


Choice (D) is correct. The passage is emphasizing here the amount of time some
writers spend in toiling over their work. F. Scott Fitzgerald is presented as an
exception to the norm because he took a longer time to produce his work. A writer
like Fitzgerald struggles with or, in other words, toils over a piece of
writing for years before being able to let it go.

Explanation for Incorrect Answer A :


Choice (A) is incorrect. The passage is emphasizing here the amount of time some
writers spend in toiling over their work. These writers are not resisting their
writing; they are spending long periods of time to write something they like.

Explanation for Incorrect Answer B :


Choice (B) is incorrect. The passage is emphasizing here the amount of time some
writers spend in toiling over their work. These writers are not battling their
writing; they are spending long periods of time to produce something they like.

Explanation for Incorrect Answer C :


Choice (C) is incorrect. The passage is emphasizing here the amount of time some
writers spend in toiling over their work. These writers are not straining against
their writing; they are spending long periods of time to produce something they
like.

Explanation for Incorrect Answer E :


Choice (E) is incorrect. The passage is emphasizing here the amount of time some
writers toil over their work. These writers are not striving toward their writing;
they are working with their writing over a long period of time to produce something
they like.

file://E:\\d7.htm

2006-11-12

13/23

The Official SAT Online Course

12

The author mentions Fitzgerald, Lee, and Plath in the first paragraph primarily in
order to
(A)

give examples of writers who were inspired by true genius

(B)

illustrate the prevalence of publishing-industry norms

(C) show ways authors have diverged from the typical career path

(D) demonstrate that masterpieces are difficult to produce

(E)

suggest that the publishing industry has changed over the years

ANSWERS

AND EXPLANATIONS

Explanation for Correct Answer C :


Choice (C) is correct. The first paragraph describes the normal process that
successful writers use to produce and to publish their work. According to the
passage, most writers produce their work, and periodically, about every three or
four years, release that work by way of a publisher to the public. The author
mentions Fitzgerald, Lee, and Plath as examples of authors who have diverged from
this typical career path. Fitzgerald was less prolific because he struggled with
his work before letting it go. Lee only published one piece and Plath died before her
genius was fully recognized.

ed

Explanation for Incorrect Answer A :


Choice (A) is incorrect. The first paragraph describes the normal career path of
successful writers. The author states that most authors, even those inspired by
true genius, write and publish on a regular basis. Fitzgerald, Lee, and Plath are
given as examples of writers who did not publish on a regular basis, not as writers
inspired by true genius, though the author clearly considers them extremely
talented.

er

is
t

Explanation for Incorrect Answer B :


Choice (B) is incorrect. The first paragraph describes the normal career path of
successful writers, which conforms to publishing-industry norms. Fitzgerald,
Lee, and Plath are given as examples of writers who diverged from these norms,
not as examples of the prevalence of those norms. The career of each of these
writers was different from that of most other successful authors, according to the
author of the passage.

nR
eg

Explanation for Incorrect Answer D :


Choice (D) is incorrect. Although the lives of Fitzgerald, Lee, and Plath may
demonstrate that masterpieces are difficult to produce, that is not the point the
author of the passage is making here. Fitzgerald, Lee, and Plath are given as
examples of writers who diverged from the normal career pattern of most
successful authors.

Explanation for Incorrect Answer E :


Choice (E) is incorrect. The author of the passage is not trying to show how the
publishing industry has changed over the years. In fact, just the opposite is being
argued. The first paragraph argues that the careers of most authors follow the
same path. Fitzgerald, Lee, and Plath are given as examples of writers who
diverged from the normal career path of most successful authors. But this
divergence is attributed to their own idiosyncrasies, not to changes in the
publishing industry.

13

The word available in line 26 is closest in meaning to


(A)

obtainable

(B)

present

(C) free

(D) accessible

(E)

file://E:\\d7.htm

observable

2006-11-12

14/23

The Official SAT Online Course

ANSWERS

AND EXPLANATIONS

Explanation for Correct Answer D :


Choice (D) is correct. The word available in this context means accessible.
The author is describing ways that writers make themselves accessible to the
public, such as giving readingsor answering fan mail. These are ways in
which the public can directly communicate with authors.

Explanation for Incorrect Answer A :


Choice (A) is incorrect. The word available in this context does not mean
obtainable. The author is describing ways that writers make themselves
accessible to the public, such as giving readingsor answering fan mail. In
other words, these are ways the public can gain access to, not obtain, the writer.

Explanation for Incorrect Answer B :


Choice (B) is incorrect. The word available in this context does not mean
present. The author is describing ways that writers make themselves
accessible to the public, such as giving readingsor answering fan mail. It
would not make sense to say that a writer is present to the public when
answering fan mail.

ed

Explanation for Incorrect Answer C :


Choice (C) is incorrect. The word available in this context does not mean
free. The author is describing ways that writers make themselves accessible to
the public, such as giving readingsor answering fan mail. These are ways in
which the public can gain access to writers, but they are not necessarily free.

er

Explanation for Incorrect Answer E :


Choice (E) is incorrect. The word available in this context does not mean
observable. The author is describing ways that writers make themselves
accessible to the public, such as giving readingsor answering fan mail.
Answering fan mail does not make a writer observable.

is
t

eg

The primary function of paragraph 2 is to


(A)

introduce J.D. Salinger as a writer who refuses to follow conventions

(B)

catalog the ways in which writers communicate with the public

nR

14

(C) illustrate the importance of publishers to a writers career

(D) explain J.D. Salingers attitude towards readers

(E)

indicate the popularity of J. D. Salingers books

ANSWERS

AND EXPLANATIONS

Explanation for Correct Answer A :


Choice (A) is correct. After describing in the first paragraph the ways most writers
interact with the public and publishers, the author of the passage introduces J.D.
Salinger in paragraph 2 as a writer who has refused to follow any of those
conventions. Salinger has rarely appeared publicly and has discouraged his
publishers from promoting his work, a complete break with convention. The
paragraph states that Salinger has never done a traditional interview, has
refused to allow his publisher to use a photograph of him on the dust jacket of
any of his books, has never communicated with his readers, and has even
instructed his agents to throw away his fan mail.

Explanation for Incorrect Answer B :


Choice (B) is incorrect. Although several ways in which writers usually communicate
with the public are listed, paragraph 2 is not meant to be a comprehensive catalog.
The examples are mentioned in order to demonstrate the different ways Salinger
has refused to communicate with the public. The primary purpose of the paragraph
is to argue that Salinger has NOT communicated with the public.

Explanation for Incorrect Answer C :


Choice (C) is incorrect. The author of the passage does argue that publishers are

file://E:\\d7.htm

2006-11-12

15/23

The Official SAT Online Course

usually important for a writers career. The author, however, claims that
Salingers career has been unusual precisely because he has ignored the usual
publishing-industry conventions. Here in paragraph 2, the author discusses ways in
which Salinger has been uncooperative with his publishers.

Explanation for Incorrect Answer D :


Choice (D) is incorrect. The primary purpose of paragraph 2 is to demonstrate that
Salinger has refused to follow the usual conventions of publishing work and
interacting with the public. One of the ways in which Salinger has differed from
other writers is his attitude towards readers, but this is just one of several
differences the paragraph presents. Furthermore, the paragraph does not offer an
explanation for Salingers attitude towards his readers. It simply states that he
has never communicated with them.

Explanation for Incorrect Answer E :


Choice (E) is incorrect. Nowhere in the paragraph does the author discuss the
popularity of Salingers books.

15

More in line 58 refers to


(A)

stories Salinger published in anthologies

(B)

years Salinger wrote novels

(C) ways Salinger distanced himself from his readers

rules of publishing generally obeyed by writers

AND EXPLANATIONS

is
te

ANSWERS

re

(E)

(D) readers who appreciated Salingers work

Explanation for Correct Answer C :


Choice (C) is correct. Paragraph 2 of the passage presents ways Salinger has
refused to communicate with his readers or cooperate with his publisher. Paragraph
3 presents even more ways Salinger has distanced himself from readers. The
author connects the two paragraphs with the phrase, But theres more. The
more ways he has distanced himself listed in the paragraph include refusing to
have his stories reproduced in anthologies, insisting that no biographical
information about himself be published with his work, and finally, refusing to
publish his work altogether.

eg

nR

Explanation for Incorrect Answer A :


Choice (A) is incorrect. Although paragraph 2 does mention anthologies, the
more does not refer to Salingers stories, but rather to the ways in which he
withdrew from his readers. One of those ways was his desire that his stories not be
reproduced in anthologies.

Explanation for Incorrect Answer B :


Choice (B) is incorrect. Although paragraph 2 does mention the years Salinger
published four of his books (between 1951 and 1963), the more does not
refer to these years, but rather to the ways in which he withdrew from his readers.

Explanation for Incorrect Answer D :


Choice (D) is incorrect. The paragraph does not mention the number of readers who
appreciated Salingers work. The more refers to the ways Salinger distanced
himself from those readers, however many of them there were.

Explanation for Incorrect Answer E :


Choice (E) is incorrect. Although the passage does continue to discuss ways writers
usually publish their work, the more does not refer to publishing rules, but
rather to the ways in which Salinger distanced himself from readers by refusing to
abide by these normal publishing conventions.

16

file://E:\\d7.htm

In paragraph 3, the author suggests that Salingers fame


(A)

is due in part to his efforts to prevent his work from being read

(B)

has declined since the publication of his works in the 50s and 60s

2006-11-12

16/23

The Official SAT Online Course

(C) is unwarranted because he published so few books

(D) would be greater if his publisher had promoted his work more vigorously

(E)

is more important to him than his writing

ANSWERS

AND EXPLANATIONS

Explanation for Correct Answer A :


Choice (A) is correct. At the end of paragraph 3, the author states that Salinger
has achieved a kind of perverse celebrity. The paragraph discusses ways Salinger
has tried to prevent his work from being published and promoted. Finally Salinger
stopped publishing his work altogether, even though he has suggested that he
was still continuing to write. By doing this, the author argues, Salinger
has become a famous writer who writes but doesnt publish. The perverse
celebrity refers to the fame that Salinger has achieved in part by his unusual
efforts to remain unknown.

Explanation for Incorrect Answer B :


Choice (B) is incorrect. Nowhere in the paragraph does the author suggest that
Salingers fame has declined. On the contrary, the author argues that Salingers
fame has persisted despite his efforts to keep his works from being read.

re

Explanation for Incorrect Answer C :


Choice (C) is incorrect. Nowhere in the paragraph does the author suggest that
Salingers fame is unwarranted.

is
te

Explanation for Incorrect Answer D :


Choice (D) is incorrect. The author does not suggest that Salingers fame would
be greater if he had been better promoted. On the contrary, the author argues that
the perverse celebrity Salinger has achieved has been, in part, due to the lack
of ways his publishers were allowed to promote his work.

eg

Explanation for Incorrect Answer E :


Choice (E) is incorrect. Nowhere in the passage does the author suggest that fame
was more important to Salinger than his writing. On the contrary, the paragraph
details Salingers efforts to keep his work from being read.

nR

17

According to the passage, Harold Bloom and Tom Wolfe agree that The Catcher in the
Rye

(A)

expresses an enduring attitude of adolescents

(B)

has a limited scope of relevance

(C) was written by an author who was a sensitive adolescent

(D) demonstrates a deep sense of sorrow

(E)

approaches the struggles of youth with flippancy

ANSWERS

AND EXPLANATIONS

Explanation for Correct Answer A :


Choice (A) is correct. In the last paragraph the author quotes Harold Bloom and
Tom Wolfe on the attitude towards adolescence expressed in The Catcher in the
Rye. Bloom states that the novel seems to appeal to sensitive young people who
are going to develop a consciousness and a distrust of the adult world. Wolfe
agrees, stating, The Catcher in the Rye captures the mood of the adolescent who
wants desperately to fit in but doesnt want to seem as if he does. Both quotes
suggest that the attitude towards adolescence expressed in The Catcher in the Rye
is one that has endured across generations.

Explanation for Incorrect Answer B :


Choice (B) is incorrect. In the last paragraph the author quotes Harold Bloom and
Tom Wolfe on The Catcher in the Rye. Although Bloom states that Salingers
writing has a purposefully limited scope and a perfection within that narrow

file://E:\\d7.htm

2006-11-12

17/23

The Official SAT Online Course

compass, Wolfe makes no mention of the scope of The Catcher in the Ryes
relevance.

Explanation for Incorrect Answer C :


Choice (C) is incorrect. In the last paragraph the author quotes Harold Bloom and
Tom Wolfe on The Catcher in the Rye. Although Bloom states that the novel
seems to appeal to sensitive young people who are going to develop a
consciousness and a distrust of the adult world, this does not necessarily imply
that Bloom thinks Salinger himself was a sensitive adolescent. Nor does Wolfe
speculate about Salingers adolescence.

Explanation for Incorrect Answer D :


Choice (D) is incorrect. In the last paragraph the author quotes Harold Bloom and
Tom Wolfe on The Catcher in the Rye. Although Wolfe states that the novel
captures of the mood of the adolescent whounderneathhas a great sorrow,
this does not necessarily imply that Wolfe thinks the book as a whole demonstrates
a deep sense of sorrow. Bloom makes no mention of a sense of sorrow in the novel.

Explanation for Incorrect Answer E :


Choice (E) is incorrect. In the last paragraph the author quotes Harold Bloom and
Tom Wolfe on The Catcher in the Rye. Although Wolfe states that The Catcher in
the Rye captures the mood of the adolescent who wants to act flippantly, this
does not mean that the novel itself treats the struggles of youth in a flippant
manner. On the contrary, both Bloom and Wolfe suggest that the work persuasively
illustrates some of the struggles of youth.

re

The author uses the quotations by Bloom and Wolfe to


(A)

support the argument that Salingers reputation is based on unpublished


books

(B)

demonstrate that The Catcher in the Rye appeals to a young audience

te

18

(C) bolster the claim that The Catcher in the Rye is an important work

(D) illustrate two competing perspectives on Salingers work

eg
is

(E)

undermine the idea that Salinger has successfully avoided publicity

ANSWERS

AND EXPLANATIONS

Explanation for Correct Answer C :


Choice (C) is correct. In the last paragraph of the passage, the author states that
Salingers reputation is built upon the books he has allowed to be published.
Among these, the author claims, The Catcher in the Rye is his major work. The
author supports this claim with quotes by Bloom and Wolfe that discuss the novel
s strengths. Bloom states that the novel struck a nerve for one generation but it
seems to appeal to sensitive young people who are going to develop a
consciousness and a distrust of the adult world. Wolfe states that The Catcher
in the Rye captures the mood of the adolescent who wants desperately to fit in but
doesnt want to seem as if he does.

nR

Explanation for Incorrect Answer A :


Choice (A) is incorrect. The quotations by Bloom and Wolfe both discuss the
strengths of The Catcher in the Rye, a work that Salinger allowed to be
published.

Explanation for Incorrect Answer B :


Choice (B) is incorrect. Although Bloom does state that The Catcher in the Rye
seems to appeal to sensitive young people, the quotations are used to support
the authors claim that The Catcher in the Rye is his major work. Wolfe does
not mention the novels audience.

Explanation for Incorrect Answer D :


Choice (D) is incorrect. The quotations by Bloom and Wolfe both discuss one of The
Catcher in the Ryes strengthsthe convincing way it captures a specific attitude
of adolescence. Bloom and Wolfe do not have competing perspectives; they both
agree that The Catcher in the Rye struck a nerve or captures a mood.

Explanation for Incorrect Answer E :


Choice (E) is incorrect. Nowhere in the passage does the author address whether or
not Salingers attempts to avoid publicity were successful. In the last paragraph of

file://E:\\d7.htm

2006-11-12

18/23

The Official SAT Online Course

the passage, the author states that Salingers reputation is built upon the books
he has allowed to be published. Among these, the author claims, The Catcher in
the Rye is his major work. The author supports this claim with quotations by
Bloom and Wolfe that discuss the novels strengths.

19

The overall tone of the passage could best be described as


(A)

critical

(B)

laudatory

(C) quizzical

(D) informative

(E)

argumentative

ANSWERS

AND EXPLANATIONS

Explanation for Correct Answer D :


Choice (D) is correct. The author of the passage relates all of the details in the
passage in an even, informative tone. First the passage discusses patterns of
publishing, then Salingers unique career, and then the importance of The Catcher
in the Rye. Nowhere in the passage does the author deviate from an informative
tone to give a strongly positive or negative opinion about Salingers behavior or
career.

re

Explanation for Incorrect Answer A :


Choice (A) is incorrect. The author of the passage relates all of the details in the
passage in an even, informative tone. Nowhere in the passage does the author
make critical, or unfavorable, comments on Salingers unusual career or work.

te

nR
eg
is

Explanation for Incorrect Answer B :


Choice (B) is incorrect. The author of the passage relates all of the details in the
passage in an even, informative tone. The details are presented in a matter-of-fact
tone, not one that noticeably lauds or praises Salingers career.

Explanation for Incorrect Answer C :


Choice (C) is incorrect. The author of the passage relates all of the details in the
passage in an even, informative tone. The tone is not quizzical, or mocking, even
when describing Salingers rather unusual career.

Explanation for Incorrect Answer E :


Choice (E) is incorrect. The author of the passage relates all of the details in the
passage in an even, informative tone. Nowhere in the passage does the author take
an argumentative stance toward Salingers work or behavior.

The following passages are about the painter Mary Cassatt (1844-1926). The first
passage is from a biography of Cassatt. The second passage discusses one of
Cassatts paintings entitled At the Opera.

Passage 1

Whether absorbed in a book,


attentive to their needlework, or deep in
personal contemplation, the women in
Line Mary Cassatts art find contentment in
5
their solitude. And it is striking how
frequently Cassatt depicted women on
their own. Their poses and gestures
define the boundaries of their privacy.
They turn their backs to the viewer or
10 incline their heads, attending only to their
file://E:\\d7.htm

2006-11-12

19/23

The Official SAT Online Course

tasks. Their faces, when not obscured by


shadow, express their deep concentration.
Moreover, by presenting her sitters as
deeply involved in their activities, Cassatt
15 reminds the viewer to keep a respectful
distance. Although these women engage
in a variety of activities, they all share a
sense of tranquil self-fulfillment. Each
woman seems at ease with herself,
20 perfectly content to be left alone.
Passage 2

30

te
re

25

Cassatts At the Opera represents a


strong feminist point of view, as it depicts
a lone woman attending a theatrical
event. The young woman is turned away
from us, while our gaze is mirrored in the
background by a sketchily painted man
with opera glasses who also intently
directs his attention toward the attractive
foreground figure. Cassatts woman is not
a beautiful seductress or a passive object
for viewing pleasure. She has her own
independence as she, too, actively looks
beyond the picture frame with a small pair
of opera glasses. Emulating other
Impressionists, Cassatt composed the
picture as a kind of photographic
snapshot, or slice of life, but she
employed the Impressionist emphasis on
the act of seeing to assert the principle of
sexual equality with men.

40

is

eg

20

nR

35

Which of the following best describes the relationship between the two passages?
(A)

Passage 1 connects the ideas expressed in Passage 2 to Cassatts


personal history.

(B)

Passage 1 questions the assumptions about a Cassatt painting made in


Passage 2.

(C) Passage 2 discusses a Cassatt painting that displays the thematic


characteristics presented in Passage 1.

(D) Passage 2 celebrates the artistic qualities of Cassatts paintings that


Passage 1 criticizes.

(E)

Passage 2 offers evidence that contradicts an argument about Cassatt


presented in Passage 1.

ANSWERS

AND EXPLANATIONS

Explanation for Correct Answer C :


Choice (C) is correct. In Passage 1, the author discusses thematic characteristics of
Mary Cassatts art, including women on their own, the boundaries of their
privacy, and their deep involvement in their activities. Specific examples of these
characteristics are given in Passage 2, which discusses one of Cassatts paintings,
At the Opera. The woman in At the Opera is a woman on her own, a lone
woman who has her own independence. The womans privacy is depicted
in her poseshe is turned away from us. Finally, the woman in At the Opera,
who actively looks beyond with a small pair of opera glasses, is deeply
involved in her own activity.

file://E:\\d7.htm

2006-11-12

20/23

The Official SAT Online Course

Explanation for Incorrect Answer A :


Choice (A) is incorrect. The author of Passage 2 provides specific examples of the
thematic characteristics in one of Marys Cassatts paintings, At the Opera,
which represents a strong feminist point of view. Although Passage 1 is from a
biography of Cassatt, this passage does not discuss the artists personal life but,
instead, discusses the thematic characteristics of Cassatts art, such as women
on their own.

Explanation for Incorrect Answer B :


Choice (B) is incorrect. In Passage 1, the author discusses the thematic
characteristics of Mary Cassatts art, including women on their own, the
boundaries of their privacy, and their deep involvement in their activities. The
author of Passage 2 actually supports the statements in Passage 1 by giving specific
examples of these characteristics as they are depicted in one of Cassatts
paintings, At the Opera.

Explanation for Incorrect Answer D :


Choice (D) is incorrect. Both passages discuss the thematic characteristics of Mary
Cassatts art. The author of Passage 1 discusses such thematic characteristics as
women on their own, the boundaries of their privacy, and their deep
concentration. The author of Passage 2 actually provides specific examples of
these characteristics as they are depicted in one of Cassatts paintings, At the
Opera. Neither passage discusses artistic qualities, such as use of color or
technique, of Cassatts paintings.

ed

Explanation for Incorrect Answer E :


Choice (E) is incorrect. Passage 1 does present an argument of sorts about
thematic characteristics of Cassatts art. Passage 2, however, does not contradict
the argument of Passage 1. Rather, Passage 2 presents evidence from one of
Cassatts specific paintings that supports the ideas expressed in Passage 1.

The authors of both passages argue that

is

(A)

Cassatt was only interested in painting women

(B)

Cassatt felt that womens work was unfulfilling

eg

21

te
r

(C) Cassatt thought that most women did not want to be alone

(D) Cassatt painted women primarily to demonstrate their beauty

Cassatt believed in the importance of a womans independence

nR

(E)

ANSWERS

AND EXPLANATIONS

Explanation for Correct Answer E :


Choice (E) is correct. The authors of both passages emphasize the independence of
the women portrayed in Mary Cassatts paintings. The author of Passage 1 argues
that these women find contentment in their solitude and that each seems
perfectly content to be left alone. The author of Passage 2 argues that Cassatts
painting At the Opera represents a strong feminist point of view and asserts
the principle of sexual equality with men. According to Passage 2, the young
woman is not a beautiful seductress or a passive object for viewing pleasure.
Rather, she has her own independence.

Explanation for Incorrect Answer A :


Choice (A) is incorrect. Neither author states that Cassatt was only interested in
painting women. In fact, the reader can infer from Passage 1 that Cassatt had other
subjects because she frequently (but not always) depicted women on their
own. The description of At the Opera in Passage 2 reveals that although the
young woman is in the foreground, a sketchily painted man with opera glasses
is intently viewing her.

Explanation for Incorrect Answer B :


Choice (B) is incorrect. The author of Passage 1 explicitly states that the women in
Cassatts paintings were quite content in their work, whatever it was: Although
these women engage in a variety of activities, they all share a sense of tranquil
self-fulfillment. Although the painting At the Opera does not portray a woman
involved in work, the author of Passage 2 perceives the young woman depicted as
someone at ease with herselfnot a beautiful seductress or a passive object for

file://E:\\d7.htm

2006-11-12

21/23

The Official SAT Online Course

viewing pleasure but someone who has her own independence.

Explanation for Incorrect Answer C :


Choice (C) is incorrect. The authors of both passages indicate that the women
portrayed by Cassatt prefer to be alone. The author of Passage 1 states that these
women find contentment in their solitude and that each seems perfectly
content to be left alone. The author of Passage 1 also argues that the poses and
gestures of the women in Cassatts paintings define the boundaries of their
privacy. These boundaries are not to be crossedviewers are to keep a
respectful distance. The author of Passage 2 describes the young woman in
Cassatts At the Opera as a lone woman, not a lonely woman, and one who
has her own independence. More important, the use of the adverb actively
to describe how she is looking through her opera glasses implies that the young
woman is not concerned about others around her but that that she is very
interested in what she is viewing.

Explanation for Incorrect Answer D :


Choice (D) is incorrect. The authors of both passages indicate that physical beauty
was not the focus or emphasis of the women portrayed by Mary Cassatt. In fact, the
author of Passage 1 informs the reader that the women turn their backs to the
viewer or incline their heads. Further, when Cassatt did reveal their faces, she did
so to show their deep concentration, not their physical beauty. Although the
author of Passage 2 describes the young woman in At the Opera as attractive,
she is not a beautiful seductress or a passive object for viewing pleasure.

re

The author of Passage 1 would probably agree that Cassatt depicted the young
woman in At the Opera as turned away from us (lines 24-25) in order to

(A)

focus on the elegance of the womans profile

(B)

show that the woman is concentrating on the opera

te

22

(C) follow the artistic conventions of the period

portray the idea that the woman is attempting to be seen

eg

(E)

is

(D) create the sense that the woman is uncomfortable

ANSWERS

AND EXPLANATIONS

nR

Explanation for Correct Answer B :


Choice (B) is correct. The author of Passage 1 states that the women portrayed in
Cassatts paintings turn their backs to the viewer or incline their heads,
attending only to their tasks. The author also argues that Cassatt presented her
subjects in such a way as to show that they were deeply involved in their
activities. The young woman in At the Opera is similarly turned away from us
and is actively looking through her opera glasses.

Explanation for Incorrect Answer A :


Choice (A) is incorrect. The author of Passage 1 indicates that physical beauty was
not the focus or emphasis of the women portrayed by Mary Cassatt. In fact, the
author of Passage 1 informs the reader that the women turn their backs to the
viewer or incline their heads, attending only to their tasks. The author argues
that Cassatt presented her subjects in such a way as to show that they were
deeply involved in their activities. The young woman in At the Opera is
similarly turned away from us and is actively looking through her opera
glasses.

Explanation for Incorrect Answer C :


Choice (C) is incorrect. The author of Passage 1 does not mention an artistic
convention, movement, or period in any way. Furthermore, the author of Passage 1
argues that Cassatt posed her subjects (presenting her sitters) the way she did
to show that they were deeply involved in their activities, not to follow an
artistic convention.

Explanation for Incorrect Answer D :


Choice (D) is incorrect. The author of Passage 1 states that each of the women
portrayed by Cassatt seems at ease with herself, definitely not uncomfortable.
Although the author does not mention any of Cassatts paintings in particular in
this passage, the reader can infer that the author would also consider the young
woman in At the Opera at ease with herself because, according to the author,

file://E:\\d7.htm

2006-11-12

22/23

The Official SAT Online Course

Cassatts women all share a sense of tranquil self-fulfillment.

Explanation for Incorrect Answer E :


Choice (E) is incorrect. According to the author of Passage 1, the poses and
gestures of the women portrayed by Cassatt, such as turning their backs to the
viewer or inclining their heads, were meant to ensure their privacy. These women
did not want to be scrutinized; in fact, viewers were to keep a respectful
distance. The young woman in At the Opera is likewise turned away from the
viewer, demonstrating her concentration, not an attempt to be seen.

23

In lines 36-37, the phrase photographic snapshot is probably meant to suggest


that Cassatt
(A)

created the painting to resemble a photograph

(B)

used a style that was a precursor to photographic art

(C) portrayed the woman in an everyday situation

(D) made the woman appear as though she were posing for a photograph

(E)

depicted the woman as caught off guard by the man in the painting

ANSWERS

AND EXPLANATIONS

re

Explanation for Correct Answer C :


Choice (C) is correct. The phrase photographic snapshot implies something
quick and casual. It portrays a simple slice of life. As such, the snapshot of the
young woman in At the Opera captured her doing something that people routinely
did in everyday life, not something that she posed for.

is
te

Explanation for Incorrect Answer A :


Choice (A) is incorrect. Passage 2 does not explicitly discuss the techniques Cassatt
used when creating At the Opera. The phrase photographic snapshot is not
meant to indicate that Cassatt wanted her painting to resemble a photograph, but
rather to suggest that the woman in the painting is engaged in her everyday
activities instead of a formal pose.

eg

nR

Explanation for Incorrect Answer B :


Choice (B) is incorrect. The author of Passage 2 does not discuss photography or
Cassatts influence on later artistic movements. The phrase photographic
snapshot is simply meant to suggest that the woman in painting is engaged in
her everyday activities instead of a formal pose.

Explanation for Incorrect Answer D :


Choice (D) is incorrect. In fact, the opposite is correct. The phrase photographic
snapshot implies something quick and casual. As such, the snapshot of the young
woman in At the Opera captured her doing something that people routinely did in
everyday life, not something that she posed, or held herself in position, for.

Explanation for Incorrect Answer E :


Choice (E) is incorrect. Being caught off guard implies that the person caught
off guard is aware of being seen by another person. According to Passage 2, the
young woman is turned away from us, and we can assume she does not see
us. Likewise, because our gaze is mirrored by the man who is gazing at the
young woman, we can infer that the young woman also does not see him.
Therefore, the young woman is not aware of being seen or watched by the man
with the opera glasses and has no cause to feel caught off guard.

24

According to Passage 1, why did Cassatt seemingly want viewers to keep a


respectful distance (lines 15-16) from the women in her paintings?
(A)

Cassatt thought her paintings could be appreciated best from a distance.

(B)

Cassatt felt it was inappropriate for strangers to be too close to a woman.

(C) Cassatt usually preferred not to paint close-ups of women.

(D) Cassatt did not want the female models she used to be recognized.

file://E:\\d7.htm

2006-11-12

23/23

The Official SAT Online Course

(E)

Cassatt wanted to emphasize that womens activities were important.

ANSWERS

AND EXPLANATIONS

Explanation for Correct Answer E :


Choice (E) is correct. From the frequency and perspective with which Cassatt
painted women involved in their activities, the reader can infer that Cassatt
considered womens activities important. As the author of Passage 1 states, it is
striking how frequently Cassatt depicted women on their own. The perspective
from which Cassatt painted women involved in activities reveals the artists
attitude toward those activities. By having her subjects turn their backs to the
viewer or incline their heads, Cassatt emphasizes the activities rather than the
physical features of the women. Also, if a subjects face was visible, it expressed a
deep concentration on the activity. The author of Passage 1 suggests that
Cassatts paintings give the sense that the viewers should not interrupt the
activities in which the women are involved or cross the boundaries of their
privacy. In other words, the viewers should keep a respectful distance.

Explanation for Incorrect Answer A :


Choice (A) is incorrect. The respectful distance refers to the distance of the
viewer from the women in the paintings, not the distance of the viewer from the
paintings. Earlier in the passage, the author notes that the poses of the women in
the paintings define the boundaries of their privacy, boundaries that the
viewers get a sense they should not cross.

re
d

Explanation for Incorrect Answer B :


Choice (B) is incorrect. Nothing in the passage indicates that Cassatt thought it was
inappropriate for strangers to be too close to a woman. Rather, Cassatt was
concerned about the women being interrupted or distracted from their activities.
From the poses of Cassatts subjects, the reader can infer that neither the artist
nor the viewers should interrupt the activities in which the women are involved or
cross the boundaries of their privacy.

te

is

Explanation for Incorrect Answer C :


Choice (C) is incorrect. Nothing in the passage indicates that Cassatt painted her
subject from a respectful distance because she preferred not to paint closeups. Rather, the artist was concerned about the women being interrupted or
distracted from their activities. The poses of and distance from her subjects imply
that neither the artist nor the viewers should interrupt the activities in which the
women are involved or cross the boundaries of their privacy.

eg

nR

Explanation for Incorrect Answer D :


Choice (D) is incorrect. Nothing in the passage implies that Cassatt was trying to
protect the identities of her subjects. Instead, the artist was concerned about the
women being interrupted or distracted from their activities. The poses of and
distance from her subjects imply that neither the artist nor the viewers should
interrupt the activities in which the women are involved or cross the boundaries
of their privacy.

Back to Score Report

Copyright 2006 The College Board. All rights reserved.

file://E:\\d7.htm

Privacy Policy

Terms of Use

Contact Us

2006-11-12

1/18

The Official SAT Online Course

Help | Profile | My Organizer | My Bookmarks | Logout

Answers and Explanations

Back to Score Report

Test Sections

Section 1

View Answers and Explanations

Section 2

Online - Practice Test #4

Section 5

Section 6

Angered that the book arrived in the mail in such a shabby condition, Elliot insisted
that the bookseller replace it with ------- copy.
(A) an imitation

Section 7

(B)

Section 8

an authentic

Section 9

(C) a pristine

Section 10

(D) a generic

(E)

a shopworn

ANSWERS

AND EXPLANATIONS

ed

Section 4

Explanation for Correct Answer C :


Choice (C) is correct. Pristine means fresh and clean. If one were to insert this
term into the text, the sentence would read Angered that the book arrived in the
mail in such a shabby condition, Elliot insisted that the bookseller replace it with a
pristine copy. Elliot was angered that the book was shabby, or ill-kept and faded
from wear, so he would most likely want the bookseller to replace it with a copy
that is pristine, or fresh and clean.

er

is
t

eg

Explanation for Incorrect Answer A :


Choice (A) is incorrect. Imitation means counterfeit or produced as a copy. If
one were to insert this term into the text, the sentence would read Angered that
the book arrived in the mail in such a shabby condition, Elliot insisted that the
bookseller replace it with an imitation copy. It does not make sense to say that
Elliot wanted an imitation, or counterfeit, copy of the book.

nR

Explanation for Incorrect Answer B :


Choice (B) is incorrect. Authentic means genuine and not an imitation. If one
were to insert this term into the text, the sentence would read Angered that the
book arrived in the mail in such a shabby condition, Elliot insisted that the
bookseller replace it with an authentic copy. Although the book arrived in a
shabby condition, nothing in the sentence indicates that the copy that Elliot initially
received was not authentic.

Explanation for Incorrect Answer D :


Choice (D) is incorrect. Generic means having no particularly distinctive
qualities. If one were to insert this term into the text, the sentence would read
Angered that the book arrived in the mail in such a shabby condition, Elliot
insisted that the bookseller replace it with a generic copy. It is reasonable to
assume that Elliot wants a replacement copy of the specific book he received in the
mail. A specific book, by definition, has distinctive qualities. Therefore, the term
generic is not logical in this context.

Explanation for Incorrect Answer E :


Choice (E) is incorrect. Shopworn means faded or otherwise worn-out from
being in a store too long. If one were to insert this term into the text, the sentence
would read Angered that the book arrived in the mail in such a shabby condition,
Elliot insisted that the bookseller replace it with a shopworn copy. Elliot was
angered that the book he initially received was shabby, or ill-kept and faded from
wear, so it is illogical to suggest that he would want the bookseller to replace it with
a copy that was also faded and worn-out.

file://E:\\d8.htm

Staring at the abstract painting, Edna could not understand how so many of her

2006-11-12

2/18

The Official SAT Online Course

fellow museumgoers could pretend to ------- its meaning, when to her its message
was so opaque.
(A) obscure
(B)

advertise

(C) comprehend

(D) disparage

(E)

ignore

ANSWERS

AND EXPLANATIONS

Explanation for Correct Answer C :


Choice (C) is correct. To comprehend means to grasp the meaning of
something. If one were to insert this term into the text, the sentence would read
Staring at the abstract painting, Edna could not understand how so many of her
fellow museumgoers could pretend to comprehend its meaning, when to her its
message was so opaque. If Edna found the message of the painting to be
opaque, or hard to understand, it makes sense to suggest that she did not
understand how the other museumgoers seemed to comprehend, or grasp the
meaning of, the painting.

re

Explanation for Incorrect Answer A :


Choice (A) is incorrect. To obscure means to conceal or to hide. If one were to
insert this term into the text, the sentence would read Staring at the abstract
painting, Edna could not understand how so many of her fellow museumgoers could
pretend to obscure its meaning, when to her its message was so opaque.
Although the other musemgoers could have obscured the actual painting, it is
illogical to suggest that they could obscure, or hide, the paintings meaning.

te

Explanation for Incorrect Answer B :


Choice (B) is incorrect. To advertise means to make something known. If one
were to insert this term into the text, the sentence would read Staring at the
abstract painting, Edna could not understand how so many of her fellow
museumgoers could pretend to advertise its meaning, when to her its message was
so opaque. It is reasonable to suggest that Ednas fellow museumgoers could
advertise, or make known, their interpretations of the abstract paintings
message, but it does not make sense in this context to say that the other
museumgoers could pretend to make the paintings message known.

eg

is

nR

Explanation for Incorrect Answer D :


Choice (D) is incorrect. To disparage means to treat with disdain. If one were
to insert this term into the text, the sentence would read Staring at the abstract
painting, Edna could not understand how so many of her fellow museumgoers could
pretend to disparage its meaning, when to her its message was so opaque. There
is nothing in the sentence to suggest that Ednas fellow museumgoers disparaged,
or treated with disdain, the meaning of the abstract painting.

Explanation for Incorrect Answer E :


Choice (E) is incorrect. To ignore means to refuse to take notice of something.
If one were to insert this term into the text, the sentence would read Staring at
the abstract painting, Edna could not understand how so many of her fellow
museumgoers could pretend to ignore its meaning, when to her its message was so
opaque. If Edna thought that the paintings meaning was opaque or not
easily comprehended, she would not likely believe that her fellow museumgoers
were pretending to "ignore" its meaning, since ignoring the meaning would imply
that it had been understood in the first place.

Mrs. Rodriguez was under the impression that discipline would ------- her wayward
students academic progress rather than advance it.
(A)

condone

(B)

foster

(C) quicken

(D) exalt

(E)

file://E:\\d8.htm

hamper

2006-11-12

3/18

The Official SAT Online Course

ANSWERS

AND EXPLANATIONS

Explanation for Correct Answer E :


Choice (E) is correct. To hamper means to restrict. If one were to insert this
term into the text, the sentence would read Mrs. Rodriguez was under the
impression that discipline would hamper her wayward students academic
progress rather than advance it. The phrase rather than indicates that the
missing term will contrast with the term advance. Something that hampers, or
restricts, the students progress cannot also be said to advance it.

Explanation for Incorrect Answer A :


Choice (A) is incorrect. To condone means to forgive or overlook a behavior. If
one were to insert this term into the text, the sentence would read Mrs.
Rodriguez was under the impression that discipline would condone her wayward
students academic progress rather than advance it. The phrase rather
than suggests that the meaning of the missing term will contrast with the term
advance. There is no inherent contrast between condoning and advancing the
students academic progress. Additionally, it is illogical to suggest that discipline
could condone, or excuse, a students progress.

Explanation for Incorrect Answer B :


Choice (B) is incorrect. To foster means to encourage or nurture. If one were
to insert this term into the text, the sentence would read Mrs. Rodriguez was
under the impression that discipline would foster her wayward students academic
progress rather than advance it. The phrase rather than indicates that the
missing term will contrast with the term advance. There is no contrast between
fostering and advancing the students academic progress. The terms foster
and advance both refer to helping the student to continue making progress.

ed

te
r

Explanation for Incorrect Answer C :


Choice (C) is incorrect. To quicken means to accelerate or make faster. If one
were to insert this term into the text, the sentence would read Mrs. Rodriguez
was under the impression that discipline would quicken her wayward students
academic progress rather than advance it. The phrase rather than indicates
that the missing term will contrast with the term advance. There is no contrast
between quickening and advancing the students academic progress. The terms
quicken and advance both refer to helping the student to continue making
progress.

eg
is

nR

Explanation for Incorrect Answer D :


Choice (D) is incorrect. To exalt means to enhance or raise. If one were to
insert this term into the text, the sentence would read Mrs. Rodriguez was under
the impression that discipline would exalt her wayward students academic
progress rather than advance it. The phrase rather than indicates that the
missing term will contrast with the term advance. There is no inherent contrast
between exalting and advancing the students academic progress. The terms
exalt and advance can both be said to refer to helping the student make
progress.

Despite the directors lifelong reputation for humility, the retirement celebration
found him unable to ------- the urge to ------- his successes.
(A)

quell . . downplay

(B)

resist . . catalog

(C) embrace . . embellish

(D) forego . . diminish

(E)

modify . . belittle

ANSWERS

AND EXPLANATIONS

Explanation for Correct Answer B :


Choice (B) is correct. To resist means to withstand the force or effect of
something. To catalog means to list. If one were to insert these terms into the
text, the sentence would read Despite the directors lifelong reputation for
humility, the retirement celebration found him unable to resist the urge to catalog
his successes. The term Despite indicates that the directors behavior at
the retirement celebration contrasted with the humble behavior for which he was

file://E:\\d8.htm

2006-11-12

4/18

The Official SAT Online Course

known. The directors inability to resist the urge to list his successes contrasts
with his reputation for humility, or humble behavior.

Explanation for Incorrect Answer A :


Choice (A) is incorrect. To quell means to pacify or reduce to submission. To
downplay means to de-emphasize or attach little importance to something. If one
were to insert these terms into the text, the sentence would read Despite the
directors lifelong reputation for humility, the retirement celebration found him
unable to quell the urge to downplay his successes. The term Despite
indicates that the directors behavior at the retirement celebration contrasted with
his earlier reputation for humility or humble behavior. If the director was unable to
quell the urge to downplay his successes, he most likely de-emphasized
his successes. This behavior does not properly contrast with his earlier reputation
for humble behavior.

Explanation for Incorrect Answer C :


Choice (C) is incorrect. To embrace means to welcome. To embellish
means to enhance. If one were to insert these terms into the text, the sentence
would read Despite the directors lifelong reputation for humility, the retirement
celebration found him unable to embrace the urge to embellish his successes. The
term Despite indicates that the directors behavior at the retirement
celebration contrasted with his earlier reputation for humility. If the director was
unable to embrace, or welcome, the urge to embellish his successes, he
most likely did not feel comfortable enhancing his successes. This behavior does not
properly contrast with his earlier reputation for humble behavior.

re

Explanation for Incorrect Answer D :


Choice (D) is incorrect. To forego means to do without or to give up. To
diminish means to belittle or to lessen the dignity or reputation of something. If
one were to insert these terms into the text, the sentence would read Despite the
directors lifelong reputation for humility, the retirement celebration found him
unable to forego the urge to diminish his successes. The term Despite
indicates that the directors behavior at the retirement celebration contrasted with
his earlier reputation for humility. If the director was unable to give up the urge to
diminish his successes, he most likely belittled his successes. This behavior
does not contrast with his earlier reputation for humble behavior.

te

is

eg

Explanation for Incorrect Answer E :


Choice (E) is incorrect. To modify means to make less extreme. To belittle
means to cause someone or something to seem little or less. If one were to insert
these terms into the text, the sentence would read Despite the directors
lifelong reputation for humility, the retirement celebration found him unable to
modify the urge to belittle his successes. The term Despite indicates that the
directors behavior at the retirement celebration contrasted with his earlier
reputation for humility. If the director was unable to lessen the urge to belittle
his successes, he most likely made his successes seem small. This behavior does
not contrast with his reputation for humble behavior.

nR

U
5

Despite his ------- rich food, the chef was able to practice ------- when his diet
became threatening to his health.
(A)

penchant for . . austerity

(B)

fondness for . . indulgence

(C) avoidance of . . luxury

(D) indifference to . . asceticism

(E)

talent for . . virtuosity

ANSWERS

AND EXPLANATIONS

Explanation for Correct Answer A :


Choice (A) is correct. A penchant is a strong liking. Austerity is strict
behavior. If one were to insert these terms into the text, the sentence would read
Despite his penchant for rich food, the chef was able to practice austerity when
his diet became threatening to his health. It is reasonable to assume that the
chefs penchant for rich food was at least somewhat responsible for his diet
becoming threatening to his health. Even though he had a strong liking for rich
food, the chef had to practice austerity, or strict behavior, when following a
healthier diet.

file://E:\\d8.htm

2006-11-12

5/18

The Official SAT Online Course

Explanation for Incorrect Answer B :


Choice (B) is incorrect. Fondness means a liking or affection. Indulgence is
the act of indulging or taking pleasure in something. If one were to insert these
terms into the text, the sentence would read Despite his fondness for rich food,
the chef was able to practice indulgence when his diet became threatening to his
health. It is illogical to suggest that indulgence would improve the chefs
diet if his fondness for, or liking of, rich food had become threatening to his
health. On the contrary, the chef would most likely have to avoid indulging in rich
food when following a healthier diet.

Explanation for Incorrect Answer C :


Choice (C) is incorrect. Avoidance means avoiding or withdrawing from
something. Luxury pertains to sumptuous living or surroundings. If one were to
insert these terms into the text, the sentence would read Despite his avoidance of
rich food, the chef was able to practice luxury when his diet became threatening to
his health. Avoiding rich food would not necessarily cause the chefs diet to
become threatening to his health, because avoiding rich food is generally a healthy
thing to do. In addition, it is illogical to suggest that indulging in "luxury" could
make the chefs diet healthier.

Explanation for Incorrect Answer D :


Choice (D) is incorrect. Indifference means having neither a strong liking nor a
strong disliking for something. Asceticism is strict self-denial. If one were to
insert these terms into the text, the sentence would read Despite his indifference
to rich food, the chef was able to practice asceticism when his diet became
threatening to his health. The word Despite indicates that the two parts of
the sentence express contradictory ideas. The chefs asceticism, or strict
denial, does not inherently contrast with the chefs indifference to rich food. If
the chef has no strong liking of rich food, he would probably not have to be strict
with himself in avoiding rich food when following a healthier diet.

ed

er

is
t

Explanation for Incorrect Answer E :


Choice (E) is incorrect. Talent means a special ability. Virtuosity is a great
technical skill. If one were to insert these terms into the text, the sentence would
read Despite his talent for rich food, the chef was able to practice virtuosity when
his diet became threatening to his health. The word Despite indicates that
the two parts of the sentence express contradictory ideas. While it is reasonable to
say that a chef has a special ability to create rich food, this ability would not
necessarily contrast with his displaying "virtuosity" when trying to make his diet
more healthy; on the contrary, a master chef might have to downplay his techincal
skill in cooking in order to follow a healthier diet.

nR
eg

Rauls game-winning goal in the closing minutes of the game ------- his well-known
ability to excel under stressful conditions.

(A)

clarified

(B)

solidified

(C) identified

(D) epitomized

(E)

decried

ANSWERS

AND EXPLANATIONS

Explanation for Correct Answer D :


Choice (D) is correct. To epitomize means to be a typical example of
something. If one were to insert this term into the text, the sentence would read
Rauls game-winning goal in the closing minutes of the game epitomized his
well-known ability to excel under stressful conditions. The winning goal he made
at the end of the game epitomized, or was a typical example of, Raul's ability
to perform under stress.

Explanation for Incorrect Answer A :


Choice (A) is incorrect. To clarify means to make something understandable or
free of confusion. If one were to insert this term into the text, the sentence would

file://E:\\d8.htm

2006-11-12

6/18

The Official SAT Online Course

read Rauls game-winning goal in the closing minutes of the game clarified his
well-known ability to excel under stressful conditions. Rauls ability to excel
under stressful conditions was well-known, so it does not make sense to say that his
ability required clarification or was difficult to understand.

Explanation for Incorrect Answer B :


Choice (B) is incorrect. To solidify means to strengthen or make solid. If one
were to insert this term into the text, the sentence would read Rauls gamewinning goal in the closing minutes of the game solidified his well-known ability to
excel under stressful conditions. Although it might make sense to say that Raul
s game-winning goal could solidify, or strengthen, his reputation for excelling under
stressful conditions, it is illogical to suggest that the game-winning goal
solidified Rauls actual ability.

Explanation for Incorrect Answer C :


Choice (C) is incorrect. To identify means to establish the identity or
characteristics of something or someone. If one were to insert this term into the
text, the sentence would read Rauls game-winning goal in the closing minutes
of the game identified his well-known ability to excel under stressful conditions.
Rauls ability to excel under stressful conditions was already well-known, so it is
illogical to suggest that his game-winning goal identified, or established the
characteristics of, his ability.

Explanation for Incorrect Answer E :


Choice (E) is incorrect. To decry means to express strong disapproval of
something. If one were to insert this term into the text, the sentence would read
Rauls game-winning goal in the closing minutes of the game decried his wellknown ability to excel under stressful conditions. It is illogical to suggest that
Rauls game-winning goal could have decried, or expressed disapproval of, his
ability to excel under stressful conditions.

er
ed

Grammatical errors like bleeded


and singed have long epitomized the
innocence and freshness of childrens
Line minds. The errors are acts of creation in
5
which children lift a pattern from their
brief experience and apply it with
impeccable logic to new words, unaware
that the adult world treats them as
arbitrary exceptions. In A Dark-Adapted
10 Eye, the novelist Barbara Vine introduces
an unlikable child by remarking, He
would refer to adults instead of
grownups and get all his past tenses
right, never saying rided for rode or
15 eated for ate.
Childrens errors with irregular
verbs also have been prominent in
debates on the nature of language and
mind. The neurologist Eric Lenneberg
20 pointed to the errors when he and linguist
Noam Chomsky first argued that language
was innate. Psychology textbooks cite the
errors to rhapsodize that children are
lovers of cognitive tidiness and simplicity;
25 researchers who study learning in adults
cite the errors as a paradigm case of the
human habit of overgeneralizing rules to
exceptional cases.
Nothing is more important to the

is
t

eg

nR

file://E:\\d8.htm

2006-11-12

7/18

The Official SAT Online Course

30 theory of words and rules than an

40

45

er

50

ed

35

explanation of how children acquire rules


and apply themindeed overapply them
to words. The simplicity of these errors is
deceptive. It is not easy to explain why
children start making them, and its
harder to explain why they stop.
Overgeneralization errors are a
symptom of the open-ended productivity
of language, which children indulge in as
soon as they begin to put words together.
At around eighteen months children start
to utter two word microsentences like
See baby and More cereal. Some are
simply telegraphic renditions of their
parents speech, but many are original
productions. More outside! says a tot
who wants to play in the park. Allgone
sticky! says another after his mother has
washed jam off his fingers. By two years
of age, children produce longer and more
complicated sentences, and begin to
supply grammatical morphemes such as
-ing, -ed, -s and the auxiliaries.
Sometime between the end of the second
year and the end of the third year,
children begin to overgeneralize -ed to
irregular verbs.
Jennifer Ganger and I suspected
that at least some of the timing of
language development, including the pasttense rule, is controlled by a maturational
clock. Children may begin to acquire a
rule at a certain age for the same reason
they grow hair or teeth at certain ages. If
the clock is partly under the control of
genes, then identical twins should develop
language in tighter synchrony than
fraternal twins, who share only half their
genes. We have enlisted the help of
hundreds of mothers of twins who send us
daily lists of their childrens new words
and word combinations. The checklists
show that vocabulary growth, the first
word combinations, and the rate of
making past-tense errors are all in tighter
lockstep in identical twins than in fraternal
twins. The results tell us that at least
some of the mental events that make a
child say singed are hereditary. The very
first past-tense error, though, is not.

is
t

55

eg

nR

65

70

75

80

file://E:\\d8.htm

60

2006-11-12

8/18

The Official SAT Online Course

When one twin makes an error like singed


for the first time, an identical twin is no
quicker to follow suit than a fraternal twin.
These gapsan average of 34 days
85 between the first past-tense errors made
by two children with the same genes
exposed to the same speechare a
reminder of the importance of sheer
chance in childrens development.
90
Childrens speech errors, which
make such engaging anecdotes in poetry,
novels, television features and Web sites
for parents, may help us untangle one of
the thickest knots in science, nature and
95 nurture. When a child says It bleeded
and It singed, the fingerprints of
learning are all over the sentence. Every
bit of every word has been learned,
including the past tense suffix -ed. The
100 very existence of the error comes from a
process of learning that is still incomplete:
mastery of the irregular bled and sang.
But learning is impossible without
innately organized circuitry to do the
105 learning, and these errors give us hints of
how it works. Children are born to attend
to minor differences in the pronunciation
of words, such as walk and walked. They
seek a systematic basis for the difference
110 in the meaning or form of the sentence,
rather than dismissing it as haphazard
variation in speech styles. They
dichotomize time into past and nonpast,
and correlate half the timeline with the
115 evanescent word ending. They must have
a built-in tendency to block the rule when
a competing form (like bled) is found in
memory, because there is no way they
could learn the blocking principle in the
120 absence of usable feedback from their
parents. Their use of the rule (though
perhaps not the moment when they first
use it) is partly guided by their genes.

re

te

is

eg

nR

The word lift in line 5 most closely means


(A)

raise

(B)

elevate

(C) make better

(D) boost up

(E)

file://E:\\d8.htm

take

2006-11-12

9/18

The Official SAT Online Course

ANSWERS

AND EXPLANATIONS

Explanation for Correct Answer E :


Choice (E) is correct. The word lift in line 5 most closely means take. The
key to this meaning is the direct object of liftpattern. The author is stating
that children take something stored in their memory or past experience, in this
case, a pattern or rule, to deal with a new experience, specifically, new words.

Explanation for Incorrect Answer A :


Choice (A) is incorrect. Although lift can certainly mean raise, that is not
its meaning here. The word lift in line 5 most closely means take. The key
to this meaning is the direct object of liftpattern. The children are not
raising a pattern or rule.

Explanation for Incorrect Answer B :


Choice (B) is incorrect. Although lift can certainly mean elevate, that is not
its meaning here. The word lift in line 5 most closely means take. The key
to this meaning is the direct object of liftpattern. The children are not
elevating a pattern or rule.

ed

Explanation for Incorrect Answer C :


Choice (C) is incorrect. Although lift can certainly mean make better, that
is not its meaning here. The word lift in line 5 most closely means take.
The key to this meaning is the direct object of liftpattern. The children are
not making a pattern or rule better.

te
r

Explanation for Incorrect Answer D :


Choice (D) is incorrect. Although lift can certainly mean boost up (as in
lift ones spirits), that is not its meaning here. The word lift in line 5
most closely means take. The key to this meaning is the direct object of
liftpattern. The children are not boosting up a pattern or rule.

is

eg

The author suggests that the unlikable child in A Dark-Adapted Eye


(A)

did not enjoy speaking with grownups

(B)

spoke like most other children

nR

(C) never wanted to be introduced to other children

(D) did not speak with childlike errors

(E)

was always making up new confusing words

ANSWERS

AND EXPLANATIONS

Explanation for Correct Answer D :


Choice (D) is correct. The unlikable child in A Dark-Adapted Eye did not speak with
the childlike errors that most children make. Instead of incorrectly
adding ed to create the past tense of irregular verbs, such as bleeded and singed,
as most children do, the child in the novel got all his past tenses right.

Explanation for Incorrect Answer A :


Choice (A) is incorrect. The child in A Dark-Adapted Eye probably did enjoy
speaking with adults because he spoke like an adult himself. In fact, the child in the
novel did not speak with the childlike errors that most children make. Instead of
incorrectly adding ed to create the past tense of irregular verbs, such as bleeded
and singed, as most children do, the child in the novel got all his past tenses
right.

Explanation for Incorrect Answer B :


Choice (B) is incorrect. The child in A Dark-Adapted Eye did not speak like most
other children. In fact, the child in the novel did not speak with the childlike errors
that most children make. Instead of incorrectly adding ed to create the past
tense of irregular verbs, such as bleeded and singed, as most children do, the child
in the novel got all his past tenses right.

file://E:\\d8.htm

2006-11-12

10/18

The Official SAT Online Course

Explanation for Incorrect Answer C :


Choice (C) is incorrect. Nothing in the description of the child in A Dark-Adapted Eye
indicates that he did or did not want to meet other children. The description focuses
on the childs language. The child in the novel did not speak with the childlike
errors that most children make. Instead of incorrectly adding ed to create the
past tense of irregular verbs, such as bleeded and singed, as most children do, the
child in the novel got all his past tenses right.

Explanation for Incorrect Answer E :


Choice (E) is incorrect. Although the description of the unlikable child in A DarkAdapted Eye focuses on his language, the child uses adult language, not confusing,
made-up words. In fact, the child in the novel did not speak with the childlike errors
that most children make. Instead of incorrectly
adding ed to create the past tense of irregular verbs, such as bleeded and singed,
as most children do, the child in the novel got all his past tenses right.

The primary purpose of the second paragraph is to


(A)

indicate the extent of scholarly disagreement about the nature of


childrens speech errors

(B)

demonstrate the universal occurrence of childrens speech errors

(C) list the scholars who have influenced academic understanding of children
s speech errors

(D) endorse a particular explanation for childrens speech errors

show that childrens speech errors have received widespread scholarly


attention

AND EXPLANATIONS

te

ANSWERS

re

(E)

Explanation for Correct Answer E :


Choice (E) is correct. The examples in paragraph 2 show that childrens speech
errors have received scholarly attention. The first example explains that two
scholars, one a neurologist (Lenneberg) and one a linguist (Chomsky), debated the
development of language in children. The second example explains how
psychologists and other scientists have used childrens speech errors in their work
and research.

eg

is

nR

Explanation for Incorrect Answer A :


Choice (A) is incorrect. Paragraph 2 does not provide specific arguments on the
nature of childrens speech errors or indicate the extent of such a debate.
Instead, the examples in this paragraph show that childrens speech errors have
received scholarly attention, but only as part of the research on language
development.

Explanation for Incorrect Answer B :


Choice (B) is incorrect. Although the universality of speech errors by Englishspeaking children might be inferred from the entire passage, paragraph 2 does not
address this issue. Instead, the examples in this paragraph show that childrens
speech errors have received scholarly attention.

Explanation for Incorrect Answer C :


Choice (C) is incorrect. Only two scholars are named in paragraph 2, and their
influence on academic understanding of language development is not discussed.
Instead, the examples in this paragraph show that childrens speech errors have
received scholarly attention.

Explanation for Incorrect Answer D :


Choice (D) is incorrect. As the entire passage shows, a particular explanation for
childrens speech errors is still being studied. The author explicitly states, It is
not easy to explain why children start making them [errors], and its harder to
explain why they stop. Instead, the examples in this paragraph show that
childrens speech errors have received scholarly attention.

10

The word tidiness in line 24 is closest in meaning to

cleanliness

file://E:\\d8.htm

2006-11-12

11/18

The Official SAT Online Course

(A)
(B)

neatness

(C) orderliness

(D) quickness

(E)

snugness

ANSWERS

AND EXPLANATIONS

Explanation for Correct Answer C :


Choice (C) is correct. From its specific context and from other statements in the
passage, the word tidiness most closely means orderliness. The author is
discussing childrens cognitive, or mental, workings, that is, how children
think. Other statements indicate how children think: they use impeccable logic
and "overgeneralization" to create new words. That is, children impose order or
orderliness on language in their attempt to learn and use that language.

Explanation for Incorrect Answer A :


Choice (A) is incorrect. From its specific context and from other statements in the
passage, the word tidiness most closely means orderliness, not
cleanliness. The author is discussing childrens cognitive, or mental,
workings, that is, how children think.

ed

Explanation for Incorrect Answer B :


Choice (B) is incorrect. Although tidiness can mean neatness in the sense
of uncluttered, its meaning in this sentence is closest to orderliness. The
author is discussing childrens cognitive, or mental, workings, that is, how
children think.

eg
is
te
r

Explanation for Incorrect Answer D :


Choice (D) is incorrect. From its specific context and from other statements in the
passage, the word tidiness most closely means orderliness, not
quickness. The author is discussing childrens cognitive, or mental,
workings, that is, how children think. Although children might be mentally quick,
other statements indicate that the meaning here is orderliness. Children use
impeccable logic and "overgeneralization" to create new words. That is,
children impose order or orderliness on language in their attempt to learn and use
that language.

nR

Explanation for Incorrect Answer E :


Choice (E) is incorrect. From its specific context and from other statements in the
passage, the word tidiness most closely means orderliness, not
snugness. The author is discussing childrens cognitive, or mental,
workings, that is, how children think.

11

The word overapply in line 32 refers to the


(A)

considerable effort children give to speaking

(B)

misunderstanding scholars have of simple speech errors

(C) faulty application of scholarly theories to childrens speech

(D) strict adherence of children to certain linguistic rules

(E)

misguided training that children often receive from parents

ANSWERS

AND EXPLANATIONS

Explanation for Correct Answer D :


Choice (D) is correct. The word overapply in line 32 refers to childrens strict
adherence to language rules. In this clause, the subject of the verb overapply
is children, and the added predicateindeed overapply thememphasizes
the authors point that children lift a pattern from past experience and
generalize the ed past-tense rule to all verbs. By generalizing the rule, children
overapply it.

file://E:\\d8.htm

2006-11-12

12/18

The Official SAT Online Course

Explanation for Incorrect Answer A :


Choice (A) is incorrect. The word overapply used in line 32 does not refer to
childrens efforts in trying to speak but to their adherence to language rules.
Childrens speech is the overall subject of the passage, but children
speaking is not the subject of this sentence.

Explanation for Incorrect Answer B :


Choice (B) is incorrect. Scholars might not fully understand childrens speech
errors, but not because of misunderstanding. In any case, the word overapply
does not refer to scholars but to childrens adherence to language rules.

Explanation for Incorrect Answer C :


Choice (C) is incorrect. The point of this sentence is not the application (faulty or
otherwise) of theories to childrens speech but the use of childrens speech in
explaining the theories. The word overapply refers to childrens strict
adherence to language rules. In this clause, the subject of the verb overapply
is children, and the added predicateindeed overapply thememphasizes
the authors point that children lift a pattern from past experience and
generalize the ed past-tense rule to all verbs. By generalizing, or expanding, the
rule, children overapply the rule.

Explanation for Incorrect Answer E :


Choice (E) is incorrect. Neither the passage nor this sentence mentions parents
training of their children. The word overapply refers to childrens strict
adherence to language rules.

re

12

is
te

The phrase open-ended productivity of language in lines 38-39 refers to


(A)

the lack of rules governing language use

(B)

the vulnerability of language to errors

(C) the creative quality inherent in language use

nR
eg

(D) the variety of different existing languages

(E)

the effort involved in using language correctly

ANSWERS

AND EXPLANATIONS

Explanation for Correct Answer C :


Choice (C) is correct. The phrase open-ended productivity of language refers to
the creativity that language allows. The adjective open-ended here means
unlimited or without an end. The author means that words can be put together in
an unlimited number of ways and provides specific examples of childrens creative
uses of language, or original productions: More outside and Allgone
sticky.

Explanation for Incorrect Answer A :


Choice (A) is incorrect. The phrase open-ended productivity of language does
not refer to a lack of language rules. In fact, the focus of the entire passage is
about language rules and childrens misapplication of those rules. Instead, the
phrase refers to the creativity that language allows.

Explanation for Incorrect Answer B :


Choice (B) is incorrect. Indeed, language is vulnerable to errors, but children make
speech errors, not because of this vulnerability, but because of the open-ended
quality of language. The phrase open-ended productivity of language refers to
the creativity that language allows.

Explanation for Incorrect Answer D :


Choice (D) is incorrect. The phrase open-ended productivity of language does
not refer here to the variety of different languages in the world but to the variety or
creativity that language allows.

Explanation for Incorrect Answer E :


Choice (E) is incorrect. The phrase open-ended productivity of language refers
here not to the correct use but to the creative use of language.

file://E:\\d8.htm

2006-11-12

13/18

The Official SAT Online Course

13

More outside and Allgone sticky (lines 46 and 47-48) are examples of
(A) overgeneralization errors frequently made by children
(B)

exceptional instances of childrens language use

(C) childrens attempts to communicate by thinking rather than mimicking

(D) speech used by parents to communicate with their children

(E)

sentences displaying childrens use of grammatical morphemes

ANSWERS

AND EXPLANATIONS

Explanation for Correct Answer C :


Choice (C) is correct. The phrases more outside and allgone sticky are
examples of childrens attempts to communicate by thinking rather than
mimicking. Unlike the phrases see baby and more cereal, which children
copy or mimic from their parents (telegraphic renditions of their parents
speech), these phrases are original productions, meaning that they are
creations of the childrens own minds.

re

Explanation for Incorrect Answer A :


Choice (A) is incorrect. As the author has explained, overgeneralization errors are
errors resulting from the application of a language rule. The phrases more
outside and allgone sticky are not examples of overgeneralization of
language rules but are examples of childrens creative use of language to
communicate.

te

eg
is

Explanation for Incorrect Answer B :


Choice (B) is incorrect. The phrases more outside and allgone sticky are
not examples of exceptional, or rare or out-of-the-ordinary, use of language by
children. In fact, they are examples of the creative use of language that most
children show at a certain stage of language development.

Explanation for Incorrect Answer D :


Choice (D) is incorrect. The phrases more outside and allgone sticky are
not phrases that parents use to communicate with their children but are phrases
that children have used to communicate with their parents.

nR

Explanation for Incorrect Answer E :


Choice (E) is incorrect. The phrases more outside and allgone sticky are
not grammatically correct and do not show childrens grasp of grammar. Instead,
these phrases are examples of childrens attempts to communicate by thinking
rather than mimicking.

14

By fingerprints of learning (lines 96-97) the author primarily means


(A)

demonstration of sustained effort

(B)

indication of parental influence

(C) results of faulty thinking

(D) evidence of acquired information

(E)

illustration of genetic ability

ANSWERS

AND EXPLANATIONS

Explanation for Correct Answer D :


Choice (D) is correct. With the phrase fingerprints of learning, the author is
using figurative language to explain that a childs use of ed to indicate past
tense is based on learned, or acquired, information, not innate knowledge. The
author explains this meaning in the next sentence: Every bit of every word has
been learned, including the past tense suffix ed.

file://E:\\d8.htm

2006-11-12

14/18

The Official SAT Online Course

Explanation for Incorrect Answer A :


Choice (A) is incorrect. The phrase fingerprints of learning does not mean the
child has demonstrated a sustained effort. Instead, the author is using figurative
language to explain that a childs use of ed to indicate past tense is based on
learned, or acquired, information, not innate knowledge.

Explanation for Incorrect Answer B :


Choice (B) is incorrect. The phrase fingerprints of learning is not referring to
parents fingerprints or to the literal fingerprints of anyone else. With the phrase
fingerprints of learning, the author is using figurative language to explain that
a childs use of ed to indicate past tense is based on learned, or acquired,
information, not innate knowledge.

Explanation for Incorrect Answer C :


Choice (C) is incorrect. The phrase fingerprints of learning does refer to
thinking, but not to faulty thinking (in fact, children use impeccable logic in
applying the ed rule). With the phrase fingerprints of learning, the author is
using figurative language to explain that a childs use of ed to indicate past
tense is based on learned, or acquired, information, not innate knowledge.

Explanation for Incorrect Answer E :


Choice (E) is incorrect. The phrase fingerprints of learning has the opposite
meaning of genetic or innate ability. With the phrase fingerprints of learning,
the author is using figurative language to explain that a childs use of ed to
indicate past tense is based on learned, or acquired, information, not innate
knowledge.

er
e

15

In paragraph 6 (lines 90 to 102), the author suggests that childrens speech errors
(A)

are overused as examples in literature and art

(B)

have important scientific implications

is
t

(C) can be easily unlearned

(E)

eg

(D) indicate problems with linguistic rules

are solely determined by genetics

AND EXPLANATIONS

nR

ANSWERS

Explanation for Correct Answer B :


Choice (B) is correct. In paragraph 6, the author suggests that childrens speech
errors have important implications for understanding one of the thickest knots in
science, nature and nurture. In other words, as the author discusses in both
paragraphs 5 and 7, childrens language development is partly innate (nature)
and partly learned (nurture). Understanding the roles of nature and nurture in
childrens language development can help scientists understand those roles in
other areas of human development.

Explanation for Incorrect Answer A :


Choice (A) is incorrect. Childrens speech errors are used in literature and in other
ways, but the author does not say they are overused.

Explanation for Incorrect Answer C :


Choice (C) is incorrect. The author does mention in paragraph 3 that children
unlearn errors (its hard to explain why they stop making errors) but does
not discuss whether this unlearning is easy or hard and does not discuss the idea in
paragraph 6.

Explanation for Incorrect Answer D :


Choice (D) is incorrect. Although a child might have a problem in learning linguistic
rules, this point is not discussed in paragraph 6 or in the passage.

Explanation for Incorrect Answer E :


Choice (E) is incorrect. The author has made quite a different pointthat speech
errors are not determined solely by genetics. In fact, in paragraph 6, the author
suggests that childrens speech errors have important implications for
understanding one of the thickest knots in science, nature and nurture.

file://E:\\d8.htm

2006-11-12

15/18

The Official SAT Online Course

16

Innately organized circuitry in line 104 refers to


(A) ways of teaching language that all parents use
(B)

rules children learn to apply to language use

(C) differences in meaning suggested by complex sentences

(D) childrens natural ability to process distinctions in language use

(E)

combinations of words occurring in all languages

ANSWERS

AND EXPLANATIONS

Explanation for Correct Answer D :


Choice (D) is correct. The phrase innately organized circuitry refers to a child
s born ability to think about and impose order on language. The author explains
that this innate, or natural, ability enables the child to pay attention to minor
differences in pronunciation, to apply linguistic rules (seek a systematic basis
for the difference), and to divide time into past and nonpast. These are not
learned skills but are abilities with which the child is born.

ed

Explanation for Incorrect Answer A :


Choice (A) is incorrect. Although children do learn language from their parents (for
example, their first words are simply telegraphic renditions of their parents
speech), they are born with an ability to think about and impose order on the
language they are learning. The author calls this ability an innately organized
circuitry and explains that it enables the child to pay attention to minor
differences in pronunciation, to apply linguistic rules (seek a systematic basis
for the difference), and to divide time into past and nonpast. These are not
learned skills but are abilities with which the child is born.

er

is
t

eg

Explanation for Incorrect Answer B :


Choice (B) is incorrect. The innately organized circuitry is an ability that
children are born with, not something learned, such as language rules. The author
explains that this innate, or natural, ability enables the child to pay attention to
minor differences in pronunciation, to apply linguistic rules (seek a
systematic basis for the difference), and to divide time into past and
nonpast. These are not skills taught by parents but are abilities with which the
child is born.

nR

Explanation for Incorrect Answer C :


Choice (C) is incorrect. The phrase innately organized circuitry refers to a
childs born ability to think about and impose order on language, not to
differences in meaning of complex sentences. The author explains that this innate,
or natural, ability enables the child to pay attention to minor differences in
pronunciation, to apply linguistic rules (seek a systematic basis for the
difference), and to divide time into past and nonpast. These are abilities with
which the child is born.

Explanation for Incorrect Answer E :


Choice (E) is incorrect. Although the development of language occurs in all
languages, this phrase does not refer to words common to all languages. Instead,
the phrase innately organized circuitry refers to a childs born ability to think
about and impose order on language. The author explains that this innate, or
natural, ability enables the child to pay attention to minor differences in
pronunciation, to apply linguistic rules (seek a systematic basis for the
difference), and to divide time into past and nonpast. These are abilities with
which the child is born.

17

The phrase attend to in lines 106-107 most closely means


(A)

look after

(B)

notice

(C) activate

(D) be present to

associate with

file://E:\\d8.htm

2006-11-12

16/18

The Official SAT Online Course

(E)

ANSWERS

AND EXPLANATIONS

Explanation for Correct Answer B :


Choice (B) is correct. In context, the phrase attend to in lines 106107 most
nearly means notice. The author is explaining that children have an innate, or
natural, ability to notice differences in pronunciation of two very similar-sounding
words.

Explanation for Incorrect Answer A :


Choice (A) is incorrect. Although attend to can mean look after (as in
attend to a sick pet), that is not the meaning in this context. As used in lines
106107, attend to most nearly means notice. The author is explaining
that children have an innate, or natural, ability to notice differences in
pronunciation of two very similar-sounding words.

Explanation for Incorrect Answer C :


Choice (C) is incorrect. The phrase attend to does not mean activate (the
child does not activate minor differences). Instead, as used in lines 106107,
attend to most nearly means notice. The author is explaining that children
have an innate, or natural, ability to notice differences in pronunciation of two very
similar-sounding words.

er
e

Explanation for Incorrect Answer D :


Choice (D) is incorrect. Although attend can mean be present, that is not
the meaning in this context. As used in lines 106107, attend to most nearly
means notice. The author is explaining that children have an innate, or natural,
ability to notice differences in pronunciation of two very similar-sounding words.

is
t

Explanation for Incorrect Answer E :


Choice (E) is incorrect. The phrase attend to does not mean associate with
(the child does not associate with minor differences). Instead, as used in lines
106107, attend to most nearly means notice. The author is explaining
that children have an innate, or natural, ability to notice differences in
pronunciation of two very similar-sounding words.

eg

nR

18

Paragraph 7 (lines 103 to 123) describes

(A)

the way children combine rules and memories during language


development

(B)

the natural tendency children have to confuse linguistic rules

(C) the forms of resistance children show to identifying new linguistic rules

(D) the methods of learning that children use in the absence of parental
feedback

(E)

the principles underlying a childs very first utterance

ANSWERS

AND EXPLANATIONS

Explanation for Correct Answer A :


Choice (A) is correct. The last paragraph describes the ways that children combine
rules and memories during language development. They pay close attention to
minor differences in pronunciation, try to apply rules to help them understand
those differences (seek a systematic basis for the difference), and
dichotomize [divide] time into past and nonpast.

Explanation for Incorrect Answer B :


Choice (B) is incorrect. The last paragraph does discuss childrens natural, or
innate, tendencies with regard to language developmentin terms of applying
language rules, not in terms of confusing those rules: Their use of the rule is
partly guided by their genes.

Explanation for Incorrect Answer C :

file://E:\\d8.htm

2006-11-12

17/18

The Official SAT Online Course

Choice (C) is incorrect. The resistance discussed in the last paragraph is not
childrens resistance to new linguistic rules but their resistance to apply the ed
past-tense rule to irregular verbs, those that form past tense differently: They
must have a built-in tendency to block the rule [that is, the ed rule] when a
competing form is found in memory.

Explanation for Incorrect Answer D :


Choice (D) is incorrect. The last paragraph describes the ways children combine
rules and memories during language development. The statement that refers to the
absence of usable feedback from their parents means that children must have
parental feedback to apply the blocking principle. In other words, the statement
describes learning with parental feedback, not learning without parental feedback.

Explanation for Incorrect Answer E :


Choice (E) is incorrect. The last paragraph describes the ways that children combine
rules and memories during language development. The principles do not deal with a
childs first words but with a childs first past-tense error and the first use of a
correct irregular past-tense form.

19

Which of the following best characterizes the authors attitude in the passage toward
childrens errors in language development?
(A)

scientific interest

(B)

personal fascination

ed

(C) scholarly indifference

(D) general confusion

tempered frustration

AND EXPLANATIONS

is
t

ANSWERS

er

(E)

Explanation for Correct Answer A :


Choice (A) is correct. The authors scientific interest in childrens language
errors is evidenced by the passages vocabulary and sentence structure,
examples, and references to authorities. The vocabulary and sentence structure
reveal a scholarly, serious tone: The errors are acts of creation in which children
lift a pattern from their brief experience and apply it with impeccable logic to new
words, unaware that the adult world treats them as arbitrary exceptions.
Although the tone is serious, the examples used to illustrate points show that the
author likes the subject of the language development of children: Grammatical
errors like bleeded and singed have long epitomized the innocence and freshness of
childrens minds. The author also cites authorities on the subject of childrens
language development: Eric Lenneberg, Noam Chomsky, psychology textbooks, and
results from one of the authors own studies.

eg

nR

Explanation for Incorrect Answer B :


Choice (B) is incorrect. The author is fascinated by the subject, but the authors
interest goes far beyond simple fascination. The authors scientific interest in
childrens language errors is evident from vocabulary and sentence structure,
examples, and references to authorities.

Explanation for Incorrect Answer C :


Choice (C) is incorrect. The authors attitude is scholarly but far from indifferent.
The authors scientific interest is evident not only from the scholarly vocabulary,
sentence structure, and use of authoritative references but also from the results of
the authors own study. Someone who is indifferent would not likely carry out
studies of the subject.

Explanation for Incorrect Answer D :


Choice (D) is incorrect. Although the author states that the simplicity of these
errors is deceptive and that it is hard to explain why children stop making those
errors, these statements do not mean that the author is confused by the subject.
On the contrary, these statements indicate why the subject is worthy of scientific
interest. The authors scientific interest is evident not only from the scholarly
vocabulary, sentence structure, and use of authoritative references but also from
the results of the authors own study.

Explanation for Incorrect Answer E :


Choice (E) is incorrect. The authors attitude toward the subject is not tempered

file://E:\\d8.htm

2006-11-12

18/18

The Official SAT Online Course


frustration. In fact, the author not only likes the subject (the author states that
childrens grammatical errors like bleeded and singed have long epitomized the
innocence and freshness of childrens minds) but has also performed scientific
research on the subject.

Back to Score Report

Privacy Policy

Copyright 2006 The College Board. All rights reserved.

Terms of Use

Contact Us

ed

er

is
t

eg

nR

file://E:\\d8.htm

2006-11-12

1/15

The Official SAT Online Course

Help | Profile | My Organizer | My Bookmarks | Logout

Answers and Explanations

Back to Score Report

Test Sections

Section 1

View Answers and Explanations

Section 2

Online - Practice Test #4

Section 4

Section 5

Section 6

rolls of wrapping
Which of the following represents the total cost, in dollars, of
each?
greeting cards that cost
each and
paper that cost
(A)

Section 7

Section 8

(B)

Section 9

(C)

Section 10

(D)
(E)

AND EXPLANATIONS

Explanation for Correct Answer B :

ed

ANSWERS

Choice (B) is correct. Since a roll of wrapping paper costs

greeting cards is

dollars, of

the cost, in

Therefore, the total cost, in dollars, of

greeting cards is

rolls

is

of wrapping paper and

Since a greeting card costs

te
r

rolls of wrapping paper is

of

the cost, in dollars,

nR
eg

Explanation for Incorrect Answer A :

Choice (A) is not correct.

would be the cost, in dollars, of

greeting

each.
rolls of wrapping paper that cost
each and
cards that cost
rolls of wrapping
However, the question asks for the total cost, in dollars, of
greeting cards.
paper and

Explanation for Incorrect Answer C :


Choice (C) is not correct. The total cost, in dollars, is not found by adding the cost
of a roll of wrapping paper and the cost of a greeting card and then multiplying the
sum by the total number of items bought.

Explanation for Incorrect Answer D :


Choice (D) is not correct. The total cost, in dollars, is not found by adding the cost
of a roll of wrapping paper and the cost of a greeting card, multiplying the sum by
the total number of items bought, and then dividing by

Explanation for Incorrect Answer E :

but the total cost,

Choice (E) is not correct.

in dollars, of

rolls of wrapping paper and

greeting cards is only

then

If

could be which of the following?

(A)
(B)
(C)
(D)
(E)

file://E:\\d9.htm

2006-11-12

2/15

The Official SAT Online Course

ANSWERS

AND EXPLANATIONS

Explanation for Correct Answer A :

by

and so we can multiply both sides of

Of the choices, only

which gives

without changing the

We can multiply both sides of this last

direction of the inequality. This gives

inequality by

It follows that

then

Choice (A) is correct. If

is less than

Explanation for Incorrect Answer B :

would imply that

then

were equal to

Choice (B) is not correct. If

Therefore,

But

was less than

Explanation for Incorrect Answer C :

But

was less than

Therefore,

would imply that

were equal to

Choice (D) is not correct. If

was less than

But

cannot be

eg
is
te
r

Explanation for Incorrect Answer D :

ed

would imply that

then

were equal to

Choice (C) is not correct. If

cannot be

then

Therefore,

cannot be

Explanation for Incorrect Answer E :

Choice (E) is not correct. If

nR

would imply that

was less than

But

then

Therefore,

cannot be

U
3

were equal to

A circle was cut into two pieces, and one of the pieces is shown above, with the
dashed line indicating the path of the cut. Which of the following could be the other
piece of the circle?

(A)

(B)

file://E:\\d9.htm

2006-11-12

3/15

The Official SAT Online Course

(C)

(D)

(E)

ANSWERS

AND EXPLANATIONS

is
te
re

Explanation for Correct Answer B :


Choice (B) is correct. The piece in this choice and the first piece can be put together
to make the complete circle, as shown in the figure below.

eg

nR

Explanation for Incorrect Answer A :


Choice (A) is not correct. The piece in this choice and the first piece do not form a
circle, as shown in the figure below.

Explanation for Incorrect Answer C :


Choice (C) is not correct. The piece in this choice and the first piece do not form a
circle, as shown in the figure below.

Explanation for Incorrect Answer D :


Choice (D) is not correct. The piece in this choice and the first piece do not form a
circle, as shown in the figure below.

file://E:\\d9.htm

2006-11-12

4/15

The Official SAT Online Course

Explanation for Incorrect Answer E :


Choice (E) is not correct. The piece in this choice and the first piece do not form a
circle, as shown in the figure below.

which of the following must be true?

and

If

(A)

re

(B)

(C)
(D)

is
te

(E)

ANSWERS

AND EXPLANATIONS

nR
eg

Explanation for Correct Answer D :


and
Choice (D) is correct. If

Therefore,

It is not necessarily true that

Therefore,

or that

Adding

which simplifies to

or that

It cannot be true that

is the only one of the five statements that must

and

be true if

then

to the right side preserves the inequality.

to the left side of this inequality and

Explanation for Incorrect Answer A :


and
Choice (A) is not correct. If

it may be true that

it does not have to be true. For example, if

and

and

but

then

but

Explanation for Incorrect Answer B :

and

then

It cannot be

Explanation for Incorrect Answer C :


and
Choice (C) is not correct. If

then

It cannot be

then

It may be true

Choice (B) is not correct. If

true that

true that

Explanation for Incorrect Answer E :


and
Choice (E) is not correct. If

that

but it does not have to be true. For example, if

then

file://E:\\d9.htm

and

and

but

2006-11-12

5/15

The Official SAT Online Course

Company R makes cars in many locations around the world. The graph above shows
cars in
If the company made
where its cars were made in
how many of these cars were made in South America?
(A)

(B)

(C)

(D)

AND EXPLANATIONS

re

ANSWERS

(E)

Explanation for Correct Answer E :


Choice (E) is correct. According to the graph,

te

of

made in South America, and

of the companys cars were

is

eg
is

Explanation for Incorrect Answer A :


Choice (A) is not correct. According to the graph,

nR

were made in South America. This choice is equal to

cars.

Explanation for Incorrect Answer B :


Choice (B) is not correct. According to the graph,

were made in South America. This choice is equal to

of the companys cars

of the companys

of the companys cars

of the companys

cars.

Explanation for Incorrect Answer C :


Choice (C) is not correct. According to the graph,

were made in South America. This choice is equal to

of the companys cars

of the companys

cars.

Explanation for Incorrect Answer D :


Choice (D) is not correct. According to the graph,

were made in South America. This choice is equal to

of the companys cars

of the companys cars.

Above are the four houses on a block of Oak Road. Each house is painted a different
color. There is a red house, which is next to a blue house. There is a green house,
which is next to the red house and a yellow house. Which house could be painted

file://E:\\d9.htm

2006-11-12

6/15

The Official SAT Online Course

red?
(A)

only

(B)

only

(C)

only

(D)

or

(E)

or

ANSWERS

AND EXPLANATIONS

Explanation for Correct Answer D :


Choice (D) is correct. The red house is next to two different houses (the blue house
could be painted red.
nor house
and the green house); thus, neither house
nor
The green house is likewise next to two different houses, so neither house
is painted blue and house
could be painted green. Thus, either house
house
is painted
is painted blue and house
is painted yellow, or vice versa. If house

which is next to the blue house, must be painted red (and

yellow, then house

house

must be green). If house

is painted yellow and house

is painted blue,

which is next to the blue house, must be painted red (and house

then house

or house

could be painted red.

ed

must be green). Therefore, either house

Explanation for Incorrect Answer A :


Choice (A) is not correct. The red house is next to two different houses (the blue
could not be the house that is painted
house and the green house); thus, house
red.

te
r

Explanation for Incorrect Answer B :


Choice (B) is not correct. It is true that house
is green, and house
is blue, house
house
could also be painted red (in this case, house
is blue).
house

is

could be painted red (in this case,


is yellow); however, house
is green, and
is yellow, house

eg

Explanation for Incorrect Answer C :


could be painted red (in this case,
Choice (C) is not correct. It is true that house
is blue); however, house
is green, and house
is yellow, house
house
is green, and
is blue, house
could also be painted red (in this case, house
is yellow).
house

nR

Explanation for Incorrect Answer E :


Choice (E) is not correct. The red house is next to two different houses (the blue
could be painted
nor house
house and the green house); thus, neither house
red.

then

If
(A)
(B)
(C)
(D)
(E)

ANSWERS

AND EXPLANATIONS

Explanation for Correct Answer A :

Choice (A) is correct. If

file://E:\\d9.htm

then

2006-11-12

7/15

The Official SAT Online Course

Explanation for Incorrect Answer B :

not

then

Choice (B) is not correct. If

This choice could be the result of an algebraic mistake.

Explanation for Incorrect Answer C :

but the question asks for

Choice (C) is not correct. The expression

the value of

Explanation for Incorrect Answer D :

not

then

Choice (D) is not correct. If

This choice could be the result of an algebraic mistake.

Explanation for Incorrect Answer E :

but the question

Choice (E) is not correct. The expression

ed

asks for the value of

is
te
r

staples that are either silver, black, or red. If


A box of staples contains
percent are black, what percent are red?
percent of the staples are silver and

(A)

(C)
(D)

nR

(E)

eg

(B)

ANSWERS

AND EXPLANATIONS

Explanation for Correct Answer A :


percent of the staples are silver and
Choice (A) is correct. If

percent are

percent
percent are either silver or black. Therefore,
black, then
of the staples are neither silver nor black. The only other staples in the box are red,
percent are red.
so

Explanation for Incorrect Answer B :


Choice (B) is not correct. This incorrect choice could be the result of an error in
computation.

Explanation for Incorrect Answer C :


Choice (C) is not correct. The percent of staples in the box that are silver is

and the percent that are black is

but the percent that are red is

Explanation for Incorrect Answer D :


Choice (D) is not correct. The percent of staples in the box that are either silver or
but the percent of staples that are red is
red is

Explanation for Incorrect Answer E :


Choice (E) is not correct. The percent of staples in the box that are not red is

file://E:\\d9.htm

2006-11-12

8/15

The Official SAT Online Course

is a right triangle, and

is a square,

In the figure above,

If

what is the probability that the

a point is chosen at random from polygon


chosen point is in the shaded region?

(A)

(B)

(C)

re

(D)

(E)

te

AND EXPLANATIONS

is

ANSWERS

Explanation for Correct Answer B :


Choice (B) is correct. The probability that the chosen point is in the shaded region
can be found by dividing the area of the shaded region by the area of the entire
then the area of the square
has length
polygon. If each side of square

eg

The shaded region is a right triangle, both of whose legs also have length

nR

is

so the area of the shaded region is

and the area of the whole polygon is

Therefore, the probability that the chosen point is in the shaded

region is

Explanation for Incorrect Answer A :


Choice (A) is not correct. The probability that the chosen point is in the shaded
region can be found by dividing the area of the shaded region by the area of the
entire polygon. The area of the shaded region is less than one half the area of the

polygon, so the probability cannot be

Explanation for Incorrect Answer C :


Choice (C) is not correct. The probability that the chosen point is in the shaded
region can be found by dividing the area of the shaded region by the area of the
entire polygon. The area of the shaded region is more than one quarter the area of

the polygon, so the probability cannot be

Explanation for Incorrect Answer D :

file://E:\\d9.htm

2006-11-12

9/15

The Official SAT Online Course

Choice (D) is not correct. The probability that the chosen point is in the shaded
region can be found by dividing the area of the shaded region by the area of the
entire polygon. The area of the shaded region is more than one fifth the area of the

polygon, so the probability cannot be

Explanation for Incorrect Answer E :


Choice (E) is not correct. The probability that the chosen point is in the shaded
region can be found by dividing the area of the shaded region by the area of the
entire polygon. The area of the shaded region is more than one sixth the area of the

polygon, so the probability cannot be

10

What is an equation of the line parallel to the


-axis?

-axis and two units above the

(A)
(B)

(C)
(D)

AND EXPLANATIONS

re

ANSWERS

(E)

Explanation for Correct Answer E :


-axis has an equation of the form
Choice (E) is correct. Any line parallel to the
is a constant. Since the line in question is two units above the
where

te

axis, it has equation

is

Explanation for Incorrect Answer A :


is a vertical line two units
Choice (A) is not correct. The line with equation
-axis must be horizontal.
-axis. A line parallel to the
to the left of the

nR
eg

Explanation for Incorrect Answer B :


is a vertical line two units to
Choice (B) is not correct. The line with equation
axis
must
be horizontal.
to
the
-axis. A line parallel
the right of the

Explanation for Incorrect Answer C :

Choice (C) is not correct. The line with equation

but it is two units below the

is parallel to the

-axis,

-axis, not above.

Explanation for Incorrect Answer D :

Choice (D) is not correct. The line with equation

is the

-axis, not the line

two units above it.

11

Which of the following graphs best represents the information in the table above?

file://E:\\d9.htm

2006-11-12

10/15

The Official SAT Online Course

(A)

(B)

(C)

re

(D)

te

is

eg

(E)

nR

ANSWERS

AND EXPLANATIONS

Explanation for Correct Answer B :


Choice (B) is correct. The graph that best represents the information in the table

should contain the

points

where

represents an age, in weeks, of the

represents the height of the plant, in


plant that is given in the table and
weeks in the table. Therefore, the graph should
centimeters, that corresponds to

contain the points

shown, choice (B) is the only one that could contain all

and

Of the graphs

of these points.

Another way to see that the graph in choice (B) is the most consistent with the
table is to see that the plants height is increasing throughout the four-week

centimeters,
and
period. The increases from week to week are
respectively; thus, the increases get smaller and smaller over the period. This
corresponds to an increasing curve that begins with a sharp rise and then levels off.
Only the graph in (B) fits this description.

Explanation for Incorrect Answer A :


Choice (A) is not correct. This graph could not represent the information in the
table, because the table shows that the increases in the plants height are large at
first and then become smaller. The graph in choice (A), however, represents a plant
whose growth follows a linear pattern, that is, whose weekly growth is constant.

Explanation for Incorrect Answer C :

file://E:\\d9.htm

2006-11-12

11/15

The Official SAT Online Course

Choice (C) is not correct. This graph could not represent the information in the
table, because the table shows that the height of the plant is increasing over the
four-week period. The graph in choice (C), however, represents a plant whose
height is decreasing.

Explanation for Incorrect Answer D :


Choice (D) is not correct. This graph could not represent the information in the
table, because the table shows that the increases in the plants height are large at
first and then become smaller. The graph in choice (D), however, represents a plant
whose weekly height increases are small at first and then become larger.

Explanation for Incorrect Answer E :


Choice (E) is not correct. This graph could not represent the information in the
table, because the table shows that the height of the plant is increasing over the
four-week period. The graph in choice (E), however, represents a plant whose
height is increasing at the beginning of the period and decreasing at the end.

12

is in the set whenever

if

A set has property

is in the set. Which of the

following sets has property


(A)

(B)

ed

(C)
(D)
(E)

eg
is
te
r

ANSWERS

AND EXPLANATIONS

Explanation for Correct Answer D :

Choice (D) is correct. The elements of this set are

and

and

and

Since

is in the set, this set has property

nR

Explanation for Incorrect Answer A :


Choice (A) is not correct. This set does not have property

element of the set but

is an

since

is an

which is not an element of the set.

Explanation for Incorrect Answer E :


Choice (E) is not correct. This set does not have property

of the set but

since

which is not an element of the set.

Explanation for Incorrect Answer C :


Choice (C) is not correct. This set does not have property

element of the set but

is an

which is not an element of the set.

Explanation for Incorrect Answer B :


Choice (B) is not correct. This set does not have property

element of the set but

since

since

is an element

which is not an element of the set.

13

file://E:\\d9.htm

2006-11-12

12/15

The Official SAT Online Course

The figure above is a right triangle. What is the value of

(A)
(B)
(C)
(D)
(E)

ANSWERS

AND EXPLANATIONS

Explanation for Correct Answer A :


Choice (A) is correct. Since the triangle is a right triangle, the Pythagorean

Expanding gives

Theorem holds:

which simplifies to

Therefore,

is
te
re
d

Explanation for Incorrect Answer B :


and
Choice (B) is not correct. Since

right triangle and

are the lengths of the legs of the

is the length of the hypotenuse, it must be true that

Therefore,

together, these imply

and

Explanation for Incorrect Answer C :


Choice (C) is not correct. Since the triangle is a right triangle, the Pythagorean

Theorem holds:

eg

Therefore,

Expanding gives

not

which simplifies to

not

Explanation for Incorrect Answer D :

nR

Choice (D) is not correct. If

were equal to

then

would be equal

But then the shorter leg of the given triangle would be of length
to
not possible.

which is

Explanation for Incorrect Answer E :


Choice (E) is not correct. Applying the Pythagorean Theorem yields

which simplifies to

But to find the

both sides of the last equation must be multiplied by

value of

Therefore,

is equal to

not

14

If

is an integer,

and

which of the following must be true?

I.

is an integer.

II.

III.

file://E:\\d9.htm

(A)

I only

(B)

III only

2006-11-12

13/15

The Official SAT Online Course

(C) I and II only

(D) I and III only

(E)

I, II, and III

ANSWERS

AND EXPLANATIONS

Explanation for Correct Answer D :

Choice (D) is correct. If

If

cannot be equal to

must be equal to
true.

were equal to

then

and if

is an integer greater than

would imply that

then

which is not possible. Therefore, statement I,

would be an integer,

were an integer, then

Statement II cannot be true. If

must be

would be an integer. However, this is not possible, because

and thus

it follows that

Since

Statement III says

er
e

Therefore, statement III must be true.

Of the three statements, therefore, only I and III must be true.

is
t

Explanation for Incorrect Answer A :


Choice (A) is not correct. Statement I must be true, but statement III must also be
true.

eg

Explanation for Incorrect Answer B :


Choice (B) is not correct. Statement III must be true, but statement I must also be
true.

nR

Explanation for Incorrect Answer C :


Choice (C) is not correct. Statement I must be true, but statement II cannot be
true.

Explanation for Incorrect Answer E :


Choice (E) is not correct. Statements I and III must be true, but statement II
cannot be true.

15

and is divided into


The circle above has an area of
the perimeter of one of these regions?

congruent regions. What is

(A)
(B)

(C)

file://E:\\d9.htm

2006-11-12

14/15

The Official SAT Online Course

(D)

(E)

ANSWERS

AND EXPLANATIONS

Explanation for Correct Answer C :


regions are congruent, so each has the same
Choice (C) is correct. The
perimeter. The part of the perimeter of each region that lies on the circle is equal to

so the radius of the

of the circumference of the circle. The circle has area

circle is

thus,

and the circumference is

the diameter is

of the

The perimeter of one of the

circumference of the circle is

of the circumference of the circle:

regions is the length of two radii plus

ed

Explanation for Incorrect Answer A :


Choice (A) is not correct. The perimeter of one of the regions cannot be

te
r

is a negative number.

because

Explanation for Incorrect Answer B :


Choice (B) is not correct. This error could be the result of incorrectly computing the
instead of
circumference of the circle to be

eg
is

Explanation for Incorrect Answer D :


Choice (D) is not correct. This error could be the result of incorrectly computing the
and also forgetting to divide the circumference
circumference of the circle to be

by

nR

Explanation for Incorrect Answer E :


Choice (E) is not correct. The curved portion of the perimeter of each of the

congruent regions is

U
16

not

percent
dollars. This price was discounted
The original price of a book was
percent off the sale price.
during a sale. James used a coupon to buy the book for
Which of the following represents the price, in dollars, that James paid for the book?
(A)

(B)

(C)

(D)

(E)

ANSWERS

AND EXPLANATIONS

Explanation for Correct Answer A :


Choice (A) is correct. The original price of the book was

file://E:\\d9.htm

dollars and this price

2006-11-12

15/15

The Official SAT Online Course

percent, so the sale price, in dollars, was

was discounted by

James used a coupon to buy the book for

percent off the sale price, so the price

James paid was

Explanation for Incorrect Answer B :


Choice (B) is not correct. This choice could be the result of an algebraic error.

therefore,

not

Remember that

was not the price James paid.

Explanation for Incorrect Answer C :


Choice (C) is not correct. This choice could be the result of an algebraic error.

not

Remember that

was not the price James paid.

therefore,

te
re
d

Explanation for Incorrect Answer D :


Choice (D) is not correct. This choice could be the result of an algebraic error.

Remember that

not

was not the price James paid.

therefore,

Explanation for Incorrect Answer E :


Choice (E) is not correct. The original price of the book was

is

dollars and this price

The

percent, so the sale price, in dollars, was

was discounted by

eg

represents the discount from the regular price to the sale

expression

was not the price James paid.

nR

price, not the sale price; therefore,

Copyright 2006 The College Board. All rights reserved.

file://E:\\d9.htm

Back to Score Report

Privacy Policy

Terms of Use

Contact Us

2006-11-12

1/10

The Official SAT Online Course

Help | Profile | My Organizer | My Bookmarks | Logout

Answers and Explanations

Back to Score Report

Test Sections

Section 1

View Answers and Explanations

Section 2

Online - Practice Test #4

Section 4

Section 5

If every nation were completely self-sufficient and operated under a free-market


economy, the world's food supply will be governed solely by the economics of supply
and demand.

Section 6

Section 7

Section 8

Section 9

(A)

will be

(B)

would have been

(C) was to be

Section 10

(D) was

would be

ANSWERS

AND EXPLANATIONS

(E)

re

Explanation for Correct Answer E :


Choice (E) is correct. It avoids the error of the original by replacing the future -tense
verb will be with the conditional form would be, which is needed in a
clause that follows a conditional if clause.

te

is

eg

Explanation for Incorrect Answer A :


Choice (A) involves an error in verb form. The if clause states a condition that
is contrary to fact. The second clause states the result of that condition, so its verb
should be conditional (would be).

nR

Explanation for Incorrect Answer B :


Choice (B) involves an error in verb tense. The present perfect tense of the verb
would have been governed is inconsistent with the past tense of the verbs
were and operated in the conditional if clause.

Explanation for Incorrect Answer C :


Choice (C) involves a shift in verb form. The first clause is a conditional if
clause that states a condition contrary to fact. The second clause states the result
of that condition, so its verb should be conditional (would be).

Explanation for Incorrect Answer D :


Choice (D) involves a shift in verb form. The first clause is a conditional if
clause that states a condition contrary to fact. The second clause states the result
of that condition, so its verb should be conditional (would be).

Gold leaf is pure gold that is hammered so thin for it to take 300,000 units to make a
stack one inch high.
(A)

for it to take

(B)

so that it takes

(C) so it takes

(D) as for it to take

(E)

that it takes

ANSWERS

AND EXPLANATIONS

Explanation for Correct Answer E :

file://E:\\d10.htm

2006-11-12

2/10

The Official SAT Online Course

Choice (E) is correct. It avoids the improper subordination of the original by


providing the second part of the subordinating conjunction so that and by
replacing the infinitive to take with the verb takes.

Explanation for Incorrect Answer A :


Choice (A) involves faulty subordination. To complete the subordinating conjunction,
that must follow so. Also, a subject and verb must follow that, and the
infinitive to take cannot serve as a verb.

Explanation for Incorrect Answer B :


Choice (B) creates redundancy. The second so repeats the first so and is
not necessary.

Explanation for Incorrect Answer C :


Choice (C) involves faulty subordination. To complete the subordinating conjunction,
the word that, not so, must follow so (so thin that ).

Explanation for Incorrect Answer D :


Choice (D) involves faulty subordination and improper phrasing. The word pair so
as is sometimes used in comparisons, but nothing is being compared in this
sentence. Also, the phrase as for it to take is not idiomatic.

re
d

By far the best-known work of poet Emma Lazarus is the sonnet New Colossus, it
was chosen to be the inscription for the pedestal of the Statue of Liberty in 1886.
(A)

it was chosen to be

(B)

choosing it to be

te

(C) which they chose for

they chose it as

eg

(E)

is

(D) which was chosen as

ANSWERS

AND EXPLANATIONS

Explanation for Correct Answer D :


Choice (D) is correct. It avoids the comma-splice error of the original by replacing
the second independent clause with a dependent clause introduced by which.

nR

Explanation for Incorrect Answer A :


Choice (A) creates a comma splice. Two independent clauses (By
Colossus and it 1886) are joined by only a comma without a
coordinating conjunction to introduce the second clause.

Explanation for Incorrect Answer B :


Choice (B) involves faulty modification, which results in faulty logic. The participial
phrase choosing 1886 has no noun that it can logically modify.

Explanation for Incorrect Answer C :


Choice (C) involves an error in pronoun use. The pronoun they has no
antecedent to which it can logically refer. None of the nouns in the main clause
(work, Emma Lazarus, sonnet, New Colossus) could logically be
said to have chosen the inscription for the Statue of Liberty.

Explanation for Incorrect Answer E :


Choice (E) creates a comma splice and includes an error in pronoun use. Two
independent clauses (By Colossus and they 1886) are joined by
only a comma without a coordinating conjunction to introduce the second clause.
The pronoun they has no antecedent to which it can logically refer. None of the
nouns in the main clause (work, Emma Lazarus, sonnet, New
Colossus) could logically be said to have chosen the inscription for the Statue of
Liberty.

file://E:\\d10.htm

2006-11-12

3/10

The Official SAT Online Course

The gong was believed to have originated in Western Asia, reached China in the sixth
century, where it continues to be used for a wide range of purposes, including as a
military signal, a rhythmic accompaniment for vocal performance, and a ritual
instrument.
(A) The gong was believed to have originated in Western Asia, reached

(B)

The gong, it is believed, originated in Western Asia, reached

(C) The gong, believed to have originated in Western Asia, reached

(D) The gong is believed to originate in Western Asia, and reaching

(E)

The gong, they believe, has originated in Western Asia, reaching

ANSWERS

AND EXPLANATIONS

Explanation for Correct Answer C :


Choice (C) is correct. It avoids the faulty coordination of the original by replacing
the verb was believed with the participle believed.

Explanation for Incorrect Answer A :


Choice (A) involves faulty coordination and an error in verb tense. The compound
verbs was believed and reached are separated by a comma and are not
connected by a coordinating conjunction (and), which is needed to connect
compound verbs. Also, the past-tense verb was believed should be the
present-tense verb is believed because the action of the sentence is still
occurring in the present time.

te
re
d

Explanation for Incorrect Answer B :


Choice (B) involves faulty coordination. The compound verbs originated and
reached are separated by a comma and are not connected by a coordinating
conjunction (and), which is needed to connect compound verbs.

Explanation for Incorrect Answer D :


Choice (D) involves errors in verb tense and parallelism. The present tense of the
infinitive to originate is not logical because the action of the sentence occurred
in the past and before the action of the main verb, is believed. Also, the two
grammatical units connected by the coordinating conjunction and should be
parallel, but they are not.

eg
is

nR

Explanation for Incorrect Answer E :


Choice (E) involves errors in pronoun use and verb tense. The pronoun they
has no antecedent to which it can logically refer. Also, the separated verbs
believe and has originated should be combined to read believed to have
originated, and reaching should be changed to reached.

U
5

Galileo was probably the first to discover Saturns rings, although his telescope was
so small, he could not see the rings properly and assumed that they were satellites.
(A)

although

(B)

but because

(C) since

(D) and

(E)

nevertheless

ANSWERS

AND EXPLANATIONS

Explanation for Correct Answer B :


Choice (B) is correct. It avoids the faulty subordination of the original. In the
original, the clause introduced by although could modify either of the other two
clauses (Galileo rings and he satellites), so the meaning of the
sentence is unclear. The coordinating conjunction but, and the subordinating
conjunction because, clearly show the relationship between the ideas in the
sentence.

file://E:\\d10.htm

2006-11-12

4/10

The Official SAT Online Course

Explanation for Incorrect Answer A :


Choice (A) involves faulty subordination. The clause introduced by although
could modify either of the other two clauses (Galileo rings and he
satellites), so the meaning of the sentence is unclear.

Explanation for Incorrect Answer C :


Choice (C) involves faulty subordination. The clause introduced by since could
modify either of the other two clauses (Galileo rings and he
satellites), so the meaning of the sentence is unclear.

Explanation for Incorrect Answer D :


Choice (D) involves faulty coordination. The conjunction and does not clearly
express the relationship between the two independent clauses (Galileo rings
and his satellites).

Explanation for Incorrect Answer E :


Choice (E) creates a comma splice. The word nevertheless cannot serve as a
coordinating or subordinating conjunction, so three independent clauses (Galileo
rings, nevertheless small, and he satellites) are connected only
by commas, without a coordinating conjunction to introduce the third clause.

(A)

people, widely

(B)

people, which are widely

(D) people will be widely

people are widely

ANSWERS

is
t

(E)

er

(C) people, they are widely

ed

The Olmec people, widely regarded as the creators of the first civilization in
Mesoamerica, the area encompassing much of Mexico and Central America.

AND EXPLANATIONS

eg

Explanation for Correct Answer E :


Choice (E) is correct. It avoids the sentence fragment error of the original by
providing a main verb (regarded) for the subject of the sentence. Without this
change, the subject of the sentence, The Olmec people has no verb to
complete its action.

nR

Explanation for Incorrect Answer A :


Choice (A) creates a sentence fragment. Neither participle, regarded or
encompassing, can serve as the main verb in this sentence, so the action of
the sentence cannot be completed. A main verb, are regarded, is needed
instead.

Explanation for Incorrect Answer B :


Choice (B) creates a sentence fragment. The relative pronoun which turns all
that follows it into a dependent clause, so the subject of the sentence (people)
has no verb to complete the action of the sentence.

Explanation for Incorrect Answer C :


Choice (C) involves unnecessary words. The pronoun they is not needed. The
phrase The Olmec people properly serves as the subject of the sentence.

Explanation for Incorrect Answer D :


Choice (D) involves an error in verb tense. The future tense of the verb will be
regarded can not express an action that is occurring in the present time.

Although their language and people that are not of European origin, Finland is
generally considered part of Scandinavia, which also includes Norway, Sweden, and
Denmark.
(A)

file://E:\\d10.htm

their language and people that are

2006-11-12

5/10

The Official SAT Online Course

(B)

it has a language and people that are

(C) they have languages and people

(D) its language and people are

(E)

there are languages and people that are

ANSWERS

AND EXPLANATIONS

Explanation for Correct Answer D :


Choice (D) is correct. It avoids the error of the original by replacing the plural
pronoun their with the singular pronoun its (to properly refer to the
singular noun Finland) and by omitting the relative pronoun that, which is
not needed.

Explanation for Incorrect Answer A :


Choice (A) involves errors in pronoun use. The plural pronoun their incorrectly
refers to the singular noun Finland, and the relative pronoun that is not
needed.

re
d

Explanation for Incorrect Answer B :


Choice (B) involves wordiness and an error in pronoun use. The use of the relative
pronoun that makes the meaning of the sentence unclear. The relative clause
beginning with that could modify language and people or people
alone. In addition, the awkwardly phrased clause it has a language and people
that are can be reduced to the simpler, more idiomatic, phrase its language
and people are.

te

Explanation for Incorrect Answer C :


Choice (C) involves an error in pronoun use. The plural pronoun they
incorrectly refers to the singular noun Finland.

is

eg

Explanation for Incorrect Answer E :


Choice (E) involves wordiness and faulty logic. There is no reason to use the clause
there are languages and people that are instead of the simpler phrase its
language and people are. Also, the use of that clause does not clearly connect
Finland and Finlands language and people.

nR

Unlike the hollow body of an acoustic guitar, which acts as a sound box to project
sound, the solid-body electric guitar is almost soundless without the aid of an
amplification system.

(A)

the hollow body of an acoustic guitar, which acts

(B)

the hollow body that an acoustic guitar has, it acts

(C) an acoustic guitars hollow body, acting

(D) an acoustic guitar, whose hollow body acts

(E)

an acoustic guitar and its hollow body, acting

ANSWERS

AND EXPLANATIONS

Explanation for Correct Answer D :


Choice (D) is correct. It avoids the faulty comparison of the original by comparing
an acoustic guitar with an electric guitar, instead of comparing the hollow body of
an acoustic guitar with an electric guitar.

Explanation for Incorrect Answer A :


Choice (A) involves faulty comparison. It compares the hollow body of an acoustic
guitar with an electric guitar, instead of comparing an acoustic guitar with an
electric guitar.

Explanation for Incorrect Answer B :


Choice (B) creates an error in pronoun use and a comma splice. The pronoun it

file://E:\\d10.htm

2006-11-12

6/10

The Official SAT Online Course

has no logical antecedent to which it can refer. Also, the clauses Unlike
sound and the solid-body electric system are connected by only a comma
without a conjunction to introduce the second clause.

Explanation for Incorrect Answer C :


Choice (C) involves faulty comparison. It compares the hollow body of an acoustic
guitar with an electric guitar, instead of comparing an acoustic guitar with an
electric guitar.

Explanation for Incorrect Answer E :


Choice (E) involves faulty comparison. It compares an acoustic guitar, as well as its
hollow body, with an electric guitar.

Fireflies, or lightning bugs, are actually soft-bodied beetles, most of whom producing
light in special organs located in the undersides of their abdomens.

(A)

of whom producing

(B)

of which produce

(C) of them produce

(D) produce

to produce

ANSWERS

AND EXPLANATIONS

(E)

re

Explanation for Correct Answer B :


Choice (B) is correct. It avoids the errors of the original by correctly using the
relative pronoun which to refer to things (Fireflies), and by replacing the
participle producing with the plural verb produce, which agrees with the
plural subject of the clause, most.

te

is

eg

Explanation for Incorrect Answer A :


Choice (A) contains errors in pronoun use and verb form. The relative pronoun
whom should be used to refer to people, not things, and the clause (most
abdomens) needs a verb instead of the participle producing to complete the
action of the clause.

nR

Explanation for Incorrect Answer C :


Choice (C) creates a comma splice. The two clauses (Fireflies beetles and
most abdomens) are connected by only a comma without a conjunction to
introduce the second clause.

Explanation for Incorrect Answer D :


Choice (D) creates a comma splice. The two clauses (Fireflies beetles and
most abdomens) are connected by only a comma without a conjunction to
introduce the second clause.

Explanation for Incorrect Answer E :


Choice (E) creates a sentence fragment. The infinitive to produce cannot serve
as a verb for the clause beginning with most.

10

To satisfy the curious, to protect their market, and it discouraged competitors when
the earliest spice traders spread fantastic tales of spices growing in shallow lakes
guarded by winged animals and in deep glens infested with poisonous snakes.
(A)

it discouraged competitors when

(B)

it discouraged competitors,

(C) discouraging competitors made

(D) to discourage competitors, when

(E)

to discourage competitors,

ANSWERS

file://E:\\d10.htm

AND EXPLANATIONS

2006-11-12

7/10

The Official SAT Online Course

Explanation for Correct Answer E :


Choice (E) is correct. It avoids the faulty parallelism of the original by replacing the
third grammatical unit connected by and with an infinitive phrase, which is
parallel to the first two infinitive phrases (To curious and to market).

Explanation for Incorrect Answer A :


Choice (A) involves faulty parallelism that results in a sentence fragment. The three
grammatical units connected by and should be parallel. The clause it
when is not parallel to the two infinitive phrases (To curious and to
market). In addition, the word when creates a sentence fragment and so
should be deleted.

Explanation for Incorrect Answer B :


Choice (B) involves faulty parallelism. The three grammatical units connected by
and should be parallel. The clause it competitors is not parallel to the
two infinitive phrases (To curious and to market).

Explanation for Incorrect Answer C :


Choice (C) involves faulty parallelism and faulty logic. The three grammatical units
connected by and should be parallel. The participle discouraging
competitors is not parallel with the two infinitive phrases (To curious and
to market). Also, the verb made illogically suggests that the
competitors made the spice traders spread tales.

re
d

Explanation for Incorrect Answer D :


Choice (D) involves a sentence fragment. The adverb when turns everything
that follows it into a subordinate clause, which cannot stand alone as a complete
sentence.

te

11

is

The most notorious quick-draw gunman of the Texas frontier, John Wesley Hardins
exploits were written about in an autobiography published posthumously, The Life of
John Wesley Hardin as Written by Himself.

eg

(A)

John Wesley Hardins exploits were written about

(B)

John Wesley Hardins exploits are recorded

(C) John Wesley Hardin wrote about his exploits

nR

(D) the exploits of John Wesley Hardin were written about

(E)

the exploits of John Wesley Hardin are recorded

ANSWERS

AND EXPLANATIONS

Explanation for Correct Answer C :


Choice (C) is correct. It is the only option that provides a noun (John Wesley
Hardin) that can logically be modified by the opening phrase, The most
notorious quick-draw gunman of the Texas frontier. In the original, John
Wesley Hardins is a possessive that modifies exploits and creates faulty
logic. It is not logical to say that exploits were The most notorious quickdraw gunman of the Texas frontier.

Explanation for Incorrect Answer A :


Choice (A) involves an error in modification that results in faulty logic. The
possessive John Wesley Hardins modifies exploits, and it is not logical to
say that exploits were The most notorious quick-draw gunman of the Texas
frontier.

Explanation for Incorrect Answer B :


Choice (B) involves an error in modification that results in faulty logic. The
possessive John Wesley Hardins modifies exploits, and it is not logical to
say that exploits were The most notorious quick-draw gunman of the Texas
frontier.

Explanation for Incorrect Answer D :


Choice (D) involves an error in modification that results in faulty logic. It is not

file://E:\\d10.htm

2006-11-12

8/10

The Official SAT Online Course

logical to say that exploits were The most notorious quick-draw gunman of
the Texas frontier.

Explanation for Incorrect Answer E :


Choice (E) involves an error in modification that results in faulty logic. It is not
logical to say that exploits were The most notorious quick-draw gunman of
the Texas frontier.

With one of the most successful African American businesses in history, the Motown
Record Corporation was founded in 1959 in Detroit by Berry Gordy, Jr.

12

(A)

With one

(B)

It was one

(C) Being one

(D) One

(E)

As one

ANSWERS

AND EXPLANATIONS

is
te
re

Explanation for Correct Answer D :


Choice (D) is correct. It avoids the faulty parallelism of the original by omitting the
preposition With, so that the appositive (One history) is parallel, or
grammatically equal, to the noun it renames (the Motown Record Corporation).

Explanation for Incorrect Answer A :


Choice (A) involves faulty parallelism. The opening phrase renames the noun
Motown Record Corporation and should be grammatically parallel, or equal, to
the noun it renames. The opening phrase is a prepositional phrase (introduced by
With), which cannot serve as a noun.

eg

Explanation for Incorrect Answer B :


Choice (B) creates a comma splice. Two independent clauses (It history and
the Motown Jr.) are connected by only a comma, without a coordinating
conjunction to introduce the second clause.

nR

Explanation for Incorrect Answer C :


Choice (C) involves faulty parallelism. The opening phrase renames the noun
Motown Record Corporation and should be grammatically parallel, or equal, to
the noun it renames. The opening phrase is a participial phrase (introduced by
Being), which cannot serve as a noun.

Explanation for Incorrect Answer E :


Choice (E) involves faulty parallelism. The opening phrase renames the noun
Motown Record Corporation and should be grammatically parallel, or equal, to
the noun it renames. The opening phrase is a prepositional phrase (introduced by
As), which cannot serve as a noun.

13

Adult ladybugs often prey on agricultural or garden pests, causing many fruit growers
to consider them among the most beneficial insects.
(A)

causing many fruit growers to

(B)

therefore, many fruit growers

(C) this causes many fruit growers to

(D) which cause many fruit growers to

(E)

many fruit growers

ANSWERS

AND EXPLANATIONS

Explanation for Correct Answer A :


Choice (A) is correct. It avoids the errors of the other options by providing a
participial phrase causing to to appropriately modify the noun ladybugs.

file://E:\\d10.htm

2006-11-12

9/10

The Official SAT Online Course

Explanation for Incorrect Answer B :


Choice (B) creates a comma splice. Two independent clauses should not be linked
by only a comma.

Explanation for Incorrect Answer C :


Choice (C) creates a comma splice. Two independent clauses should not be linked
by only a comma.

Explanation for Incorrect Answer D :


Choice (D) creates an error in pronoun reference. The relative pronoun which
has no noun to which it can logically refer.

Explanation for Incorrect Answer E :


Choice (E) creates a comma splice. Two independent clauses should not be linked
by only a comma.

Environmentally hazardous contamination of the water and air is one of the more
serious and complex issues facing urban centers in the United States, where they
have an effect on public health, property values, and the quality of life is magnified by
the density of population.
(A)

they have an

(B)

their

14

re

(C) an

(D) it has an

ANSWERS

te

its

AND EXPLANATIONS

is

(E)

Explanation for Correct Answer E :


Choice (E) is correct. It avoids the error of the original by replacing the plural
pronoun they with the singular pronoun its, which correctly refers to the
singular noun contamination.

nR
eg

Explanation for Incorrect Answer A :


Choice (A) involves an error in pronoun agreement. The plural pronoun they
incorrectly refers to the singular noun contamination. The singular pronoun
its is needed instead.

Explanation for Incorrect Answer B :


Choice (B) involves an error in pronoun agreement. The plural pronoun their
incorrectly refers to the singular noun contamination.

Explanation for Incorrect Answer C :


Choice (C) involves an error in word choice, which creates faulty logic. The article
an is used instead of the pronoun its to refer to the noun
contamination and does not make clear that the effect of contamination is what
is magnified.

Explanation for Incorrect Answer D :


Choice (D) involves wordiness and awkward phrasing. The phrase it has an
should be reduced to the simpler, more idiomatic its to indicate that the
density of population magnifies the effect of hazardous contamination on three
things: public health, property values, and the quality of life.

Back to Score Report

Copyright 2006 The College Board. All rights reserved.

file://E:\\d10.htm

Privacy Policy

Terms of Use

Contact Us

2006-11-12

10/10

The Official SAT Online Course

ed

eg
is
te
r

nR

file://E:\\d10.htm

2006-11-12

You might also like